Sei sulla pagina 1di 234

Commented [AL1]:

Prepared by: Dr. Ali Saleim Lashhab


University of Al-Mergib

1
Revised by: Prof. Mohamed Benghazil
University of Tripoli

DEDICATION

To the pure sole of Mr. Fathi Khalifa Sneid and to my patients who are always
teaching me something new

‫ رحمه هللا وأدخله فسيح جناته‬/‫فتح خليفه اسنيد‬


‫ي‬ ‫اىل روح الفقيد السيد‬

Acknowledgement

2
First of all, I praise Allah almighty who has helped me to finish this work
satisfactorily.

An acknowledgement of deep gratitude goes to Professor Mohamed Benghazil ,


head of the dermatology department at Tripoli Medical Centre and a teaching staff
member at Tripoli university who did not hesitate to go over the work and supply
me with significant suggestions from which I benefited a lot in this work.

My deepest appreciation and gratitude also extended to my colleagues at Sok-


Elkhamis Hospital, Tripoli Central Hospital, Tripoli Medical Centre and Bir-Osta-
Millad Hospital for their encouragement and support and also to my wife and
children for their endless patience.

A special thanks to the all member teaching staff of dermatology at the University
of Tripoli who helped me applying these questions.

Thanks extend to Engineer Jebril Ahmed Shehub, who helped me a lot in the
computer skills.

At last, a special apologize to any one for any mistake in this book.

3
Choose the only one correct answer

Q1. The followings are true about the skin except?


a. The surface area of an adult skin is about 1.5m2
b. The sebaceous glands are present all over the skin surface
c. The sub-cutis is composed predominantly of fat
d. The basal cell layer is generally one cell thick
e. The epidermis is thickest on the palms and soles and it is very thin on the
eyelid.

Q2. The function of the skin include the followings except?


a. Protection of the internal organs against injury
b. Thermoregulation
c. Absorption of ultraviolet radiation and production of vitamin A
d. It is the mirror of the internal organ
e. It is an organ of beauty and expression.

Q3. The following cells are present in the epidermis except?


a. Melanocytes
b. Mast cells
c. Keratinocytes
d. Langerhans cells
e. Merkle cells.

Q4. The following cells are present in the dermis except?


a. Fibroblasts
b. Keratinocytes
c. Mast cells
d. Lymphocytes
e. Macrophages.

Q5. The followings are true regarding eccrine sweat gland except?
a. It is found in abundance throughout the skin surface
b. It is found in highest concentration on the palms, soles and in the axilla
c. It is not found in the mucus membrane such as the lips
d. It is important in heat regulation
e. It secretes the sebum, which lubricates and protects the hair and skin.

Q6. The followings are true regarding apocrine sweet glands except?
a. They are found predominantly in the ano-genital region and axilla
b. They become functionally active at puberty
c. In man, it serve no known function
d. In animals, it has a protection as well as sexual functions
4
e. They are important in heat regulation.

Q7. The followings are true regarding sebaceous glands except?


a. They are present on the whole parts of the body except on the palms and soles
b. They empty into the outer portion of the hair follicle
c. They are found in great number on extrimities
d. The function of these glands to produce an oil (sebum)
e. The sebum lubricate the hair, skin and control of epidermal water loss.

Q8. The followings are true regarding the nails except?


a. The root is overlapped by the proximal nail fold
b. The lunula is usually completely visible in the thumb nail but it’s completely covered
in the 5th nail
c. Nail grow about 1 mm per day
d. Nails growing faster in the summer than the winter
e. The finger nails growing faster than toenails.

Q9. The function of the keratinocyte include the followings except?


a. Production of keratin
b. Play a role in the immune function of the skin
c. Release of interleukin-1 which aid in T-cell activation
d. Macrophage of the epidermis
e. Producing several cytokines that are involves in inflammatory process.

Q10. The followings are true regarding melanocyte cells except?


a. Highly dendritic
b. They are wedged between keratinocyte in ratio of approximately 1:10
c. It is derived from neural crest
d. The number of melanocyte in the epidermis is the same regardless of race
or color
e. It is usually find only by electron microscope.

Q11. The followings are true regarding Langerhans cells except?


a. It is normally found scattered among keratinocyte of the prickle cell layer
of the epidermis
b. It is also found in the dermis
c. It appears as dendritic cell in section impregnated with gold chloride
d. It is derived from the bone marrow
e. It distinguished by containing up-to 1000 granules.

Q12. The followings are true in psoriasis except?


a. It may commence at any age
b. It may be symptomless or severely itchy
c. It affects males more than females

5
d. It is characterized by symmetrical, well-defined red plaques with silvery
scale
e. A strong association with human leukocyte antigen (HLA) B13 and B17.

Q13. The followings are true in psoriasis except?


a. Emotional stress, drugs and trauma are precipitating factors
b. Generalized pustular psoriasis is a life threatening disorder
c. Onycholysis and psoriatic nails are best treated with antifungal
d. Negative rheumatoid factor is infavor of psoriatic arthritis
e. Antibiotic and emollients are the treatment of choice for guttate psoriasis.

Q14. The followings are true in psoriasis except?


a. Alcohol, antimalarial and lithium may exacerbate the disease
b. Excess ultraviolet irradiation may produce a Koebner phenomenon
c. Systemic steroid should never use to treat psoriasis vulgaris
d. One-third of patients recall positive family history of psoriasis
e. Guttate psoriasis commonly affects middle aged and old patients.

Q15. The followings are true in psoriasis except?


a. Streptococcal infection is a provoking agent for guttate psoriasis
b. It may first appear after parturition
c. Joint involvement occurs in a majority of patients who have psoriasis of the
skin and nails
d. Generalized pustular psoriasis can be precipitated by hypocalcaemia
e. Skin lesions precede arthritis in the majority of cases.

Q16. The followings are true in psoriasis except?


a. Increased epidermal cell proliferation in psoriasis may be the result
of increased recruitment of resting cells
b. Toenails are more often affected than fingernails
c. Psoriatic arthritis affecting the distal interphalangeal joints is usually
associated with nail dystrophy
d. Nail pitting is the most common finding in nail psoriasis
e. Acantholytic cells are frequent findings in histopathology.

Q17. Accepted clinical features of psoriasis include the followings except?


a. Mucosal lesions are rare
b. Pitting of nail
c. Clubbing of the nails
d. Development of a typical lesion at the site of an operation scar
e. Erythematous papules and or plaques with silvery scales.

Q18. In guttate psoriasis, the followings are true except?


a. It is a common in adolescence or young adult
b. It commence two to three weeks after a sever streptococcal throat
infection despite adequate antibiotic therapy
6
c. It appears as very small (drop-like) papules all over the trunks and limbs
d. Topical and systemic steroid are the treatment of choice
e. The condition resolves completely in most of cases within four months.
Q19. In pustular psoriasis, the followings are true except?
a. The patient feel ill, has a fever and leukocytosis
b. Extensive sheets of small, sterile, yellow pustules cover the skin
c. Secondary bacterial infection may complicate the disease
d. large amount of topical steroid may precipitate the disease if they are
reduced or withdrawn too quickly
e. Methotrexate and etretinate are not used in the treatment.

Q20. In erythrodermic psoriasis, the followings are true except?


a. It may be an end stage of sub-acute or acute psoriasis
b. The redness results from capillary proliferation
c. Such patients are unwell with a fever and leukocytosis
d. It may be complicated by central hypothermia and heart failure
e. Treated by emollient and topical steroid.

Q21. In nail psoriasis, the followings are true except?


a. Pitting is a constant feature
b. There may be subungual hyperkeratosis
c. Nail pterygium
d. Onycholysis
e. Oil drop discoloration.

Q22. Psoriatic arthropathy is characterized clinically by the followings except?


a. Joint involvement occurs in a minority of patient
b. It affects commonly the proximal interphalangeal joints
c. It may occur without any cutaneous evidence of the disease
d. It is usually seronegative
e. Methotrexate is the drug of choice.

Q23. The following drugs may exacerbate psoriasis except


a. Chloroquine
b. Thiazide
c. Alcohol consumption
d. Lithium
e. Propranolol.

Q24. The following drugs may exacerbate psoriasis except


a. Aspirin
b. Paracetamol
c. Beta blocker
d. Interferon
e. Indomethacin.

7
Q25. The histological feature of psoriasis include the followings except
a. Acanthosis of the epidermal ridges
b. Hyperkeratosis and parakeratosis
c. Peri-vascular lymphocytic infiltrate in the upper dermis
d. Munro-microabscess which represent an accumulation of polymorphs in
the stratum corneum are characteristic
e. Band-like infiltrate of lymphocyte in the dermis.

Q26. Local treatment of psoriasis include the followings except


a. Emollient
b. Keratolytics
c. Topical steroids
d. Benzoyl peroxide
e. Vitamine D analogue.

Q27. Systemic treatment of psoriasis include the followings except


a. PUVA
b. Retinoids
c. Systemic tetracycline
d. Cytotoxic drugs (methotrexate)
e. Immunosuppression (cyclosporine).

Q28. Indication for systemic treatment of psoriasis include the followings except
a. Erythrodermic psoriasis
b. Generalized pustular psoriasis
c. Psoriatic arthropathy
d. Scalp psoriasis
e. Extensive psoriasis vulgaris.

Q29. The clinical picture of lichen planus include the followings except
a. Papules and or plaques commonly affect the trunk and extrimities
b. Associated with severe itching
c. Mucus membrane lesion usually painless
d. May be caused by antimalarial drugs
e. Nails involvement are very rare.

Q30. In lichen planus, the followings are true except


a. Mucous membrane lesions commonly involve the tongue
b. Typical lesions are shiny papules with overlying Wickham’s striae
c. Annular lesions are common on the glans penis
d. Thinning of the nail plate is the commonest nail change
e. Alopecia as a complication is usually non-scarring.

8
Q31. In lichen planus, the followings are true except
a. Can cause scarring alopecia
b. It is commonly associated with erythema nodosum
c. Exhibits the koebner phenomenon
d. May be associated with chronic active hepatitis
e. May be drug induced.

Q32. In lichen planus, the followings are true except


a. The hypertrophic form is a risk factor for malignant transformation
b. It may occur as a contact reaction to colour film developer
c. Requires high-dose of cyclosporine to attain remission
d. It may result from treatment with chlorothiazide
e. Pruritus occurs in the majority of patients.

Q33. Nail changes in lichen planus include the followings except


a. Longitudinal lines in the nail plate
b. Depression in the nail plate
c. Permanent destruction of the nail
d. Subungual hyperkeratosis
e. Pterygium formation.

Q34. The following drugs may cause lichenoid eruption except


a. Antimalarials drugs
b. Penicillamine
c. Dapsone
d. Arsenic
e. Gold.

Q35. Topical treatment of lichen planus include the followings except


a. Emollient
b. Topical steroid
c. Topical antihistamine
d. Intralesional steroid
e. Keratolytics (salicylic acid).

Q36. Systemic treatment of lichen planus include the followings except


a. Phototherapy
b. Retinoid
c. Re-PUVA
d. Low molecular weight heparin
e. Systemic antimalarial drugs.

9
Q37. In the herald patch of pityriasis rosea, the followings are true except
a. It may be a round to oval patch or plaque
b. It may be single or multiple
c. Commonly located on the face and extrimities
d. It may occur anywhere on the body
e. It is followed after few days by an eruption of similar but much smaller lesion over
the trunk.

Q38. In pityriasis rosea, the followings are true except


a. Common and affects women a little more frequently than men
b. Seen more frequently in the autumn and winter
c. The lesion occurs in a symmetrical distribution
d. The eruption usually chronic and last a number of months
e. The herald patch is sometimes absent.

Q39. In pityriasis rosea, the followings are true except


a. It is most commonly occurs in young adults
b. It may be mistaken for guttate psoriasis
c. Typically, it lasts between 4 to 6 weeks
d. Often cause prolonged post-inflammatory hypopigmentation
e. It may be treated by emollient and topical steroid.

Q40. Differential diagnosis of pityriasis rosea include the followings except


a. Tinea corporis
b. Atopic dermatitis
c. The rash of the 2ry stage of syphilis
d. Guttate psoriasis
e. Drug eruption.

Q41. The treatment of pityriasis rosea include the followings except


a. Emollient
b. UVB therapy
c. Topical steroid
d. Systemic steroid
e. Systemic acyclovir.

Q42. The followings are causes of erythroderma except


a. Psoriasis
b. Dermatitis
c. Drug reaction
d. Lymphoma
e. Erythema nodosum.

10
Q43. Causes of erythroderma include the followings except
a. Cutaneous T-cell lymphoma
b. Pemphigus foliaceus
c. Norwegian scabies
d. Pityriasis rubra pilaris
e. Pityriasis rosacea.

Q44. Nail changes may be seen in the following skin diseases except
a. Psoriasis
b. Lichen planus
c. Alopecia areata
d. Pityriasis rosea
e. Fungal infection.

Q45. Onycholysis of the nails occur in the following skin diseases except
a. Psoriasis
b. Lichen planus
c. Fungal infection
d. Local trauma
e. Eczema.

Q46. Koebner’s Phenomenon is present in the following skin diseases except


a. Pityriasis alba
b. Viral warts
c. Psoriasis
d. Lichen planus
e. Molluscum contagiosum.

Q47. The followings are the characteristic features of eczema except


a. Delayed hypersensitivity reaction in allergic contact dermatitis
b. The level of IgE concentration is raised in all patients of atopic dermatitis
c. Epidermal oedema and intra-epidermal vesicles
d. Asthma, hay fever and urticaria are commonly associated with atopic
eczema
e. Atopic patients have increased susceptibility to viral and bacterial
infection.

Q48. These are examples of endogenous types of eczema except


a. Atopic dermatitis
b. Seborrhoeic dermatitis
c. Phytophotodermatitis
d. Discoid eczema
e. Pompholyx .

11
Q49. These are examples of exogenous types of eczema except
a. Irritant contact dermatitis
b. Allergic contact dermatitis
c. Stasis dermatitis
d. Photo-contact dermatitis
e. Phyto-contact dermatitis.

Q50. In dermatitis, the followings are true except


a. Seborrheic dermatitis is provoked by malassezia furfur
b. Irritant contact dermatitis is common in housewives
c. Pompholyx is an itchy vesicular eczema of the hands and feet
d. Patch testing is an important diagnostic procedure in discoid eczema
e. Langerhans cells play an important role in the pathogenesis of allergic
contact dermatitis.

Q51. In atopic dermatitis, the followings are true except


a. It is more prone to herpes simplex virus (HSV) infection
b. The diagnosis cannot be made in the absence of asthma
c. In infants, the face is often affected
d. It may have significant blood hypereosinophilia
e. The skin of patients with atopic dermatitis is more prone to irritation.

Q52. In atopic dermatitis, the followings are true except


a. Asthma, hay fever and urticaria are uncommon (rare) associated with
atopic dermatitis either in the patient or the family
b. White dermogrophism
c. Eosinophilia
d. Skin prick test; these are frequently +ve to food allergen
e. Patch test may also be positive.

Q53. In atopic dermatitis, the followings are true except


a. The level of IgE in the blood are raised in some but not in all patient and
these levels may reflect the activity of the disease
b. It can be associated with a reduced delayed hypersensitivity reaction
c. ↑ Susceptibility to viral infection such as; wart, molluscum contagiosum
and herpes simplex
d. ↑ Susceptibility to bacterial infection such as; staphylococcus and
streptococcus infection
e. Eczema is much worse for most patient during the summer months.

Q54. Aggravating factors of atopic dermatitis include the followings except


a. Infection
b. Humidity
c. Rough and woolen clothes
12
d. Immunization
e. Hot climate.

Q55. Complications of atopic dermatitis include the followings except


a. 2ry bacterial infection
b. Increase risk of viral infection
c. Herpes simplex lesion may cause widespread lesion or sever illness
(eczema herpeticum)
d. Erythroderma
e. Renal failure.

Q56. The topical treatment of atopic dermatitis include the followings except
a. Emollient
b. Wet compress, such as burrow solution or potassium permanganate
c. Mild topical steroid cream
d. Topical antihistamine
e. Steroid and antibiotic locally for secondary bacterial infection.

Q57. In infantile seborrheic dermatitis, the followings are true except


a. Common
b. It runs a chronic course
c. Self-limiting
d. Inflammatory dermatosis, occurring after six months of life, affecting the
face and extrimities
e. If it is sever, it may be sign of HIV infection in sexually active adults.

Q58. In adulthood type seborrheic dermatitis, the followings are true except
a. Common in males
b. Gradual onset
c. It runs a chronic course
d. Characterized by remission and relapse
e. Improve in winter and flare in summer.

Q59. In seborrheic dermatitis, the common sites of involvement include the followings
except
a. Scalp, ears, nasolabial folds and eyebrows
b. Pre-sternal and inter-scapular region
c. Axillae and groin
d. Extensor surface of elbows and knee
e. Umbilicus.

Q60. In contact dermatitis, the followings are typical sensitizing agents except
a. Chromium
13
b. Lanolin
c. Aluminum
d. Rubber
e. Resin.

Q61. In primary irritant contact dermatitis, the followings are true except
a. Dermatitis which results from overexposure of the skin to an irritant
substance
b. An irritant dermatitis which can occur in any individuals
c. It results of an immunological reaction
d. Dribbling, lip licking may result in primary irritant dermatitis in children
e. Napkin dermatitis, housewife’s hand dermatitis are examples of primary
irritant contact dermatitis.

Q62. In primary irritant contact dermatitis, the followings are true except
a. Anyone can be affected
b. It may be bullous
c. Diagnosis can be confirmed by the patch test technique
d. It may resolve if the irritant is removed from the skin
e. Respond to topical steroids.

Q63. In napkin dermatitis, the followings are true except


a. An eczematous response to prolonged exposure to body fluids
b. Faecal lipase and protease, hydrolyse urinary urea, releasing ammonia,
which in turn acts as an irritant
c. It is more common in breast fed babies
d. It involves the lower abdomen, pubic area, buttocks and genitalia
e. Treated by keeping the child as dry as possible.

Q64. In housewife's dermatitis, the followings are true except


a. The skin is itchy and ill-defined, erythematous, fissured patches
b. It is due to frequent exposure to water and irritants
c. It occurs usually on the backs of the fingers and hands
d. The condition is much less common in those who have a family or a
personal history of atopy, especially atopic eczema
e. Patch testing is necessary to rule out a contact allergen.
Q65. In allergic contact dermatitis, the followings are true except
a. It is due to delayed hypersensitivity reaction which results in eczema
b. The most common industrial agents are chromates in cement, resin in the
plastic industry, dyes, rubber and glues
c. Patch testing is not necessary to rule out a contact allergen
d. Metal dermatitis is the commonest cause of contact dermatitis and
predominantly affects females
e. Pigments used for tattooing may produce a contact dermatitis after several
years.

14
Q66. In pompholyx (dyshidrotic eczema), the followings are true except
a. Recurrent attacks of vesicles which are intensely itchy appear at the palms
and sides of the fingers and soles
b. The eruption is remarkably symmetrical
c. It is usually precipitated by cold weather
d. Secondary infection is a recognized complication
e. Local steroid and antibiotics are helpful.

Q67. In phytophotodermatitis, the followings are true except


a. It is a reaction between a plant chemical and ultraviolet light
b. The plant chemical is a furocoumarin containing 5-methoxypsoralen
c. It is non immunological dermatitis and anyone can develop the reaction
d. It is usually vesiculobullous and followed by post inflammatory
hypopigmentation
e. Treated by a potent topical steroid and systemic steroid may be required.

Q68. An acute weeping eczema is commonly treated with one of the following
a. Corticosteroid ointment
b. Coal tar
c. Dithranol
d. Local antihistamine
e. Potassium permanganate compresses.

Q69. Vesicular rash on the hands are due to the followings except
a. Palmo-planter psoriasis
b. Scabies
c. Chickenpox
d. Pemphigus
e. Contact dermatitis.

Q70. The following conditions may present as vesicles except


a. Erythema nodosum
b. Tinea pedis
c. Irritant contact dermatitis
d. Pemphigus
e. Dermatitis herpetiformis.
Q71. Vesicles and bullae may occur in the followings except
a. Acute dermatitis
b. Pityriasis rubra pilaris
c. Epidermolysis bullosa simplex
d. Steven Johnson syndrome
e. Bullous pemphigoid.

Q72. Causes of intraepidermal bullae include the following except


a. Eczema
b. Herpes simplex infection
15
c. Epidermolysis bullosa simplex
d. Toxic epidermal necrolysis
e. Pemphigus vulgaris.

Q73. In pityriasis alba, the followings are false except


a. A type of vitiligo
b. Common in elderly people
c. Occurs commonly on the trunk
d. Worse in winter
e. Heals spontaneously.

Q74. Napkin rash in an infant may be caused by the followings except?


a. Zinc deficiency
b. Psoriasis
c. Candidiasis
d. Contact dermatitis
e. Pediculosis corporis.

Q75. The differential diagnosis of patchy hair loss include the followings except?
a. Alopecia areata
b. Tinea capitis
c. Traction alopecia
d. Trichotillomania
e. Seborrheic dermatitis.

Q76. In alopecia areata, the followings are true except


a. Round or oval patch or patches of complete hair loss with no visible change in the
skin
b. It may heal spontaneously
c. It is due to infection with staphylococcus aureus
d. It is usually on the scalp, beard area, eyebrows and eyelashes
e. In some patients, there is progression of the disease into alopecia totalis or
universalis.

Q77. Scarring alopecia can be caused by the followings except


a. Inflammatory tinea capitis
b. Scalp psoriasis
c. Lichen planus
d. Discoid lupus erythematosus
e. Cicatritial pemphigoid.

Q78. The following are causes of scarring alopecia except


a. Scaly type of tinea capitis
b. Lamellar ichthyosis
16
c. Scalp lichen planus
d. Severe trauma to the scalp
e. Deep burn.

Q79. One of the following can cause scarring alopecia


a. Systemic lupus erythematosus
b. Secondary syphilis
c. Trichotillomania
d. Fevic type of tinea capitis (favus)
e. Alopecia areata.

Q80. Non-scarring hair loss is seen in the following except


a. After a crash diet
b. Alopecia areata
c. Lupus erythematosus
d. Lichen planus
e. Secondary syphilis.

Q81. The followings are true of hair fall except


a. Discoid lupus erythematosus causing scarring alopecia
b. Anaemia is a cause of hair fall
c. Androgenetic alopecia heals on its own
d. Fungal infections cause hair to break
e. Minoxidil is used in male pattern baldness.

Q82. Hair fall are reversible in the followings except


a. After surgery
b. When lichen planus is seen
c. During the postpartum period
d. Alopecia areata
e. Secondary syphilis.

Q83. Hirsutism can be caused by the followings except


a. Idiopathic
b. Ovarian tumour
c. Adrenal tumour
d. Pituitary tumour
e. Retinoid therapy.

Q84. The following drugs are causing hypertrichosis except


a. Oral retinoids
b. Cyclosporine A
17
c. Minoxidil
d. Corticosteroid
e. Phenytoin.

Q85. Poor prognostic factors of alopecia areata include the followings except
a. Recurrent
b. Presence of thyroid disease
c. Females have a poorer prognosis
d. Presence of ophiasis
e. Presence of atopy.

Q86. The followings are using in the treatment of alopecia except


a. Topical steroid
b. Intralesional steroid
c. Systemic steroid
d. Dithranol
e. Systemic tetracycline.

Q87. Differential diagnoses of hypopigmented patches include the following except


a. Early vitiligo
b. Pityriasis alba
c. Pityriasis versicolor
d. Lepromatous leprosy
e. Ash leaf macule of tuberous sclerosis.

Q88. Hypopigmentation can occur in the following except


a. Tuberculoid leprosy
b. Tuberous sclerosis
c. Pityriasis rosea
d. Morphea
e. Pityriasis alba.

Q89. In melasma, the followings are true except


a. Can be caused by oral contraceptive
b. Can be caused by pregnancy
c. It can be caused by exposure to sunlight
d. It occurs only in women
e. MSH level is quiet normal in a patient with melasma.

Q90. In melasma, the followings are true except


a. It occurs only in females
b. Fair-skinned individuals have the best prognosis
18
c. Many patients recover completely after pregnancy
d. In darker skinned individuals, the condition can be persistent
e. Complete sun block and skin lightening cream can be used in the
treatment.

Q91. The following lesions may be pigmented except


a. Seborrhoeic keratosis
b. Basal cell carcinoma
c. Melanocytic naevus
d. Dermatofibroma
e. Pyogenic granuloma.

Q92. Generalized hyperpigmentation of the skin can be caused by the followings


except
a. Addison's disease
b. Cushing syndrome
c. Hypertension
d. Carcinoma of the lung
e. Chronic renal failure.

Q93. The following drugs are associated with hyperpigmentation except


a. Arsenic
b. Amiodarone
c. Aspirin
d. Cyclophosphamide
e. Phenytoin.

Q94. In vitiligo, the followings are true except


a. It affect approximately 1-2% of the population
b. More common in male than female
c. It has peak incidence in the 2nd and 3rd decades of life
d. Characterized by one or multiple patches of depigmented skin
e. In fair-skinned patient, examination under wood’s lamp may be
necessary to see the extent of vitiligo.

Q95. In vitiligo, the followings are true except


a. Possibly an autoimmune disease
b. It may occur in dermatomal distribution
c. The melanocyte are absent in the affected patches
d. Depigmenting agents are used to treat an early vitiligo
e. PUVA therapy can be used in the treatment of vitiligo.

Q96. Vitiligo may be associated with the following diseases except?


a. Diabetes mellitus
b. Thyroid disease
19
c. Pernicious anaemia
d. Alopecia
e. Neurofibromatosis.

Q97. In pigmentary problems, the followings are true except


a. Post-inflammatory hyperpigmentation is more intense in blacks when
compared to whites
b. Idiopathic guttate hypomelanosis commonly affect the lower extrimities.
c. Vitiligo may respond to topical steroids
d. Vitiligo may involve one segment of the body only
e. Melasma is exclusively a female problem.

Q98. In melanocytic abnormalities, the followings are true except


a. Albinism is characterized by functional defect of melanocyte
b. Vitiligo is characterized by absence of melanocytes
c. In Mongolian spot, there is localized increase in epidermal pigmentation
d. In café au lait macules, there is an increase in number of melanocytes
e. In freckles, there is an increase in melanogenesis.

Q99. In acne vulgaris, the followings are true except


a. It may occur in as young as a 10 years old
b. Pustules, when cultured, often grow staphylococcus aureus
c. It may responds to tetracycline treatment
d. Acne cysts may be treated with intralesional steroids
e. Post acne scars can be improved using laser treatment.

Q100. Acne vulgaris is usually treated with the followings except


a. Doxycycline
b. Penicillin
c. Clindamycin
d. Erythromycin
e. Minocycline.

Q101. Acne can be caused by the following drugs except?


a. Androgenic and estrogenic steroid therapy
b. Systemic corticosteroid therapy
c. Lithium carbonate
d. Anticonvulsants
e. Dapsone.

Q102. In nodulocystic acne, the followings are true except?


a. Severe form of acne
b. It occurs in both sexes but more common in females
20
c. Characterized by painful papules and nodules and cysts
d. The face, chest and back may be involved
e. Treated by retinoids.

Q103. In acne fulminans, the followings are true except


a. It is usually a mild form of acne
b. Characterized by inflammatory nodules and cysts
c. Fever and leukocytosis are common
d. Polyarthralgia, polymyalgia, destructive arthritis have been reported in association
with it
e. Treated by prednisolone tablets (short course) together with isotretinoin.

Q104. The followings are true regarding retinoid treatment of acne except
a. It is the treatment of choice for mild acne
b. It is the treatment for acne that is unresponsive to conventional treatment
c. It is given in a dose of 0.5-1 mg/kg/day orally for at least 4 months
d. It does not work immediately but an effect should be seen within 6 weeks
e. The face responds better than trunk.

Q105. Topical treatment of acne include the followings except


a. Topical erythromycin
b. Topical clindamycin
c. Benzoyl peroxide
d. Benzyl benzoate
e. Retinoic acid.

Q106. In rosacea, the followings are true except


a. It is common in females and more sever in males
b. Telangiectasia and absence of comedones differentiates it from acne
c. Rhinophyma, blephritis and conjunctivitis are recognized complication
d. Periorbital and perioral are mostly affected
e. Can be treated with systemic tetracycline.

Q107. The following are features of rosacea except


a. Comedones
b. Erythematous papule and pustule
c. Telangiectasia
d. Rhinophyma
e. Tetracycline for a short term is effective.

Q108. Rosacea is characterized by the following except


a. Papules and pustules that occurs on the face, associated with erythema
and flushing
21
b. Appear at any time from the fourth decade onward
c. Fluctuation in temperature aggravate the disorder
d. Hot drinks, spicy food can exacerbate the disorder
e. Corticosteroid therapy may precipitate the condition.

Q109, The following are features of rosacea except


a. Papules and pustules
b. Facial erythema
c. Telangiectasia
d. Comedones
e. Response to tetracycline.

Q110. In the treatment of rosacea, the followings are true except


a. It is better to avoid hot tea or coffee or reducing them
b. Spicy food should be restricted
c. Alcohol should be stopped
d. Rosacea responds to therapy within few days (more quickly than acne)
e. In severe cases, retinoids is not effective.

Q111. The following diseases are sexually transmitted except


a. Molluscum contagiosum
b. Scabies
c. Granuloma inguinal
d. Dermatitis herpetiformis
e. Pediculosis pubis.

Q112. In sexually transmitted diseases, the followings are true except


a. Sign of groove is seen in bubo of chancroid
b. Genital lymphoedema is a complication of lymphogranuloma venereum
c. Malignant transformation can occur in genital ulcer of granuloma inguinale
d. Tysonitis and littritis are chronic complication of gonorrhea in males
e. Mucus patches and snail track ulcers are features of secondary syphilis.

Q113. Urethral discharge in a male may be caused by the following except


a. Syphilis
b. Trichomoniasis
c. Gonorrhoea
d. Nonspecific urethritis
e. Reiter disease.

Q114. In gonorrhea, the followings are true except


a. Urethral discharge is the usual presentation
b. Female is usually asymptomatic carrier
c. Vertical transmission may lead to ophthalmia neonatorum
22
d. Bartholinitis is a manifestation of complicated gonorrhea
e. Penicillin is the drug of choice.

Q115. The followings are causing genital ulcer except


a. Syphilis
b. Granuloma inguinale
c. Chancroid
d. Gonorrhoea
e. Herpes simplex infection.

Q116. The following organisms can cause genital ulcer except


a. Haemophilus ducreyi
b. Treponema pallidum
c. Herpes virus
d. Chlamydia trachomatis
e. Human papilloma virus.

Q117. Genital ulcerations may be seen in the following except


a. Pemphigus vulgaris
b. Syphilis
c. Psoriasis
d. Fixed drug eruption
e. Behcet’s syndrome.

Q118. In genital ulcers, the followings are true except


a. Syphilitic ulcer is characteristically hard and painless
b. Gonorrhea is characterized by purulent ulcers
c. Malignancy is an important differential diagnosis
d. Chancroid results in multiple painful ulcers
e. Snail tract ulcers are seen in the secondary syphilis.

Q119. In syphilitic chancre, the followings are true except


a. Painless ulcer with indurated edge
b. Caused by treponema pallidum
c. Usually heal spontaneously within 3 months
d. Associated with lymph node enlargement which is quite painful
e. May occur anywhere on the body.

Q120. In syphilitic chancre, the followings are true except


a. It is caused by treponema pallidum
b. It is usually single but can be multiple
c. It is soft and easily bleed
23
d. Coronal sulcus is the commonest site of involvement
e. Usually heal spontaneously within 3 months.

Q121. The followings are considered in the differential diagnosis of secondary syphilis
except
a. Guttate psoriasis
b. Drug eruption
c. Pityriasis rosea
d. Lupus vulgaris
e. Lichen planus.

Q122. Secondary syphilis is characterized by the following except


a. High VDRL titer
b. Vesicular skin rash
c. Generalized lymphadenopathy
d. Mucous patches
e. Condylomata latum.

Q123. In the serological tests of syphilis, the followings are true except
a. A positive result of VDRL and TPHA can confirms the diagnosis of secondary
stage of syphilis
b. A negative result of VDRL does not exclude the diagnosis of primary
of syphilis
c. TPHA is the first serological marker detectable after infection
d. A positive result of TPHA can persist through life even if the patient had
been adequately treated for syphilis
e. A positive result of TPHA invariably means that the patient is highly
infectious.

Q124. In the treatment of syphilis, the following drugs are effective except
a. Na stibogluconate
b. Tetracycline
c. Ceftriaxone
d. Azithromycin
e. Penicillin.
Q125. In chancroid, the followings are true except
a. Usually occur 1 to 5 days after sexual intercourse
b. Caused by treponema pallidum
c. May occur simultaneously with syphilis
d. Females may be asymptomatic carriers
e. Immunity is not conferred with infection.

Q126. In chancroid, the followings are true except


a. It is associated with suppurative regional lymphadenopathy
b. The genital ulcers are classically indurated and painless
c. The diagnosis is usually based on the culture
24
d. It is a major risk factor for HIV transmission
e. The treatment of choice is erythromycin or ceftriaxone.

Q127. In granuloma inguinale, the followings are true except


a. Characterized by progressive ulceration of the genitalia
b. The ulcers are usually painless
c. The regional lymph node are not enlarged
d. Incubation period is 5-7 days
e. Malignant transformation in chronic untreated ulcers has been reported.

Q128. The recommended drugs for the treatment of the following STDs are true
except
a. Penicillin for gonorrhoea
b. Azithromycin for non-gonococcal urethritis
c. Acyclovir for genital herpes
d. Podophyllin for genital warts
e. Metronidazole for trichomoniasis.

Q129. In genital herpes, the followings are true except


a. Common
b. May cause pre-term labour in pregnancy
c. It is due to herpes simplex type 2, but can be caused by herpes simplex
type 1
d. Is transmissible through contact with herpetic skin lesions
e. It is completely cured after treatment with acyclovir.

Q130. In genital warts, the followings are true except


a. All genital warts have a tendency for malignant transformation
b. Genital warts are best treated with podophyllin paint or cryotherapy
c. The risk of recurrence of genital warts is greatest within 3 months
of clearance
after therapy
d. There is strong links between genital wart and cervical carcinoma
e. There is a 50 % chance of transmission of the disease by a single sexual
contact.

Q131. The skin disease more often seen in AIDS patient is


25
a. Seborrheic dermatitis
b. Erythema nodosum
c. Psoriasis
d. Herpes zoster
e. Atopic dermatitis.

Q132. Presenting features of HIV infection include the following except


a. Oral hairy leukoplakia
b. Atypical pneumonia
c. Thrombocytopenic purpura
d. Pulmonary tuberculosis
e. Oral candidiasis.

Q133. Herpes simplex may present as the followings except


a. Keratoconjunctivitis
b. Vulvovaginitis
c. Eczema herpeticum
d. Herpes gestations
e. Gingivostomatitis.

Q134. Herpes simplex infection is characterized by the followings except


a. An acute, self-limited, grouped vesicles on an erythematous base
b. It is caused by varicella zoster virus
c. Recurrent attacks are caused by virus reactivation
d. The primary infection of herpes simplex may produce sever systemic and local
reaction
(herpetic gingivostomatitis)
e. All the primary infection results in latent disease which may recur in mild form.

Q135. Complication of herpes simplex infection include the followings except


a. Dissemination of herpetic infection
b. Erythema multiforme
c. Erythema nodosum
d. Eczema herpeticum
e. Herpetic kerato-conjunctivitis.
Q136. In herpes zoster, the followings are true except
a. Dermatomal distribution
b. Characterized by group of vesicle on an erythematous base
c. Situated bilaterally within the distribution of cranial or spinal nerve
d. Herpes zoster may be the earliest clinical sign of HIV infection
e. Acyclovir either orally or intravenous infusion is effective.

Q137. In herpes zoster, the followings are true except


a. Dermatomal distribution
b. It usually associated with pain
c. A past history of chicken pox during childhood is relevant
26
d. It presents with group of papules on erythematous base
e. May be followed by post herpetic neuralgia.

Q138. Complication of herpes zoster include the followings except


Q153. In genital warts, the followings are true except
a. Patient should be examined for other sexually transmitted infection
b. Anoscopy or proctoscopy is indicated for patient with anal warts
c. Cervical smear taking are important and should be done in a female patient with
genital warts
d. Human papilloma virus infection may persist throughout a patient life-time in a
dormant state and becomes infectious intermittently
e. Systemic podophyllin is the drug of choice.

Q154. Anti-viral drugs include the followings except


a. Famciclovir
b. Valacylovir
c. Panciclovir
d. Foscarnet
e. Desloratadine.

Q155. In Orf, the followings are true except


a. A viral infection of the skin contracted from infected sheep or goats
b. It is caused by a pox virus
c. It is an occupational hazard for farmers and veterinary doctors
d. It is occur commonly on the index finger
e. Widespread infection in adults is characteristic of HIV infection.

Q156. In hand, foot and mouth disease, the followings are true except
a. It is caused by coxsakie virus A16
b. It is an occupational hazard for farmers and veterinary doctors
c. Yellow vesicles surrounded by erythema on the both hands and feet
d. In the mouth, the vesicle break easily and erosion results
e. The condition lasts 7 days and does not recur.

Q157. Streptococcal throat infection may be implicated in the development of the


following except
a. Guttate psoriasis
b. Cutaneous vasculitis
c. Scarlet fever
d. Lichen planus
e. Erythema nodosum.

Q158. In impetigo, the followings are true except


a. A contagious disease
27
b. Bullae can be the presenting feature
c. An autoimmune disease
d. Glomerulonephritis is a recognized complication
e. Culture can confirm the diagnosis.

Q159. In impetigo, the followings are true except


a. A contagious disease
b. It can be caused by staph aureus
c. It can be caused by streptococcus pyogenes
d. Heals with scarring
e. Glomerulonephritis is a known complication.

Q160. In cellulitis, the followings are true except


a. Commonly affects legs
b. Characterized by erythema with well-defined edges
c. There is hotness and tenderness at the site of erythema
d. Oedema is a common feature
e. It can be caused by staphylococcus or streptococcus bacteria.

Q161. In erysipelas, the followings are true except


a. Commonly affects the face
b. It involves the dermis and upper subcutaneous tissue
c. The margins of the erysipelas are much more demarcated than those of
cellulitis
d. Almost always due to group A B-haemolytic streptococcus
e. Flucloxacillin is the drug of choice.

Q162. In carbuncle, the followings are true except


a. It is a single hair follicle infection
b. Occurs commonly in diabetic patients
c. Affects commonly the back of the neck
d. It is usually associated with constitutional symptoms
e. Treated by systemic antibiotics and surgical drainage.

Q163. In sycosis barbae, the followings are false except


a. Allergic cause
b. Bacterial cause
c. Viral cause
d. Fungal cause
e. Immunological.

Q164. In erythrasma, the followings are true except


a. An intertrigo caused by corynebacterium minutissimum
28
b. Common in the obese and in warm, humid environments
c. It fluorescence pale green under wood's lamp
d. The condition may be asymptomatic or itchy
e. The organism can be seen in scrapings of skin treated with potassium
chloride.

Q165. Scarlet fever caused by one of the following


a. Group A B-haemolytic streptococcus
a. Streptococcus pyogenes
b. Staphylococcus aureus
c. Haemophilus influenza
d. Corynebacterium minutissimum.

Q166. Types of cutaneous tuberculosis include the following except


. a. Warty tuberculosis
b. Scrofuloderma
c. Lupus pernio
d. Erythema induratum
e. Lichen scrofulosorum.

Q167. In lupus vulgaris, the followings are true except


a. It is due to spread of the organism via sinuses from underlying caseous
lymph nodes
b. It is the commonest form of cutaneous tuberculosis
c. Apple jelly nodules are a characteristic feature
d. Untreated lesions heal with scarring
e. Malignant changes have been reported in healed scar.

Q168. In leprosy, the followings are true except


a. Caused by mycobacterium leprae
b. Leprosy is a disorder of tropical countries, particularly India and Africa
c. It is uncommon in western countries
d. It is transmitted by inhalation or ingestion of infected nasal droplets
e. Lepromin test is positive in lepromatous leprosy and negative in
tuberculoid type.

Q169. In leprosy, the followings are true except


a. The clinical lesion of tuberculoid leprosy is hypopigmented patch
b. The greater auricular nerve is thickened in lepromatous leprosy
c. The lepromin test is positive in tuberculoid leprosy
d. Type I reaction is a downgrading reaction
e. Erythema nodosum leprosum treated with thalidomide.

Q170. In tuberculoid leprosy, the followings are true except


29
a. The immunological resistance to the leprosy bacillus is very low
b. The skin lesions are solitary or few and asymmetrically distributed
c. The patches are well-defined and often annular in shape
d. The sites of predilection are the cool and peripheral parts of the body
e. The peripheral nerves become thickened and palpable.

Q171. In lepromatous leprosy, the followings are true except


a. The immunological response of the host is high
b. Widespread and progressive with bacilli invading the skin, nerves and
reticuloendothelial system
c. The skin lesions are symmetrical and extensive
d. The disease is highly infectious from nasal discharge
e. Neurological involvement occurs latterly.

Q172. In lepromatous leprosy, the followings are true except


a. The first symptoms are usually nasal such as discharge and epistaxis
b. The early cutaneous lesions are symmetrical, ill-defined, slightly
hypopigmented macules
c. May be presented as papules, plaques and nodules
d. The face, arms, legs and buttocks are rarely affected
e. Dapsone, rifampicin and clofazimine are used in the treatment.

Q173. In lepromatous leprosy, the followings are true except


a. Diffuse infiltration of the skin of the forehead causes a leonine appearance
b. The eyebrows and eyelashes disappear
c. Swelling of the ankles and lower legs is very rare
d. Glove and stocking anaesthesia develops from polyneuritis
e. Destruction and shortening of the digit result from repeated unappreciated
trauma.

Q174. In tinea capitis, the followings are true except


a. Occurring commonly in school children
b. Boys have tinea more frequently than girls
c. More common in children of lower socio-economic group
d. It can be transmitted by sharing combs, hats and pillows
e. Treated by topical antifungal.

Q175. In tinea corporis, the followings are true except


a. The lesion is annular and well-defined
b. May occur on the trunk and limbs
c. If the source is animal, the margin may be quite inflamed and even pustular
d. Scrapings should be taken from the centre of the lesion for identification
of hyphae.
e. Treated by antifungal drugs.
30
Q176. Differential diagnosis of tinea corporis include the followings except
a. Discoid eczema
b. The herald patch of pityriasis rosea
c. Annular psoriasis
d. Annular lichen planus
e. Atopic dermatitis.

Q177. Differential diagnosis of tinea cruris include the followings except


a. Candidal intertrigo
b. Erythrasma
c. Seborrheic dermatitis
d. Flexural psoriasis
e. Lichen planus.

Q178. In candida albicans, the followings are true except


a. Is a normal commensal of the mouth and GIT
b. Exist both in a yeast and hyphal form
c. Satellite papules and pustules are characteristic
d. May cause chronic paronychia
e. Treated by griseofulvin.

Q179. Systemic disorders predispose to candidiasis include the followings except


a. Diabetes mellitus
b. Erysipelas
c. AIDS
d. Neutropenic patient
e. Broad spectrum antibiotics.

Q180. Diseases caused by candida include the followings except


a. Oral thrush
b. Intertrigo
c. Impetigo
d. Chronic paronychia
e. Balanitis and vulvovaginitis.

Q181. Woods lamp examination reveals the following except


a. Green fluorescence in tinea capitis
b. Golden yellow fluorescence in pityriasis versicolor
c. Coral red fluorescence in erythrasma
d. Yellow fluorescence in urine from porphyria cutanea tarda patients
31
e. Milky white in vitiligo.

Q182. The followings are false regarding pityriasis versicolor except


a. High incidence in hot, humid climate
b. Good response to oral griseofulvin therapy
c. It is easily spread from person to person
d. Sever pruritus in the majority of cases
e. It is spread by close physical contact.

Q183. The following drugs are effective in the treatment of pityriasis versicolor except
a. Selenium sulphide
b. Ketoconazole
c. Griseofulvin
d. Clotrimazole
e. Itraconazole.

Q184. The following drugs are antifungal except


a. Itraconazole
b. Griseofulvin
c. Terbinafine
d. loratadine
e. Ketoconazole.

Q185. The side effects of griseofulvin include the followings except


a. Headache
b. Nausea
c. Photo-sensitive rash
d. Fixed drug eruption
e. Skin pigmentation.

Q186. In scabies, the followings are true except


a. The causative organism is a male mite
b. Can be diagnosed by scrapping and KOH examination
c. Commonly affect the palms and soles in infants
d. Burrow is pathognomonic of scabies
e. Spread by close physical contact.

Q187. In scabies, the followings are true except


a. Caused by the male mite of sarcoptes scabiei
b. Transmitted by skin to skin contact
c. Causing itching
d. Can be treated by malathion lotion
e. Management includes treatment of close contacts.
32
Q188. Sites of the scabetic eruption in adults include the followings except
a. Scalp
b. Finger webs
c. Wrist
d. Ante-cubital fossa
e. Anterior and post folds of axillae.

Q189. Sites of the scabetic eruption in adults include the followings except
a. Female areola and glans penis in male
b. Around the umbilicus
c. Lower abdomen
d. Face
e. The extensor surface of elbow and knee.

Q190. Scabicidal drugs include the followings except


a. Ivermectin
b. Permethrin
c. Sulphur
d. Dithranol
e. Benzyl benzoate.

Q191. The following disease are itchy or pruritic except?


a. Scabies
b. Lichen planus
c. Acute urticaria
d. Dermatitis herpetiformis
e. Psoriasis.

Q192. Itchy skin diseases include the followings except


a. Papular urticaria
b. Atopic dermatitis
c. Pemphigus
d. Bullous pemphigoid
e. Drug eruption.

Q193. In pediculosis corporis, the followings are true except


a. Rare
b. It occurs under the disaster condition
c. The body louse is found on the skin
d. It lives and breeds in clothing and bed linen
e. The body louse may transmit rickettsia.

Q194. In pediculosis pubis, the followings are true except


33
a. It is the most contagious sexually transmitted infection known
b. Caused by body lice
c. Transmitted sexually and may be contracted by wearing infested clothing
d. It cause severe itching in the genital region
e. It may spread to the limbs, chest, axilla and even eyebrows and eyelashes.

Q195. In cutaneous leishmaniasis, the followings are true except


a. It is an endemic in Libya
b. Spread by male sand fly
c. Commonly affect the exposed parts of the body
d. Slit and smear examination done for diagnosis
e. Treated by pentavalent antimony injection.

Q196. In cutaneous leishmaniasis, the followings are true except


a. It is endemic in Asia, Countries around the Mediterranean sea
b. Old peoples are commonly affected
c. The exposed areas are the commonest sites
d. Caused by leishmania parasite
e. Treated by pentavalent antimony injection.

Q197. Side effects of sodium stibogluconate include the followings except


a. Nausea and epigastric discomfort
b. Renal failure
c. Respiratory irritation
d. Cardiac arrhythmia
e. Hyperglycaemia.

Q198. Skin diseases associated with blistering eruption include the following except
a. Pemphigus vulgaris
b. Bullous pemphigoid
c. Erythema multiforme
d. Rosacea
e. Dermatitis herpetiformis.

Q199. The followings are recognized causes of blistering at birth except


a. Herpes simplex infection
b. Epidermolysis bullosa
c. Congenital syphilis
d. Albinism
e. Chickenpox.

Q200. Oral lesions are seen in the followings except


a. Pemphigus vulgaris
34
b. Erythema multiforme
c. Scabies
d. Lichen planus
e. Secondary stage of syphilis.

Q201. Oral lesions are common in which one of the following disease
a. Pemphigoid
b. Lichen planus
c. Psoriasis
d. Lupus vulgaris
e. Pityriasis rosea.

Q202. Mucus membrane lesions are seen in the following disease except
a. Lichen planus
b. Pemphigus
c. Dermatitis herpetiformis
d. Erythema multiforme
e. Behcet's disease.

Q203. Vesicles are seen in all of the following except


a. Aphthous ulcers
b. Herpes simplex infection
c. Hand, foot and mouth disease
d. Hand eczema
e. Chicken pox.

Q204. Bullous eruption are seen in the following except


a. Erythema multiforme
b. Barbiturate poisoning
c. Chronic chloroquine therapy
d. Pemphigus
e. Herpes infection.

Q205. In pemphigus, the followings are true except


a. It is a disease of middle age
b. Common in Jews, strongly associated with HLA-DR4
c. Characterized by the finding of IgG auto-antibodies directed against the
antigen on the surface of keratinocytes (desmoglein)
d. The disease activity often does not correlates well with the antibody titre
e. The primary cause of death is the infection with staph and the most
common site of the infection are the blood, lung and the skin.

35
Q206. In pemphigus vulgaris, the followings are true except
a. It is an auto immune skin disease
b. Oral lesions are uncommon
c. May involve only the buccal mucosa
d. May be caused by certain drugs
e. Controlled by systemic steroid with or without immunosuppression.

Q207. Drug induced pemphigus include the following except


a. Penicillin
b. Penicillamine
c. Dapsone
d. Cephalosporin
e. Rifampicin.

Q208. Bullous pemphigoid can present in these clinical pictures except


a. Urticarial plaques
b. Flaccid bullae
c. Tense Bullae
d. Hemorrhagic bullae on flexors of limbs
e. The mucus membranes are involved in a minority of patients.

Q209. In bullous pemphigoid, the followings are true except


a. Occur commonly in old ages
b. Commonly occur on the lower abdomen and thighs
c. Intradermal blistering disease
d. Tzank smear positive for acantholytic cells
e. Diagnosis confirmed by direct immunofluorescence.

Q210. In bullous pemphigoid, the followings are true except


a. Itchy, large, tense blister
b. Blisters often begin on the inner thighs, then spreading down the legs, arms
and trunk
c. The blisters heals with scarring
d. IgG and C3 found at the basement membrane zone in a linear manner
e. May be associated with malignancy.

Q211. In dermatitis herpetiformis, the followings are true except


a. It is uncommon bullous disease
b. Tends to occur only in the elderly
c. It affects the scalp, interscapular area, buttocks, knee, and elbow
d. Dapsone is very effective in its treatment
e. Present as itchy grouped vesicles on extensor surfaces of limbs and body.

Q212. In dermatitis herpetiformis, the followings are true except


36
a. The eruption is symmetrical and affects the scalp, scapulae and buttocks
b. Characterized clinically by a very pruritic grouped vesicles
c. Associated with deposits of IgG in the skin
d. There is a small risk of GIT malignancy and in particular lymphoma
e. Treated by dapsone or a gluten-free diet.

Q213. Direct immunoflurescence is helpful in the diagnosis of the following conditions


except
a. Pemphigus vulgaris
b. Bullous pemphigoid
c. Psoriasis vulgaris
d. Cutaneous vasculitis
e. Dermatitis herpetiformis.

Q214. In a patient with skin rash, the finding of mucus membrane involvement will
lend support to the diagnosis of the followings except
a. Lichen planus
b. Pemphigus vulgaris
c. Measles
d. Erythema multiforme
e. Eczema.

Q215. Nikolsky sign is positive in one of the following


a. Bullous pemphigoid
b. Erythema multiforme
c. Pemphigus vulgaris
d. Acquired epidermolysis bullosa
e. Dermatitis herpetiformis.

Q216. In erythema nodosum, the followings are true except


a. It represents a delayed hypersensitivity reaction to a variety of antigenic
stimuli
b. Less than 10% of cases are idiopathic
c. The disease occurs chiefly in young women
d. Can be associated with fever, malaise and muscle pain
e. Arthralgia occurs in more than 50% of patient.

Q217. Erythema nodosum may be caused by the following except


a. Streptococcal infection
b. Sarcoidosis
c. Tuberculosis
d. Pregnancy
e. Herpes simplex infection.

37
Q218. Erythema nodosum is characterized by the following except
a. Nodules commonly affect the extensor aspect of the legs
b. Ulceration of lesions occurs in a majority of cases
c. Bilateral hilar lymphadenopathy, when associated, sarcoidosis may be
the cause
d. Arthralgia is a commonly associated symptom
e. Tuberculosis is a recognized cause.

Q219. Drugs causing erythema nodosum include the following except


a. Penicillin
b. Sulphonamides
c. Oral contraceptive pills
d. Erythromycin
e. Barbiturates.

Q220. Treatments of erythema nodssum include the following except:


a. Non-steroid-anti-inflammatory-drugs
b. Potassium iodide
c. Systemic steroid
d. Colchicines
e. Methotrexate.

Q221. Side effects of potassium iodide include the following except


a. Urticaria
b. Acne-form rash
c. Erythema multiform
d. Erythema nodosum
e. Pemphigus.

Q222. In erythema multiforme, the followings are true except


a. Usually self-limited reaction of the skin and mucus membrane
b. Multiforme lesions such as macules, papules, vesicles, and bullae
c. Target lesion
d. Commonly affect the trunk
e. May be idiopathic or secondary to drugs or underlying infection or disease.

Q223. The main sites of predilection of erythema multiforme include the followings
except
a. The face
b. Neck
c. Extremities (extensor aspects of the elbow and knee)
d. The dorsal surface of the hands and feet
e. The trunk (common site).

Q224. Types of erythema multiforme include the followings except


a. Erythema multiforme minor
38
b. Erythema multiforme major
c. Dermatitis herpetiformis
d. Stevens-Johnson syndrome
e. Toxic epidermal necrolysis.

Q225. In erythema multiforme minor, the followings are true except


a. It is the commonest type of erythema multiforme seen (80% of cases)
b. Commonly caused by bacterial infection
c. Symmetrically distributed on the face, neck, and extremities
d. Target lesions
e. The disorder is self-limiting and the patient usually recovered within 3
weeks.

Q226. Erythema multiforme may be triggered by the following except


a. Pregnancy
b. Mycoplasma infection
c. Herpes simplex infection
d. Radiotherapy
e. Post streptococcal infection.

Q227. Erythema multiforme may be triggered by the following except


a. Systemic lupus erythematosus
b. Discoid lupus erythematosus
c. Atopic dermatitis
d. Measles and mumps
e. Infectious mononucleosis.

Q228. Erythema multiforme may be triggered by the following except


a. Lymphoma
b. Leukaemia
c. Pregnancy
d. Menstruation
e. Viral warts.

Q229. Toxic epidermal necrolysis is characterized by the followings except


a. Very common
b. Life threatening variant of erythema multiforme
c. It is usually occurs in adults
d. It is usually secondary to drugs
e. It is lethal in about 30% of cases.

Q230. Toxic epidermal necrolysis (TEN) is characterized by the followings except


a. TEN in children is usually secondary to staphylococcal infection
b. Sulphonamide most commonly implicated as the cause of TEN
c. The main difference between TEN and Steven-Johnson Syndrome is that
39
Patients with Steven-Johnson Syndrome have involvement of mucosal
surfaces
d. The mortality rate of TEN is approximately 25-30%
e. Methotrexate is the drug of choice.

Q231. In Stevens-Johnson syndrome, the followings are true except


a. Widespread involvement of the skin and mucus membrane
b. The patient is unwell and febrile
c. Conjunctival involvement may lead to corneal ulceration
d. Genital and urethral involvement may be followed by urinary retention
e. When it caused by sulphonamide, the condition usually begins 1 - 2 days
after ingestion of the drug.

Q232. Drugs causes Steven-Johnson syndrome include the following except


a. Oral retinoid
b. Penicillin
c. Sulphonamides
d. Salicylates
e. Rifampicin.

Q233. The following diseases are examples of autosomal dominant conditions except
a. Neurofibromatosis
b. Xeroderma pigmentosum
c. Tuberous sclerosis
d. Ichthyosis vulgaris
e. Epidermolysis bullosa (simplex).

Q234. The following diseases are examples of autosomal recessive conditions except
a. Xeroderma pigmentosa
b. Neurofibromatosis
c. Lamellar ichthyosis
d. Albinism
e. Epidermolysis bullosa (junctional).

Q235. In neurofibromatosis, the followings are characteristic finding except


a. Crowe's sign
b. Neurofibroma
c. Ash leaf hypopigmented macule
d. Plexiform neuroma
e. Lish nodules in the iris.

Q236. In tuberous sclerosis, the characteristic skin lesions are of the following except
a. Angiofibroma
b. Shagreen patch
c. Ash leaf hypopigmented patch
40
d. Periungual fibromas
e. Axillary freckling.

Q237. In epidermolysis bullosa simplex, the followings are true except


a. Inherited as an autosomal dominant condition
b. Blisters develop at the site of friction
c. Hair, teeth and nails are unaffected
d. Mucus membranes are severely affected
e. It is usually improve at puberty.

Q238. In junctional epidermolysis bullosa, the followings are true except


a. Severe
b. An autosomal dominant inheritance
c. Caused by an abnormality of hemidesmosomes
d. The mucus membrane, nails and teeth are affected
e. Generalized blistering of the skin and mucous membrane with a tendency
to scarring.

Q239. In xeroderma pigmentosum, the followings are true except


a. Photophobia
b. Skin malignancy
c. Freckles
d. Autosomal dominant inheritance
e. Premature aging of the skin.

Q240. In xeroderma pigmentosum, the followings are true except


a. Photoprotection remains essential for management
b. The skin is carefully supervised with view to the early diagnosis and
treatment of malignant tumour
c. Retinoids may prevent the appearance of new skin cancer
d. Use of the X-Ray therapy to treat cancer is contraindicated in these
patients
e. Death often occurs in adulthood mainly from metastasis of the malignant
disease.

Q241. In ichthyosis vulgaris, the followings are true except


a. An autosomal dominant mode of inheritance
b. Onset in early childhood
c. Large dark scales over the entire skin
d. The extensor surfaces of the extremities are the most prominently involved
e. The antecubital, popliteal fossa, axillary and gluteal folds are usually not
involved.

41
Q242. In x-linked ichthyosis, the followings are true except
a. Uncommon disorder which occur in male
b. Present within the first year of life
c. Fine and white scale that appears pasted-on over the entire body
d. All the skin is involved in particular the extensor surface
e. Corneal opacity, often do not appear until adult life.

Q243. In lamellar ichthyosis, the followings are true except


a. An autosomal dominant disorder
b. A collodion baby
c. The hands and feet may be severely fissured
d. The nails may be abnormal
e. Ectropion is an important diagnostic sign.

Q244. In systemic lupus erythematosus, the followings are true except


a. Scarring alopecia
b. Butterfly rash
c. Widespread maculo-papular eruption occurs on the light-exposed areas
d. A vasculitis rash which affects the extensor surfaces of the forearms,
dorsa of the hands and legs may occur
e. Raynaud’s phenomenon.

Q245. In discoid lupus erythematosus, the followings are true except


a. A chronic autoimmune disorder which is almost confined to the skin
b. More common in men of old age
c. The eruption is largely confined to the face and scalp
d. Healing follows centrally with scarring
e. Associated with some degree of photosensitivity.

Q246. The characteristic features of dermatomyositis include the followings except


a. Gottron's papules
b. Periorbital oedema and a mauve erythema
c. Butterfly rash
d. Photosensitivity
e. Proximal muscle weakness.

Q247. In dermatomyositis, the followings are true except


a. Difficulty in combing the hair
b. Difficult in rising from chair
c. Difficulty in eating and drinking
d. Difficulty in getting up or down stairs
e. Difficulty in raising the neck when in bed.

42
Q248. In dermatomyositis, the followings are true except
a. Heliotrope rash
b. Gottron’s papule
c. Fingertip and periungual erythema
d. Scarring alopecia
e. Poikiloderma.

Q249. Cutaneous manifestation of scleroderma include the followings except


a. Loss of the normal skin line on the forehead
b. The nose has a beak like appearance
c. The oral cavity becomes larger and there is easily in opening the mouth
d. Math-like telangiectasia on the face, hand and trunk
e. Raynaud’s phenomenon.

Q250. In morphea or cutaneous scleroderma, the followings are true except


a. A localized sclerosis of the skin of unknown etiology
b. Morphea is not localized to the skin and has systemic consequences
c. Cases have been reported secondary to penicillamine therapy
d. It can occur anywhere on the body
e. It may disappear spontaneously after number of years.

Q251.The followings are features of Behcet’s disease except


a. Aphthous oral ulceration
b. Scleritis
c. Meningoencephalitis
d. Erythema marginatum
e. Positive pathergy test.

Q252. The following diseases are aggrevated by sunlight except


a. Darier disease
b. Xeroderma pigmentosum
c. Nummular eczema
d. Systemic lupus erythematosus
e. Dermatomyositis.

Q253. Chronic sun exposure results in an increased risk in developing of the following
except
a. Squamous cell carcinoma
b. Malignant melanoma
c. Basal cell carcinoma
d. Melasma
e. Psoriasis.

43
Q254. Phototherapy may be useful in the treatment of the following except
a. Mycosis fungoides
b. Erysipelas
c. Lichen planus
d. Vitiligo
e. Psoriasis.

Q255. PUVA is indicated in the treatment of the following except


a. Xeroderma pigmentosum
b. Psoriasis
c. Lichen planus
d. Vitiligo
e. Alopecia.

Q256. Side effects of PUVA include the followings except


a. Skin burning
b. Skin erythema
c. Diffuse hypopigmentation
d. Skin and genital cancer
e. Cataract.

Q257. Contraindication of PUVA include the followings except


a. Known photosensitivity to the psoralen
b. Liver disease
c. SLE
d. Vitiligo
e. Children under the age of 12 years.

Q258. Side effects of anti-malaria include the followings except


a. Skin rashes
b. Hypertension
c. Toxic psychosis
d. Exacerbate psoriasis
e. Bleach the hair.

Q259. Side effects of dapsone include the followings except


a. Peripheral motor neuropathy
b. Methemoglobinemia
c. Agranulocytosis and aplastic anemia
d. Sulfone syndrome
e. Bone marrow suppression.

44
Q260. Evaluation of patient who is being considered for retinoids therapy include the
followings except
a. Pregnancy test
b. Pretreatment fasting lipid profiles
c. Liver function test
d. Renal function test
e. Serum zinc level.

Q261. The following are side effects of systemic retinoids except


a. Hyperlipidemia
b. Haemolytic anaemia
c. Diffuse hair loss
d. Foetal malformation
e. Dryness of the skin.

Q262. Side effects of isotretinoin include the followings except


a. Depression
b. Myalgia
c. Cheilitis
d. Teratogenicity
e. Hyperostosis.

Q263. The most common side effect of isotretinoin


a. Headache
b. Hepatitis
c. Cheilitis
d. Hyperlipidaemia
e. Hair loss.

Q264. Uses of systemic retinoids in dermatology include the followings except


a. Acne
b. Ichthyosis
c. Impetigo
d. Lichen planus
e. Psoriasis.

Q265. Side effects of topical steroid include the following except


a. Skin atrophy
b. Striae particularly in the body folds
c. Delayed healing of skin ulcer
d. Hyperpigmentation
e. Spread of skin infection.

45
Q266. Complication of cryotherapy include the following except
a. Pain
b. Pigmentary changes
c. Haemorrhagic blisters
d. Loss of sensation
e. Over-treatment can results in the recurrence of the wart around the site of
the blister.

Q267. The following principles should kept in mind when prescribing


topical dermatologic therapy
a. The more severe the skin reactions, the milder the local therapy as
oral therapy is needed
b. The morphological stage of disease determines the choice of the vehicle
c. The choice of vehicle is determined by the site of the skin lesion
d. Atopic dermatitis in children is more severe and therefore more
potent steroids are needed
e. Only a thin layer on direct contact with the skin is important
for penetration.

Q268. Systemic corticosteroids may be required in the treatment of the following


except
a. Pemphigus vulgaris
b. Inflammatory tinea capitis (kerion)
c. Ichthyosis vulgaris
d. Dermatomyositis
e. Erythema nodosum.

Q269. Systemic corticosteroids may be required in the treatment of the followings


except
a. Pemphigus vulgaris
b. Epidermolysis bullosa dystrophica
c. Acute urticaria
d. Dermatomyositis
e. Erythema nodosum.

Q270. In methotrexate, the followings are true except


a. Folic acid antagonist
b. It has a great affinity for dihydrofolate reductase
c. It is not used in pregnancy and breast feeding
d. It is given once daily (0.2 - 0.4 mg/kg) either orally or injection (IV, IM or S/C)
e. It is excreted by the kidney.

46
Q271. Side effects of methotrexate include the followings except
a. Bone marrow suppression
b. Lung Abcess
c. Oral ulceration
d. Pneumonitis
e. Liver fibrosis.

Q272. Contraindication of methotrexate include the followings except


a. Hyperlipidemia
b. Alcohol abuse
c. Pregnancy and lactation
d. Liver and kidney disease
e. Blood anomaly (anaemia, leucopoenia and thrombocytopenia).

Q273. Uses of cyclosporine in dermatology include the followings except


a. Severe psoriasis
b. Severe atopic dermatitis
c. Chronic dermatitis on the hands
d. Pemphigus
e. Dermatitis herpetiformis.

Q274. Side effects of cyclosporine include the followings except:


a. Nephrotoxicity
b. Hypotension
c. Gum hyperplasia
d. Hirsutism or hypertrichosis
e. Increase the incidence of lymphoma.

Q275. Side effect of cyclosporine includes the following except:


a. GIT upset
b. Tremor
c. Hypertension
d. Hyperkalaemia
e. Alopecia.

Q276. Drug interaction of cyclosporine include the followings except


a. Non-steroidal anti-inflammatory drugs
b. Dapsone
c. Ketoconazole
d. Erythromycin
e. Rifampicin.

Q277. In keratoacanthoma, the followings are true except


a. Slow growing tumor

47
b. Histopathological features are almost indistinguishable from squamous
cell carcinoma
c. A tumour with erythematous base and central keratin plug
d. Malignant in nature
e. Self-resolving tumour.

Q278. In pyogenic granuloma, the followings are true except


a. Benign vascular papule, possibly as a response to injury
b. Sudden in onset and tends to bleed
c. The lesion is neither granulomatous in pathology nor pyogenic in origin
d. Occurs mostly in middle age and elderly
e. The lesion can be removed by curettage and cautery or application of silver
nitrate.

Q279. In dermatofibroma, the followings are true except


a. Common skin tumour, sometimes deeply pigmented, on the lower legs
b. It is a raised papule or a nodule with a smooth surface
c. It is very hard and cannot be gripped between the finger and thumb
d. It may be single or multiple
e. A characteristic feature is the dimple sign.

Q280. In lipoma, the followings are true except


a. A common benign tumour of adipose tissue
b. It commence in early adult life
c. It may be single or multiple
d. It is soft, sometimes lobulated and have a normal skin surface
e. Commonly located on the face and feet.

Q281. Changes which suggest that a pigmented naevus has become malignant include
the followings except
a. Increase pigmentation of the lesion
b. Increase of hair in hairy naevus
c. Ulceration of lesion
d. Onset of itching
e. Development of vitiligo.

Q282. Malignant change can occur in the followings except


a. Scars
b. Nevus sebaceous
c. Dermatofibroma
d. Chronic radiodermatitis
e. Lichen planus.

Q283. Cutaneous manifestation of systemic malignancy include the followings except


a. Dermatomyositis
b. Exfoliative dermatitis
48
c. Generalized hyperpigmentation
d. Chancroid
e. Generalized pruritus.

Q284. The following conditions are associated with an increased risk of


cutaneous malignancies except
a. Chronic arsenic poisoning
b. Xeroderma pigmentosum
c. Oral lichen planus
d. Chronic venous ulceration
e. Chronic urticarial.

Q285. An underlying malignancy may be associated with the followings except


a. Dermatomyositis
b. Discoid lupus erythematosus
c. Cutaneous vasculitis
d. Erythema nodosum
e. Chronic urticarial.

Q286. The typical features of squamous cell carcinoma include the following except
a. Malignant tumour arising from Langerhan's cell
b. Ultraviolet irradiation is the most common cause
c. Previous arsenic ingestion may induce squamous cell carcinoma
d. SCC may be a late sequel of scarring either from burn or skin disease
e. It may metastasize to lymph nodes and end fatally.

Q287. Predisposing factors to squamous cell carcinoma include the followings except
a. Granuloma inguinale
b. Xeroderma pigmentosum
c. Actinic keratosis
d. Condyloma lata
e. Albinism.

Q288. The typical features of basal cell carcinoma include the following except
a. Commonly affect the elderly
b. Metastatic spread to the lung if untreated
c. Commonly occur on the face
d. Presented clinically as papule with surface telangiectasia or ulcerated
nodule
e. Treated by excision or radiotherapy.

Q289. In urticaria, the followings are true except


a. Subcutaneous adrenaline is a life saving measure in angio-oedema of
laryngeal area
b. The C1 esterase inhibitor is lacking in patients with hereditary angioedema
49
c. Aspirin is the common cause of drug induced urticaria
d. Dermographism is a form of physical urticaria
e. Treatment with systemic steroid should be avoided in acute urticarial.

Q290. The most frequent causes of urticaria include the followings except
a. Water
b. Food
c. Drugs
d. Infection
e. Emotional stress.

Q291. Complication of acute urticaria include the followings except


a. Bronchospasm
b. Laryngeal edema
c. Hypotension
d. hypertension
e. Anaphylactic shock.

Q292. Physical urticaria include the followings except


a. Dermogrophism
b. Papular urticaria
c. Pressure urticaria
d. Cholinergic urticaria
e. Aquagenic urticarial.

Q293. In hereditary angioedema, the followings are true except


a. It is an inherited deficiency of the inhibitor of the first component of
complement (C1 Esterase inhibitor)
b. It is an autosomal dominant inherited disease
c. It is commonly appears in the 2nd to 4th decade of life
d. Urticaria or itching commonly occurs
e. There is little response to antihistamine, epinephrine or steroid.

Q294. Peripherally acting non-sedating antihistamine include the followings except


a. Desloratadine
b. Loratadine
c. Hydroxyzine
d. Cetirizine
e. Terfenadine.

Q295. Treatment of anaphylactic shock include the followings except


a. Maintenance of a patent airway (oxygen should be administered)
b. An intravenous line with either normal saline or ringer lactate
c. Epinephrine either subcutaneous or intravenous
d. Systemic steroid
50
e. Antihistamine are the drug of choice in sever life-threatening cases.

Q296. Causes of generalized pruritis including the following except


a. Diabetes mellitus
b. Thyroid disease
c. Liver disease
d. Chronic renal failure
e. Heart failure.

Q297. Severe pruritus is not a feature of


a. Atopic dermatitis
b. Lichen planus
c. Seborrheic dermatitis
d. Dermatitis herpetiformis
e. Scabies.

Q298. Systemic causes of pruritis include the following except


a. Iron deficiency anaemia
b. Lymphoproliferative and myeloproliferative diseases
c. Thyroid disease
d. Opiate and antidepressant drugs
e. Syphilis.

Q299. The following conditions are usually self-limiting except


a. Pityriasis rosea
b. Urticaria
c. Alopecia totalis
d. Erythema multiforme
e. Herpes simplex infection.

Q300. The following are recognized associations except


a. Streptococcal throat infection and guttate psoriasis
b. Herpes simplex infection and erythema nodosum
c. Diabetes mellitus and candidal intertrigo
d. Ulcerative colitis and Behcet's disease
e. Thyroid disease and vitiligo.

Q301. The following are recognized associations except


a. Staphylococcal infection and scalded skin syndrome
b. Behcet’s disease and erythema multiforme
c. Diabetes mellitus and candidal intertrigo
d. Diabetes mellitus and vitiligo
51
e. Alopecia areata and thyroid disease.

Q302. The following are recognized associations except


a. Dermatomyositis and nasopharyngeal carcinoma
b. Erythrasma and staphylococcus infection
c. Erythema multiforme and herpes simplex infection
d. Erythema nodosum and tuberculosis
e. Aphthous oral ulceration and Behcet’s syndrome.

Q303. The following are recognized associations except


a. Periungual fibromas and neurofibromatosis
b. V-shaped notches at the nail free edge and Darier disease
c. Nail pterygium and lichen planus
d. Splinter hemorrhage and pityriasis rubra pilaris
e. Subungual hyperkeratosis and psoriasis.

Q304. Causes of facial telangiectasia include the following except:


a. Rosacea
b. Topical steroid
c. Acne
d. Photoaging
e. Scleroderma.

Q305. Eye manifestation may be seen in the following except:


a. Rosacea
b. Xeroderma pigmentosum
c. Herpes simplex infection
d. Ecthyma
e. Behcet's disease.

Q306. Family history may be positive in the following except:


a. Psoriasis
b. Alopecia
c. Vitiligo
d. Dermatitis herpetiformis
e. Scabies.

Q307. Diagnostic morphological features of psoriatic lesion include all of the following
except:

a. Surrounding clear peripheral zone


b. Deep red colour often referred as salmon pink
52
c. Well-defined borders
d. Asymmetrical distribution
e. Silvery-white scaling.

Q308. All the following have been used as treatment for symptomatic pityriasis rosea
except:
a. Mid-potency topical corticosteroids
b. Oral H1 antihistamine
c. Short course of oral corticosteroid
d. Short course of oral retinoids (isotretinoin)
e. Ultraviolet B (UVB) phototherapy.

Q309. In patients with atopic dermatitis, which of the following adverse effects is not
associated with topical steroid?
a. Skin atrophy
b. Skin barrier dysfunction
c. Telangiectasia
d. Hyperpigmentation
e. Hypertrichosis.

Q310. Which of the following concerning pityriasis rosea is correct?


a. It is due to a fungal infection
b. It characterized by flat scaly patches
c. It is frequently associated with oro-genital itching
d. May be preceded by intense itching
e. Tends to commonly recur after apparent cure.

Q311. A 75-year-old female presents with chronic leg ulceration, which is a


consequence of venous insufficiency. Which one of the following is the most
appropriate management?
a. Appropriate systemic antibiotic in preparation for skin grafting
b. Compression bandaging
c. Improve the venous return by limb elevation
d. Skin biopsy to exclude neoplasm
f. Vein surgery.

Q312. A 68-year-old woman presents with a 2 months history of a widespread


pruritic rash. Examination reveals widespread erythema with several small tense
blisters containing straw-colored fluid. What is the most likely diagnosis?

53
a. Bullous impetigo
b. Bullous pemphigoid
c. Insect bite
d. Scabies
e. Urticarial vasculitis.

Q313. A 50-year-old man presented in the summer complaining of itching and


blistering of his hands and forehead. On examination, there were small areas of
excoriation on the backs of his hands. What is the most likely diagnosis?
a. Dermatitis herpetiformis
b. Lupus erythematosus
c. Pemphigoid
d. Pemphigus
e. Porphyria cutanea tarda.

Q314. A 40-year-old female presents with six months history of pruritic papules,
vesicles and excoriations on the elbows, knees, buttocks and scalp. Her GP has
prescribed topical betamethasone therapy, which has been unhelpful. What is the
most likely diagnosis?
a. Atopic dermatitis
b. Dermatitis herpetiformis
c. Hennoch-Schonlein purpura
d. Psoriasis
e. Scabies.

Q315. A 22 year-old male presents with generalized pruritus mainly at night of six
weeks duration. Examination reveals erythematous, excoriated papules between the
webs of the fingers and wrist. Which of the following therapies would be most
appropriate for this patient?
a. Topical steroid
b. Calamine lotion
c. Topical retinoid
d. Antihistamine cream
e. Permethrin cream.

Q316. A 40-year-old man presented with pityriasis versicolor. What is the most
appropriate treatment?
a. Methotrexate
b. Oral terbinafine
c. Psoralen with ultraviolet light (PUVA) therapy
d. Topical antifungal
e. Topical steroid.

Q317. Increased fragility of the skin, is characteristic of which one of the following
conditions?
a. Pityriasis rosea
54
b. Epidermolysis bullosa
c. Neurofibromatosis
d. Lichen planus
e. Tuberous sclerosis.

Q318. A 74-year-old man with a thirty years history of psoriasis presented with
generalized erythroderma of 3-days duration. Examination reveals generalized
erythema and scale covering most of the body. The patient treated as an inpatient
with emollients and attention to fluid replacement and temperature control but the
patient failed to improve after five days. What is the most appropriate next
treatment?
a. Oral hydroxycholoroquine
b. Oral methotrexate
c. Oral prednisolone
d. Topical coal tar
d. Topical dithranol.

Q319. A 43-year old woman with atopic dermatitis (atopic eczema) presented with
an acute generalized exacerbation of her disease. She admitted to hospital but not
improved with emollients, topical steroid and oral antihistamine. Which one of the
following drugs is the most appropriate treatment?
a. Acetretin
b. Amoxicillin
c. Cyclosporine
d. Colchicine
e. Dapsone.

Q320. Which of the following is a recognized feature of psoriasis?


a. Angular stomatitis
b. Iridocyclitis
c. Koebner Phenomenon
d. Hair loss
e. Response to chloroquine.

Q321. Which is true regarding eczema herpeticum?


a. It is invariably fatal if untreated
b. It has usually an indolent onset
c. a single group of vesicles usually appears at the site of the lesion
d. It is typically associated with a high fever for over a week
e. It is more severe in reactivation disease.

Q322. A 16-year-old boy presents with erythema nodosum and GIT symptom. Which
of the following should be considered?
a. Reiter's disease
55
b. Ulcerative colitis
c. Cytomegalovirus infection
d. Toxoplasmosis
e. Kawasaki Disease.

Q323. Which statement regarding tinea capitis is correct?


a. It is most commonly caused by microsporum canis
b. Its presence should suggest immunological deficiency
c. It often results in permanent alopecia
d. It may cause patches that fluoresce dull green under Wood's lamp
e. It effectively treated with topical nystatin ointment.

Q324. The following may be responsible for an acute relapse of systemic lupus
erythematosus in a 38-year old female except?
a. Hydralazine therapy
b. Menstruation and Pregnancy
c. Physical or mental stress
d. Cyclosporine therapy
e. Infection

Q325. A 38-year-old female presents with red target lesions confined to the hands
and is diagnosed with erythema multiforme. Which of the following could be the
cause?
a. Cytomegalovirus infection
b.Ureaplasma urealyticum infection
c. Group B Streptococcal infection
d. Herpes simplex infection
e.Chlamydial infection.

Q 326. The Following diseases are bacterial sexually transmitted except?


a.Syphilis
b.Gonorrhea
c. Chancroid
d. Granuloma inguinale
e. Scabies.

Q 327. The primary lesion in herpes simplex virus is:


a. Papules and nodules
b. Vesicles on erythematous base
c. Plaques
d. All of the above
e. None of the above.

Q328. Koebner's phenomenon is +ve in the following diseases except?


a. Vitiligo
b. Lichen planus
56
c. Viral warts
d. Psoriasis
e. All of the above.

Q329. The following diseases are caused by fungus except?


a. ORF infection
b. Chronic paronychia
c. Tinea capitis
d. Onychomycosis
e. All of the above.

Q330. Scapulae (sulfur cups), are characteristic which of the following disease?
a. Kerion
b. Tinea corporis
c. Favic type of tinea capitis
d. Scaly ringworm
e. None of the above.

Q331. Which of the following are the clinical types of cutaneous tuberculosis?
a. Tuberculous chancre
b. Lupus vulgaris
c. Scrofuloderma
d. ALL of the above
e. None of the above.

Q332. Which of the following period is the incubation period of syphilis?


a. One day
b. 2-5 days
c. 9-90 days
d. 2-5 weeks
e. One year.

Q333. In Kerion, the Following statements are correct except?


a. Caused by fungi of animal origin
b. Treated by systemic antifungal
c. May be mistaken with an abscess of bacterial origin
d. Treated by surgical excision
e. It may be fluorescence bright green under wood light.

Q334. What is the diagnosis of the loss of the hair when occur all over the body?
a. Alopecia universals
b. Alopecia Areata
57
c. Alopecia Totalis
d. Trichotillomania
e. Traction alopecia.

Q335. Annular lesion is seen in which of the following disease?


a. Acne vulgaris
b. Tinea corporis
c. Urticaria
d. Erythema multiforme
e. Viral warts.

Q336.The following skin diseases are premalignant except?


a. Arsenic keratosis
b. Seborrheic keratosis
c. Bowen's disease
d. Naevus sebaceous
e. Actinic keratosis.

Q337.Which of the following disease is the topical steroids is useful?


a. Dermatitis artifacta
b. Perioral dermatitis
c. Acne vulgaris
d. Rosacea
e. Hypertrophic scar.

Q338. Which of the following diseases are transmitted sexually?


a. Granuloma inguinale
b. Erythroplasia of queyrat
c. Pearly papules of penis
d. Lichen sclerosis et atrophicans
e. Fordyce spots.

Q339. The blood serology are very useful in which of the following disease?
a. Latent syphilis
b. Herpes genitals
c. Trichomonas vaginalis
d. Granuloma inguinale
e. Gonorrhea.

Q340. Which of the following medication based on FDA classifications is safe in


pregnancy?
a. Topical erythromycin/benzoyl peroxide gel
b. Topical tretinoin
58
c. Azarotene 0.1% cream
d. Azelaic acid 20% cream
e. Tetracycline.

Q341.In which disease, the lacrimal gland involvement is characteristically seen?


a. Lymphoma
b. Sarcoidosis
c. Rosacea
d. Syphilis
e. Pemphigus.
.
Q342. Which type laser would be appropriate to treat tattoo with red, orange,
yellow, and purple pigment?
a. Q switched Nd:YAG (532nm)
b. Q switched alexandrite (755nm)
c. Q switched ruby (694nm)
d. Nd:Yag (1064nm)
e. Alexandrite (755nm)

Q343. Which of the following is true regarding acquired C1 esterase inhibitor


deficiency?
a. The condition generally occurs in the first or second decade of life
b. Serum C1q is normal
c. C2 and C4 are both decreased
d. C1 esterase inhibitor may be at normal levels with functional impairment
e. Positive family history is common

Q344. Which type of test is likely to help in diagnosis of a patient who presents with
an annular patch on the body?
a. Gram’s stain
b. KOH scarping and microscopic examination
c. Tissue smear
d. Wood’s lamp examination
e. None of the above.

Q345. A 7-years old child presented with scaly hypopigmented patches on the face.
What is the most likely diagnosis?
a. Pityriasis alba
b. Post inflammatory hypopigmentation
c. Vitiligo
d. Post dermal leishmaniasis
e. Lepromatous leprosy.

Q346. Uses of wood’s lamp in dermatology include the followings?


a. Urine examination in porphyria
b. Tinea capitis
59
c. Vitiligo
d. Erythrasma
e. All of the above.

Q347. What is the commonest site of childhood atopic dermatitis?


a. Scalp
b. Ante-cubital fossa
c. Trunk
d. Feet
e. Hands.

Q348. The diagnosis of atopic dermatitis can be best done by?


a. Clinical evaluation
b. Patch testing
c. IgE level in the blood
d. Skin biopsy
e. IgG level in the blood.

Q349. Kaposi variceliform eruptions are seen in which of the following disease?
a. Atopic dermatitis
b. Dermatitis herpetiformis
c. Lymphoma
d. Coxsackie Virus
e. All of the above.

Q350. Air-borne contact dermatitis can be best diagnosed by?


a. Skin biopsy
b. Patch test
c. Prick test
d. Estimation of serum IgE levels
e. Estimation of serum IgM levels.

Q351.Which type of hypersensitivity reaction, the contact dermatitis is?


a. Type I
b. Type II
c. Type III
d. Type IV
e. None of the above.

Q352. Berloque dermatitis is due to contact with which of the following material?
a. Metals
b. Cosmetics
c. Food
60
d. Plants
e. Fish.

Q353. What is the treatment of choice of erythrodermic psoriasis?


a. Dapsone
b. Methotrexate
c. Systemic steroid
d. Topical steroid
e. Topical dithranol

Q354. Koebner’s phenomenon is seen in which of the following diseases?


a. Lichen planus
b. Psoriasis
c. Vitiligo
d. Viral warts
e. All of the above

Q355. Which of the following disease is itchy or pruritic?


a. Lichen planus
b. Psoriasis
c. Icthyosis
d. Secondary syphilis
e. Viral warts.

Q356. Photochemotherapy (PUVA) is used as treatment, in which of the following


disease?
a. Psoriasis
b. Pemphigus
c. Tinea capitis
d. Dermatitis herpetiformis
e. Trichotillomania.

Q357. The following can occur in psoriasis except?


a. Nail changes
b. Extensor distribution
c. Arthritis
d. Vesiculobullous lesion
e. Scalp involvement.

Q358. Which of the following drugs aggravate or induced psoriasis?


a. Lithium
b. B-Blockers
61
c. Antimalarial drugs
d. Aspirin
e. None of the above.

Q359. Vitamin D analogue (calcipitriol), is useful in the treatment of the following?


a. Cellulitis
b. Psoriasis
c. Pemphigus
d. Leprosy
e. Tinea capitis.

Q360. The following are causes of erythroderma or exfoliative dermatitis ?


a. Psoriasis
b. Eczema
c. Pityriasis rubra pilaris
d. Cutaneous lymphoma
e. All of the above.

Q361. Pitting of nails are seen in which of the following?


a. Alopecia areata
b. Psoriasis
c. Eczema
d. All of the above
e. None of the above.

Q362. Which part of the human body is tinea ungium affects?


a. Nail folds
b. Nail plates
c. Joints
d. Inter digital spaces
e. None of the above.

Q363. The following can be affected by psoriasis except?


a. Scalp
b. Nails
c. Central nervous system
d. Cardiovascular system
e. Joints.

Q364. Which of the following is a wrong statement?


a. Koilonychia (spoon nails) in vitamin B12 deficiency
b. Onycholysis in psoriasis
c. Mees lines in arsenic poisoning
62
d. Pterygium of nails in lichen planus
e. Auspitz sign in psoriasis.

Q365. Civatte bodies (colloid bodies) are found in which of the following?
a. Lichen planus
b. Psoriasis
c. Psoriasis
d. Dermatophytosis
e. Vitiligo

Q366. A 27 year-old man presents with numerous erosions of his mucous


membranes. Examination of his oral cavity reveals painful oral ulcers and a friable
mucosa. Immunofluorescence evaluation demonstrates intra-epidermal band of IgG.
Which of the following is the most likely diagnosis in this patient?
a. Herpetic ulcers
b. Measles
c. Dermatitis herpetiformis
d. Pemphigus vulgaris
e.Bullous pemphigoid

Q367. A woman presents to a dermatologist because she has lost almost all the hair
on her body, including scalp hair, eye brows, eye lashes, arm pit and groin hair, and
the fine hairs on her body and extremities. She most likely has a variant of which of
the following?
a. Alopecia areata
b. Androgenic alopecia
c. Chronic cutaneous lupus erythematosus
d. Lichen planopilaris
e. Trichotillomania

Q368. A 23 year-old obese female with known tuberculosis presents with ulcerating
nodules on the back of her legs. Which of the following is the most likely diagnosis.
a. Erythema marginatum
b. Erythema nodosum
c. Erythema induratum
d. Lupus vulgaris
e. Lupus pernio

Q369. Recognized causes of erythema nodosum include all the following except?
a. Behcet’s disease
b. Sarcoidosis
c. Oral contraceptives

63
d. Inflammatory bowel diseases
e. Diabetes mellitus

Q370. Which of the following would be prescribed for acne?


a. Benzoyl peroxide
b. Topical antibiotic
c. Topical retinoid
d. All of the above
e. None of the above

Q371. Diabetic male patient presented to the dermatology clinic with a curdy white
patch on the tongue. The most probable diagnosis is?
a. Oral Candidiasis
b. Oral Lichen planus
c. Oral pemphigus
d. Erythema multiforme
e. None of the above.

Q372. A 55-year-old obese patient presents with dirty, velvety patches on the back
of the neck:
a. Keloids
b.Erythema nodosum
c. Acanthosis nigricans
d. Erythema multiforme
e. Acne vulgaris.

Q373. A37-year-old man, who works in a fish market, presents with a burning pain in
his right hand for one-week duration. Physical examination reveals a large,
violaceous plaque on his finger. Gram stain reveals no organism. Which of the
following is the most likely diagnosis?
a. Erythrasma
b. Ecthyma
c. Erysipelas
d. Erysipeloid
e. Discoid eczema.

Q374. A 17-year-old patient presents with severe pruritus that is worse at night. On
examination of the skin, multiple excoriated papules are observed in the interdigital
spaces. Family members report similar symptoms. Which of the following is the most
likely diagnosis?
64
a. Scabies
b. Cutaneous larva migrans
c. Contact dermatitis
d. Dermatitis herpetiformis
e. Impetigo.

Q375. A 16-year-old student with a history of herpetic gingivostomatitis develops a


generalized and symmetric rash. On examination, there are multiple patches with
concentric rings surrounding a central disk. The rash is burning and pruritic. A few
erosive lesions are visible in the oral mucosa. Which of the following is the most
likely diagnosis?
a. Erythema multiforme
b. Secondary syphilis
c. Systemic lupus erythematosus
d. Pemphigus vulgaris
e. Urticaria

Q376. What is the most common type of skin cancer?


a. Melanoma
b. Basal cell carcinoma
c. Squamous cell carcinoma
d. Merkel cell carcinoma
f. Kaposi sarcoma

Q377. A 7-years-old girl presented to the dermatology clinic because of a rash that
appeared 3 days ago. Her temperature is 37.2°C (99°F) and her face has an intense
rash with a “slapped-cheek” appearance. The most likely etiologic agent is?
a. Adenovirus infection
b. Rotavirus infection
c. Parvovirus infection
d. Coxsackie virus infection
e. Echovirus infection

Q378. A 3-month-old male infant developed otitis media for which he was given a
course of antibiotics. A few days later, he developed a macular erythema followed by
diffuse epidermal exfoliation. There were no mucosal lesions and the baby was not
toxic. The most likely diagnosis is?
a. Toxic epidermal necrolysis
b. Staphylococcal scalded skin syndrome
c. Steven Johnson syndrome
d. Infantile pemphigus
e. Erythema infectiosum

Q379. What is the most common types of melanoma?


a. Superficial spreading melanoma
b. Acral lentiginous melanoma
65
c. Mucosal melanoma
d. Nodular melanoma
e. Polypoid melanoma.

Q380. What skin condition is caused by poxvirus?


a. Verruca
b. Molluscum contagiosum
c. Impetigo
d. Cellulitis
e. Chicken pox.

Q381. What condition leads to squamous cell carcinoma (SCC)?


a. Keratoacanthoma
b. Actinic keratosis
c. Viral warts
d. ORF infection
e. Seborrheic keratosis.

Q382. An elderly woman presents to the dermatology clinic complaining of a raised,


round discolored plaque, ‘stuck on appearance’ on her face. What skin condition is
this?
a. Ichthyosis vulgaris
b. Seborrheic keratosis
c. Basal cell carcinoma
d. Melasma
e. keratoacanthoma.

Q383. What childhood infection is associated with Koplik spots?


a. Measles
b. Rubella
c. Fifth disease or erythema infectiosum
d. Chicken pox
e. Herpes simplex infection.

Q384. What skin condition is characterized by an initial ‘herald patch’ which is then
followed by scaly erythematous plaques usually in a ‘Christmas tree’ distribution?
a. Pityriasis rosea
b. Lichen planus
c. Psoriasis
d. Pityriasis rubra pilaris
e. Ichthyosis.

Q385. How does impetigo present?


a. Golden honey colored crust over an erythematous base
b. Salmon colored plaque with silvery scale
66
c. Comedones, pustules and nodules
d. Flesh colored papule with a rough surface
e. Dome-shaped papule with central umbilication.

Q386. What is the most common mole or nevus found in adults?


a. Junctional nevus
b. Compound nevus
c. Intradermal nevus
d. Congenital melanocytic nevus
e. Mongolian spot.

Q387. What is the most common causative agent of erythema multiforme (EM)?
a. Drugs (Penicillin and sulphonamide)
b. Systemic lupus erythematosus
c. HSV infection
d. Malignancy
e. Pregnancy.

Q388. What is the infective agent implicated in acne?


a. Staphylococcus aureus
b. Streptococcus pyogenes
c. Staphylococcus epidermidis
d. Propionibacterium acnes
e. Escherichia coli.

Q389. What is the pathogenesis of pemphigus vulgaris?


a. IgG antibody against desmoglein
b. IgG antibody against hemidesmosome components
c. Autoimmune deposition of IgA at tips of dermal papillae
d. Enzyme defect in tyrosinase enzyme
e. None of the above.

Q390. What is the best indicator of prognosis for a melanoma?


a. Asymmetry
b. Colour
c. Shape
d. Diameter
e. Invasion of the dermis.

Q391. What is the pathogenesis of vitiligo?


a. Congenital lack of pigmentation
b. Increase in the number of melanosomes
67
c. Autoimmune destruction of melanocytes
d. Benign proliferation of melanocytes
e. Defect in tyrosinase enzyme.

Q392. What is Leser-Trélat sign?


a. Sudden appearance of multiple seborrheic keratosis and is an indicator
of a gastrointestinal tract carcinoma
b. Separation of the nail plate from the nail plate
c. Metastasis of gastric carcinoma to the periumbilical region
d. Pin point bleeding appear when psoriatic scale is forcely removal
e. Appearance of the lesion on the site of trauma.

Q393. What are the histological findings of psoriasis?


a. Inflammation of the dermal-epidermal junction
b. Peripheral palisading of basal cells
c. Acanthosis, Parakeratosis and Munro microabscesses
d. Keratin pseudocysts
e. Band-like infiltration of lympho-histiocytic cells in the upper dermis.

Q394. What type of melanoma is often seen in dark skinned individuals?


a. Superficial spreading melanoma
b. Lentigo maligna melanoma
c. Nodular melanoma
d. Acral lentiginous melanoma
e. None of above

Q395. What condition is associated with this presentation? – A pink pearly nodule
with telangiectasia, ulceration and rolled borders on the upper lip?
a. Squamous cell carcinoma
b. Basal cell carcinoma
c. Melanoma
d. Kaposi sarcoma
e. Eczema.

Q396. How does lichen planus present clinically?


a. Salmon colored plaques with silvery white adherent scale
b. Pruritic, red, oozing rash with edema
c. papular lesion with golden colored crusts
d. Pruritic, purple, polygonal, planar papules and plaques
e. Dome-shaped papule with central umbilcation or depression.

Q397. What is the wavelength of a Wood's light?


a. 290 nm
b. 311 nm
68
c. 330 nm
d. 365 nm
e. 410 nm.

Q398. The following are true about UVA radiation except?


a. 10 times more abundant than UVB
b. It penetrates to a greater depth in the dermis than UVB
c. It is responsible for phototoxic drug reactions
d. It is approximately 50% of exposure occurs in the shade
e. Virtually all blocked by car window glass.

Q399. What is the best location for photo-testing patients suspected of


photosensitivity?
a. Affected skin of the buttock
b. Unaffected skin of the lower back
c. Affected skin of the ventral forearm
d. Unaffected skin of the upper back
e. Unaffected skin of the outer thigh.

Q400. Which of the following medications on the patient's medication list is your top
choice for discontinuation in patient with porphyria cutanea tarda?
a. Naproxen
b. Paracetamol
c. Enalapril
d. Chloroquine
e. Multivitamins.

Q401. What is the wavelength of narrow-band UVB bulbs?


a. 290-320 nm
b. 311 nm
c. 352 nm
d. 305 nm
e. 360 nm.

Q402. In solar urticaria, what is the time the wheals typically start?
a. Start at 45 minutes after exposure and last 2 hours
b. Start at 1-2 h and last 2-4 hours
c. Start at 15-30 minutes and last 1 hour
d. Start at 30-60 minutes and last 1-4 hours
e. Start at 15-30 seconds and last about 15 minutes.

Q403. Which of the following is not true regarding polymorphous light eruption?
a. Usually appears in the first three decades
b. May be a manifestation of a type IV hypersensitivity reaction
c. Vesicles and an eczematous dermatitis are a common presentation
69
d. Not all exposed areas show lesions
e. It may occur through window glass, which filters out UVB.

Q404. What is the wavelength of UVC portion of the electromagnetic spectrum?


a. 10-200 nm
b. 200-290 nm
c. 290-320 nm
d. 400-760 nm
e. None of these answers is correct.

Q405. The following medication can cause photosensitivity except?


a. Thiazides
b. Cephalosporin
c. Phenothiazenes
d. Quinolones
e. Doxycycline.

Q406. Which of the following is true regarding immediate pigment darkening?


a. Contributes to constitutive skin color
b. Caused by UVA radiation
c. Prominent in lightly pigmented individuals
d. Requires the synthesis of new melanin
e. Becomes prominent 48 hour after exposure.

Q407. Photo-onycholysis has been attributed to which of the following drugs?


a. Quinolones
b. Tetracyclines
c. Psoralens
d. Quinine
e. All of the above.
Q408. A joule, is a measurement of which of the following?
a. Energy
b. Power
c. Irradiance
d. Wavelength
e. Distance.

Q409. In solar urticaria, which wavelength the wheals may be brought on?
a. UVA
b. UVB
c. UVC
d. Visible light
e. UVA, UVB and visible light.
Q410. In blue-gray pigmentation of the lunulae, which of the following is true?
70
a. Can be seen in patients on amiodarone
b. Occurs only after many years of treatment with chlorpromazine
c. Is seen in argyria (prolonged contact with or ingestion of silver salts)
d. Is a rare side effect of treatment with tricyclic anti-depressants
e. None of these answers are correct.

Q411. Which of the following skin type, a patient that rarely burns and usually tans?
a. Type II
b. Type II
c. Type III
d. Type IV
e. Type V.

Q412. Which of the following skin type, a patient that always burns and never tans?
a. Type I
b. Type II
c. Type III
d. Type IV
e. Type V.

Q413. Which of the following skin type, a patient that usually burns and never tans?
a. Type I
b. Type II
c. Type III
d. Type IV
e. Type V.

Q414. Which of the following skin type, a patient that never burns and always tans?
a. Type I
b. Type II
c. Type III
d. Type IV
e. Type V.

Q415. Which of the following skin type, the highly pigmented individuals?
a. Type I
b. Type II
c. Type III
d. Type IV
e. Type V.

71
Q416. Which spectrum of ultraviolet is responsible for the conversion of 7-
dehydrocholesterol in the skin to pre-vitamin D3?
a. 200-290 nm
b. 290-320 nm
c. 320-340 nm
d. 340-400 nm
e. 10-200 nm.

Q417. What is the wavelength of the UVAII portion of the electromagnetic


spectrum?
a. 200-290 nm
b. 290-320 nm
c. 320-400 nm
d. 320-340 nm
e. None of these answers are correct.

Q418. The most common presentation of a patient with drug photosensitivity?


a. Photo-onycholysis
b. Lichenoid eruptions
c. Diffuse erythema in sun-exposed areas
d. Pseudo-porphyria
e. Fixed erythematous patch.

Q419. Which of the following is a manifestation of dermatoheliosis (photo-aging)?


a. Hydroa vacciniforme
b. Actinic prurigo
c. Chronic actinic dermatitis
d. Brachioradial pruritis
e. Cutis rhomboidalis nuchae.

Q420. Which of the following drug is the most likely cause of photosensitivity?
a. Quinolones
b. Sulfonamides
c. Doxycycline
d. Minocycline
e. Penicillin V.

Q421. Which of the following medications would be least likely to cause onycholysis
after sun exposure?
a. Quinolones
b. Tetracyclines
c. Psoralens
d. Quinine
e. Chlorpromazine.

72
Q422. Which of the following disease, PUVA has been used to treat?
a. Cutaneous T-cell Lymphoma
b. Mastocytosis
c. Graft-versus-host disease
d. Generalized granuloma annulare
e. All of these answers are correct.

Q423. The following are the acute effects of ultraviolet radiation from the sun except?
a. Epidermal thickening
b. Photosynthesis of vitamin D
c. Sun-burning
d. Immediate pigment darkening
e. Increased immune surveillance.

Q424. Which patient most likely has a chronic actinic dermatitis?


a. A 30 year old female with erythematous papules on the chest that recur
each summer
b. A girl with excoriated papules and nodules on sun-exposed and non-sun-
exposed areas with cheilitis
c. A Native American adult with papules excoriated dermatitis on the face
with cheilitis
d. A middle-aged male with recurrent wheals begin 20 minutes after sun
exposure
e. An elderly man with persistent eczematous dermatitis in
a photo-distribution.

Q425. The following drugs are the most common cause of photosensitivity except?
a. Tetracycline
b. Sulfonamides
c. Diphenhydramine
d. Paracetamol
e. Hydroxychloroquine.

Q426. Which of the following is true as a result of ultraviolet radiation?


a. There is mast cell degranulation and release of histamine
b. Prostaglandins are increased
c. Epidermal thickening occurs
d. All of these answers are correct
e. None of these answers are correct.
Q427. Which type of fruits / vegetables can cause phyto-photo-dermatitis?
a. Potatoes and leeks
b. Tomatoes and bananas
c. Celery and radishes
d. Parsnips and limes
73
e. Lemons and pears.
Q428. Currently the most common cause of photoallergic contact dermatitis is?
a. Halogenated salicylanilides
b. Musk ambrette
c. 6-methylcoumarin
d. Sunscreens
e. Topical antibiotics.

Q429. What is the wavelength of the narrow band UVB ultraviolet radiation?
a. 300-306nm
b. 308-310nm
c. 311-313nm
d. 312-320nm
e. 320-330nm.
Q430. The followings are the side effects of PUVA except?
a. Nausea
b. Hair loss
c. Painful erythema
d. Prolonged pruritus
e. Squamous cell carcinoma.

Q431. Which of the following ultraviolet radiation causing erythema of the skin?
a. UVC
b. UVB
c. UVA1
d. UVA2
e. Visible light.

Q432. Which of the following is true in solar urticaria?


a. It can present with headache, nausea, and syncope
b. Rarely lasts for more than 6 months
c. Usually occurs to only UVB radiation
d. Antihistamines is very effective.
e. It Is an idiopathic, type IV photosensitivity disorder.

Q433. Which of the following drugs is commonly known to produce photosensitivity?


a. Quinidine
b. Sulfonylureas
c. Griseofulvin
d. All of these answers are correct
e. None of these answers are correct.
Q434. What is the wavelength of UVA II radiation?
a. 290-320 nm
b. 320-400 nm
c. 400-450 nm
d. 320-340 nm

74
e. 340-400 nm.

Q435. Which of the following statements about the spectrums of UVR that cause of
solar urticaria is most correct?
a. Visible light causes solar urticaria
b. UVA causes solar urticaria
c. UVB causes solar urticaria
d. Both UVA and UVB cause solar urticaria
e. Patients can react to visible light, UVA and/or UVB.
*Q436. Which of the following is the most common photo-dermatosis?
a. Hydroa vacciniforme
b. Chronic actinic dermatitis
c. Actinic prurigo
d. Polymorphous light eruption
e. Solar urticaria.

*Q437. In solar urticaria. Which test should be considered initially?


a. ANA
b. ANCA
c. Urine analysis
d. Chest x-ray
e. ESR.

*Q438. Which of the following conditions would be least likely to be photo-


exacerbated?
a. Herpes simplex
b. Pellagra
c. Transient acantholytic dermatosis
d. Psoriasis
e. Pyridoxine deficiency

*Q439. A patient presents with blue-gray pigmentation on sun-exposed areas but


does not have involvement of the sclerae, lunulae or mucous membranes. Which of
the following medications could be causing this pigmentation?
a. Clindamycin
b. Chlorpromazine
c. Ciprofloxacin
d. Tetracycline
e. Griseofulvin.

Q440. In which of the following material, the contact allergens (5-methoxypsoralen) is


present?
a. Oil of Bergamot
75
b. Balsam of Peru
c. Diproserone
d. Dithranol
e. Epoxy resin.

Q441. The following are the advantages of narrowband UVB over PUVA therapy
except?
a. No need for protective eyewear
b. No nausea
c. Safe in childhood
d. Safe in pregnancy
e. More effective in treating thick plaques of CTCL.

Q442. Which of the following cell types induce susceptibility to tumor growth?
a. Suppressor T-cells
b. Natural Killer cells
c. Helper T-cells
d. Mast cells
e. Langerhans cells
Q443. UVB converts 7-dehydrocholesterol in the skin to which of the following?
a. 1,25-dihydroxyvitamin D3
b. 25-hydroxyvitamin D
c. Calcitriol
d. Previtamin D3
e. None of the above

Q444. Which of the following is true regarding immediate pigment darkening?


a. Is associated with an increase in melanocyte number
b. Is predominately brought on by UVB
c. Start 45-60 minutes after exposure
d. Is predominately brought on by UVA and visible light
e. Is caused by an increase in tyrosinase activity
Q445. Which of the following statement is true about ultraviolet C?
a. UV-C does not reach the earth's surface
b. UV-C is absorbed by atmospheric ozone
c. UV-C has an electromagnetic spectrum from 200-290 nm
d. UV-C has higher energy than UV-B
e. UV-C has a higher wavelength than UV-B

*Q446. Which of the following is not true about UVB radiation?


a. Responsible for sunburn
76
b. More intense in the summer than winter months
c. Peaks at noon
d. Decreased with high wind velocity
e. Virtually all blocked by car window glass
f.
*Q447. A patient presents with purple polygonal pruritic papules on sun-exposed
areas. Which of the following medication would not be suspect of causing this
eruption?
a. Enalapril
b. Quinidine
c. Antimalarial
d. Furosemide
e. Hydrochlorothiazide
Q448. The following genodermatoses are exacerbated by sun exposure except?
a. Tuberous sclerosis
b. Cockayne's Syndrome
c. Hailey-Hailey Disease
d. Hartnup Disease
e. Darier's Disease
Q449. Which of the following is true regarding actinic prurigo?
a. Lesions generally continue through late adulthood
b. Lesions persist for 1-2 days
c. Lesions never occur on non-sun-exposed areas
d. Cheilitis is frequently seen
e. Thalidomide has been ineffective for the majority of patients

Q450.What is the treatment of choice for patient with widespread plaque-type


psoriasis without arthritis?
a. PUVA
b. Cyclosporine
c. Narrowband UVB
d. Etretinate
e. Methotrexate
Q451.The following are true regarding polymorphous light eruption except:
a. An itchy rash appears within hours or days after exposure to sunlight
b. It lasts for up to 2 weeks and heal without scarring
c. It usually appear on the sun-exposed parts and the face is not always
affected.
d. It is more common in men than women
e. People with polymorphic light eruption are at greater risk of vitamin D
deficiency.

77
Q452.What is the ideal time of lesions of polymorphous light eruption typically
appear?
a. About one hour after exposure
b. Hours to days after exposure
c. Days to two weeks after exposure
d. 15-30 minutes after exposure
e. Immediately

Q453.Which of the following patients would be the most likely to present with chronic
actinic dermatitis?
a. Children
b. Teenagers
c. An elderly
d. Young males
e. Young females

Q454.The following drugs can cause lichenoid eruption or lichen planus except?
a. Anticonvulsants (tegretol or phenytoin)
b. Antihypertensive (beta-blockers or methyldopa)
c. Diuretics (furosemide or hydrochlorothiazide)
d. Hydroxychloroquine
e. Griseofulvin

*Q455. What range of ultraviolet radiation has been shown to be most efficient in
inducing neoplasia in mice?
a. 290-320nm
b. 320-340nm
c. 340-400nm
d. 400-760nm
e. >760nm

*Q456. Which of the following treatment used in mild cases of polymorphic light
eruption?
a. Sunscreen and sun avoidance between 11 am and 3pm
b. UVB hardening/desensitization
c. Chloroquine
d. Prednisone
e. Cyclophosphamide

*Q457. What is the diagnosis of malnourished individual presented with crusting and
hyperpigmentation in a photo-distribution?
a. Chronic actinic dermatitis
b. Photo allergic contact dermatitis
c. Scurvy
d. Pellagra
e. Polymorphic light eruption
78
*Q458. Which of the following treatment used for patient with chronic actinic
dermatitis?
a. Filters for car windows
b. Broad spectrum sunblock to which the patient is not allergic
c. Systemic steroid
d. PUVA
e. All of these answers are correct.

*Q459. Which of the following treatment used for patient with chronic actinic
dermatitis?
a. Emollient and Topical steroids
b. Topical tacrolimus or pimecrolimus
c. Azathioprine
d. Cyclosporine
e. All the answers are correct

Q460. The following are true regarding phototoxic drug reaction except?
a. The drug may become activated by exposure to sunlight and cause damage to
the skin
b. The skin appearance resemble sunburn
c. UVA radiation is most commonly associated with Phototoxicity
d. Common phototoxic drugs include antibiotics and antihistamines
e. Immunologically mediated reaction.

Q461. Which of the following is true regarding phototoxic reactions?


a. Are immunologically mediated
b. Occur only in predisposed individuals
c. Rarely occur on the first exposure to the chemical
d. the skin appearance resembles sunburn
e. Resolve with hyperpigmentation

Q462. The following treatment used in the treatment of vitiligo except?


a. Topical steroids
b. Narrowband UVB
c. Broadband UVB
d. Oral PUVA
e. Acitretin

Q463. The wavelength range that most effectively induce cutaneous


immunosuppression?
a. 200-290nm
79
b. 290-320nm
c. 320-340nm
d. 340-400nm
e. 400-410nm

Q464. The immunologic effects of UVR include all of the following except?
a. Alteration of Langerhans cell morphology
b. Decrease in IL-1, IL-6, TNF-alpha
c. Suppression of delayed-type hypersensitivity
d. Alteration of lymphocyte population
e. Alteration of Langerhans cell function

*Q465. Which of the following is true about ultraviolet light?


a. UVA radiation is 1000 times greater than UVB during midday hours
b. UVB radiation is 1000 times more erythrogenic than UVA
c. Sunlight early in the morning and late in the day contains relatively more
UVB
d. The wavelength of UVAII light is between 340-400nm
e. Clouds absorb most UVA light

Q466. What is the main component of the lamina densa?


a. Melanin
b. Keratin
c. Collagen III
d. Collagen VII
e. Collagen I

Q467. What is the histological finding in allergic contact dermatitis?


a. Psoriasiform dermatitis
b. Lichenoid infiltrate
c. Spongiosis
d. Parakeratosis
e. Granuloma.

Q468. A patient who works in a photography developing laboratory, complains of


eczematous dermatitis on his hands. Which one of the following chemicals is he most
likely allergic to?
a. Mercaptobenzothiazole
b. Lanolin
c. Epoxy resin
d. Para-pheylene-diamine
e. Colophony.

Q469. In which of the following dermatosis the characteristic eosinophilic abscesses


are seen?
a. Bullous drug eruption
80
b. Pemphigus vegetans
c. Herpes gestationis
d. Incontinentia pigmenti
e. Paraneoplastic pemphigus.

Q470. Which of the following can prevent fiberglass dermatitis?


a. Water
b. Acetic acid 5%
c. Sodium chloride
d. Alkali
e. Talcum powder.

Q471. The following is true regarding Leiner's disease (erythroderma desquamativum)


except?
a. Generalized seborrheic dermatitis
b. Recurrent diarrhea
c. Recurrent skin and internal infection
d. Failure to thrive
e. More common in males than females.

Q472. Which of the following drugs associated with the exacerbation of pemphigus
foliaceus?
a. Captopril
b. Penicillamine
c. Piroxicam
d. Rifampicin
e. All the answers are correct
Q473. The following drugs can cause SJS/TEN except?
a. Sulfonamides
b. Carbamazepine
c. Allopurinol
d. ketoconazole
e. NSAIDs

Q474. What is most common allergen causing allergic contact dermatitis?


a. Bacitracin
b. Formaldehyde
c. Quaternium-15
d. Neomycin
e. Nickel

Q475. The following are true regarding pemphigus erythematosus except?


81
a. Is also called Hallopeau syndrome
b. May be an abortive form of sub-corneal pustulosis
c. Is often in a malar/seborrheic distribution
d. Does not have linear IgG and C3 at the basement membrane zone
e. All of these answers are correct.

Q476. Papillary dermal deposits of IgA and a papillary dermal infiltrate of neutrophils
is diagnostic of which of the following ?
a. Sweet's syndrome
b. Leukocytoclastic vasculitis
c. Dermatitis herpetiformis
d. Linear IgA dermatosis
e. Bullous pemphigoid

Q477. Which of the following substances is a known to cause a delayed positive patch
test reaction?
a. Gold
b. Nickel
c. Bacitracin
d. Fragrance mix
e. Rosin

Q478. In which stage of syphilis, bullous lesions can be seen?


a. Congenital syphilis
b. Primary syphilis
c. Secondary syphilis
d. Tertiary syphilis
e. All of these answers are correct

Q479. A 35-year-old dentist presents with tingling in his fingertips. What allergen is
most likely to cause this dermatitis?
a. Methyl methacrylate
b. Paraphenylenediamine
c. Colophony
d. Thiuram mix
e. Ethylenediamine dihydrochloride

Q480. Paraneoplastic pemphigus, is associated with which of the following neoplasm?


a. Thymoma
b. Chronic lymphocytic leukemia
c. Castleman’s disease
d. Non-Hodgkin's lymphoma
e. All of the answers are correct

82
Q481. Irritation of the hands produced by capsaicin can relieved by application with
what of the following?
a. Water
b. Acetic acid 5%
c. Sodium chloride
d. Alkali
e. Talcum powder.

Q482. Which of the following is true regarding para-neoplastic pemphigus?


a. Severe painful oral erosion, frequently hemorrhagic, which spread to
involve the entire vermilion and tongue.
b. It is most often seen in association with lung cancer
c. It does not remit even if the cancer is excised completely
d. It cannot be caused by a benign neoplasm
e. All of these answers are correct.

Q483. The most common malignancy associated with para-neoplastic pemphigus is?
a. Non-Hodgkin's lymphoma
b. Chronic lymphocytic leukemia
c. Multiple myeloma
d. Acute myelocytic leukemia
e. Hodgkin's lymphoma.

Q484. Patients with Senear-Usher syndrome are most likely to present with:
a. Erythematous papules and plaques around the umbilicus
b. Transient vesicles on the oral mucosa
c. Severely pruritic grouped vesicles symmetrically distributed primarily
on extensor surface
d. Erythema multiforme-like oral ulcerations which are severe
e. Erythematous crusts and hyperkeratotic lesions on the nose, ears, ch
eeks, scalp and chest.

Q485. What is the most likely allergen in a patient develops erythema and vesiculation
on the dorsal foot that spares the toe webs and soles after buying a new pair of
shoes.?
a. Rubber
b. Carbamates
c. Latex
d. Mercaptobenzothiazole
e. Formaldehyde

Q486. Which of the following drug can induce linear IgA (drug-induced linear IgA)?
83
a. Vancomycin
b. Paracetamol
c. Griseofulvin
d. Metronidazole
e. Etretinate.

Q487. Which association is incorrect?


a. Epidermolysis bullosa acquisita and inflammatory bowel disease
b. Dermatitis herpetiformis and small bowel lymphoma
c. Paraneoplastic pemphigus and non-Hodgkin lymphoma
d. Herpes gestationis and menopause age
e. Psoriasis and Koebner phenomenon

Q488. What is the finding of direct immunofluorescent studies in a patient with


bullous pemphigoid?
a. Linear IgA at the basement membrane
b. Linear C3 at the basement membrane
c. Granular IgA in dermal papillae
d. Intercellular IgG throughout the epidermis
e. C3 in the dermal papillae.

Q489. Which of the following is associated with dermatitis herpetiformis?


a. Gluten-sensitive enteropathy
b. Inflammatory bowel disease
c. Autoimmune hepatitis
d. Herpes labialis
e. Rheumatoid arthritis

*Q490. Which form of epidermolysis bullosa presents with generalized bullae, absent
nails, dysplastic teeth (due to enamel defects), non-healing granulation tissue
periorally, and is often fatal by age 3-4?
a. Weber-cockayne
b. Herlitz type of junctional epidermolysis bullosa
c. Non-Herlitz type of junctional epidermolysis bullosa
d. Hyperplastic cockayne-touraine
e. Albapapuloid Pasini variant
*Q491. Which of the following can be responsible for contact dermatitis to K-Y Jelly?
a. Lanolin alcohol
b. Budesonide
c. Alpha tocopherol
d. Propylene glycol
e. Triclosan

Q492. What is the typical site for Bullous diabeticorum?


a. Face
b. Chest
84
c. Arms
d. Legs
e. Groin

Q493. Where the defect in junctional epidermolysis bullosa is?


a. Granular layer
b. Spinous layer
c. Lamina lucida
d. Lamina densa
e. Sublamina densa

Q494. Which disease is commonly associated with herpes gestationis?


a. Grave's disease
b. Hashimoto's thyroiditis
c. Diabetes mellitus
d. Lupus erythematosus
e. Rheumatoid arthritis

Q495. What is the name of the antigen associated with dermatitis herpetiformis?
a. Tissue transglutaminase
b. Epilegrin
c. Desmocollin
d. Desmoglein-1
e. Periplakin

Q496. In which layer of the skin the split occur in penicillamine-induced pemphigus?
a. Intraspinous
b. Suprabasal
c. Intraepidermal and subepidermal
d. Subepidermal
e. Subcorneal

Q497. A one-day old infant presents with papules and pustules on the face, trunk,
and proximal extremities. Palms and soles are spared. The patient is otherwise doing
well. A biopsy shows numerous eosinophils. What is the diagnosis?
a. Acropustulosis of infancy
b. Herpes infection
c. Erythema toxicum neanotorum
d. Scabies
e. Transient neonatal pustular melanosis.

Q498. Which of the following is commonly associated with herpes gestationis?


a. Lymphoma
b. Multiparty
85
c. Grave's disease
d. Inflammatory bowel disease
e. Lupus erythematosus.

Q499. What is the direct immunofluorescent finding in chronic bullous disease of


childhood?
a. IgA deposition in the superficial blood vessels
b. Linear IgG at the basement membrane
c. Linear IgA deposition at the basement membrane
d. Granular IgG at the basement membrane
e. Linear C3 at the basement membrane.

Q500. In staphylococcal scalded skin syndrome, the exfoliative toxin cleaves:


a. Desmoglein 1
b. Desmoglein 3
c. Desmocollin 1
d. Desmocollin 3
e. Bullous pemphigoid antigen-2 (BPAg-2)

.
Q501. Which of the following drug is the first line treatment for dermatitis
herpetiformis?
a. Methotrexate
b. Dapsone
c. Colchicine
d. Prednisone
e. IVIG
.
Q502. Which of the following diseases associated with paraneoplastic pemphigus?
a. Lympho-proliferative disorders
b. Colorectal adenocarcinoma
c. Breast Cancer
d. Sarcomas
e. Melanoma

Q503. Which of the following agent(s) has been most effective in treating severe
ocular cicatritial pemphigoid?
a. Cyclophosphamide
b. Mycophenolate mofetil
c. Cyclophosphamide + corticosteroids
d. Cyclosporine
e. Azathioprine

Q504. An elderly gentleman with a history of hypertension and a thymoma, presents


to the emergency room with fever and painful rash on the arms, legs, trunk, neck
and face. What is the most likely diagnosis?
86
a. Cicatritial pemphigoid
b. Sweet syndrome
c. Toxic epidermal necrolysis
d. Paraneoplastic pemphigus
e. Erythema multiforme

Q505. Which of the following is known as "prickly heat"?


a. Miliaria crystalline
b. Miliaria rubra
c. Miliaria profunda
d. Grover's disease
e. Fox-Fordyce disease

Q506. Which type of epidermolysis bullosa has the greatest risk of developing
squamous cell carcinomas?
a. Weber-Cockayne
b. Dowling-Meara
c. Herlitz type of junctional epidermolysis bullosa
d. EB Simplex with muscular dystrophy
e. Hallopeau-Siemens type of recessive dystrophic EB
Q507. Which of the following aggravate herpes gestationis?
a. Oral contraceptives
b. Menstruation
c. Third trimester
d. Postpartum state
e. All of these answers are correct

Q508. Which of the following immunosuppressive agents has been most effective in
cicatritial pemphigoid?
a. Cyclophosphamide
b. Azathioprine
c. Mycophenolate mofetil
d. Cyclosporine
e. Methotrexate

Q509. A patient with a history of leukemia that had severe, persistent painful
stomatitis extending from the lips into the pharynx, larynx and esophagus.
Conjunctival involvement may lead to blindness. The cutaneous changes are
polymorphic, ranging from erythematous macules to lichenoid papules to blisters and
erosions. What is the most likely diagnosis?
a. Paraneoplastic pemphigus
b. Erythema multiforme
c. Cicatritial pemphigoid
d. Chemotherapy induced stomatitis
e. Persistent herpes simplex

87
Q510. Which HLA type is the dermatitis herpetiformis associated?
a. HLA-Cw6
b. HLA-B27
c. HLA-DR4
d. HLA-DQ1
e. HLA-DR3
Q511. What is Asboe-Hansen sign?
a. Lateral slipping of the epidermis when perilesional is rubbed
b. Lateral dissection of the blister when pressure is applied directly to a
blister
c. blister formation following stroking of the lesion
d. Blister induction with incised trauma to skin
e. Blister formation after cryosurgery.

Q512. An older patient with erythema and crusting in a butterfly distribution and
seborrheic areas with erosions is diagnosed with pemphigus erythematosus.
The other synonym for pemphigus erythematosus is:
a. Senear-Usher syndrome
b. Wildfire pemphigus
c. Fogo selvagem
d. Duhring disease
e. Hansen disease.

Q513. The following is associated with atopic dermatitis except?


a. Ichthyosis hystrix
b. Central facial pallor
c. Pityriasis alba
d. Nipple eczema
e. Hyperlinear palms.

Q514. Which of the following disease antibodies against type VII collagen is seen?
a. Epidermolysis bullosa simplex
b. Pemphigus erythematosus
c. Cicatritial pemphigoid
d. Epidermolysis bullosa acquisita
e. Bullous pemphigoid.

Q515. A pregnant woman in the 2nd trimester of pregnancy presents to the


dermatology clinic with urticarial plaques and papules around her umbilicus, chest
and extremities. Tense vesicles are present within a few of the
erythematous plaques. What is the most likely diagnosis?
88
a. Pemphigus vulgaris
b. Dermatitis herpetiformis
c. Pruritis urticarial papule and plaque of pregnancy
d. Pemphigoid of pregnancy
e. Prurigo of pregnancy.

Q516. Which of the following is considered the most safe in pregnancy?


a. Benzoyl peroxide
b. Epinephrine
c. Methotrexate
d. Azelaic Acid
e. Isotretinoin.

Q517. A patient presents with a likely fixed drug eruption. Her medications include
glyburide, lisinopril, hydrochlorothiazide, and aspirin, as well as an over-the-counter
laxative. Which is the most likely culprit?
a. Glyburide
b. Lisinopril
c. Hydrochlorthiazide
d. Aspirin
e. Laxative

Q518. Which one of the following antifungals requires an acidic environment for its
absorption?
a. Amphotericin B
b. Fluconazole
c. Griseofulvin
d. Terbinafine
e. Itraconazole

Q519. A 10-year-old child with a seizure disorder develops a morbiliform eruption and
elevated LFT's two weeks after starting Dilantin therapy. What action must be taken
next?
a. Continue Dilantin and treat rash with topical corticosteroids
b. Discontinue Dilantin and begin Phenobarbital
c. Discontinue Dilantin and begin carbamazepine
d. Restart Dilantin once the rash resolves
e. Discontinue Dilantin and begin valproic acid

Q520. Which of the following may cause an acneiform eruption?


a. Griseofulvin
b. Valproic acid
c. ACTH
89
d. Finasteride
e. Methotrexate

Q521. If a patient develops a Dilantin (phenytoin) hypersensitivity reaction. What is


the alternative therapy?
a. Carbamazapine
b. Phenobarbital
c. Valproic acid
d. All of the answers are correct
e. None of these answers are correct.

*Q522. Which of the following systemic corticosteroids has the highest glucocorticoid
activity?
a. Betamethasone
b. Methylprednisolone
c. Triamcinolone
d. Cortisone
e. Hydrocortisone

Q523. A 35-year-old patient presents with dark streaking in the nails, diffuse and oral
hyperpigmented macules and trichomegaly (long eyelashes). Which of the following
medication he is on?
a. Zidovudine
b. Hydroxyurea
c. Cyclophosphamide
d. Bleomycin
e. Infliximab.

Q524. Which of the following retinoids is most lipophilic?


a. Etretinate
b. Acitretin
c. Tretinoin
d. Bexarotene
e. Isotretinoin

*Q525. Which drug has been associated with cholestatic hepatitis?


a. Clindamycin
b. Erythromycin
c. Rifampin
d. Doxycycline
e. Ampicillin

Q526. What antihistamine can cause gynecomastia, impotence, and loss of libido?
a. Cyproheptadine
90
b. Chlorpheniramine
c. Cimetidine
d. Doxepin
e. Fexofenadine

Q527. Tertracycline should not given together with isotretinoin, because of which of
the following?
a. Vitamin A exaggerated effect
b. Pseudotumor cerebri
c. Hepatotoxicity
d. Elevated triglycerides
e. Arthralgia

*Q528. How long after isotretinoin therapy can a woman safely begin pregnant?
a. Immediately
b. Two weeks
c. One month
d. One year
e. 2 years

Q529. What is the best medication to lower isotretinoin induced


hypertriglyceridemia?
a. Simvastatin
b. Gemfibrozil
c. Niacin
d. Cholestyramine
e. All are equally effective

*Q530. Which of the following is a low sedation metabolite of hydroxyzine?


a. Fexofenadine
b. Cetirizine
c. Loratadine
d. Cyproheptadine
e. Ranitidine

*Q531. Imiquimod is FDA approved as the treatment in which of the following?


a. Actinic keratosis
b. Squamous cell carcinoma
c. Infiltrative basal cell carcinoma
d. Superficial spreading melanoma
e. Psoriasis vulgaris

*Q532. Which of the following statements is true regarding antifungal medications?


a. Griseofulvin is safe for patients with variegate porphyria
b. Ketoconazole has been associated with gynecomastia
c. Griseofulvin is a cytochrome P450 inhibitor
91
d. Terbinafine is fungistatic
e. Ketoconazole is fungicidal

Q533. Which of the following drugs have been implicated in drug-induced sub-acute
cutaneous lupus erythematosus?
a. Terbinafine
b. Verapamil
c. Pravastatin
d. All the above are correct
e. None of the above are correct

Q534. Which site of the body has the highest penetration ability when topical
medications applied?
a. Face
b. Eyelids
c. Scrotum
d. Mucous membrane
e. Upper arms and legs.

Q535. Which of the following drugs may cause acute generalized exanthematous
pustulosis (AGEP)?
a. Penicillin
b. Sulphonamide
c. Hydroxychloroquine
d. Terbinafine
e. All of these answers are correct.

Q536. What is the treatment of choice for methemoglobinemia?


a. Methylene blue
b. Aspirin
c. Iron
d. Hydration
e. Observation

Q537. Which of the following drugs binds iron and thereby significantly prevents
absorption?
a. Methotrexate
b. Azathioprine
c. Cyclosporine
d. Mycophenolate mofetil (cellcept)
e. Tetracycline

Q538. Onycholysis is a potential side effect of treatment with which therapy?


a. Dapsone
92
b. Ketoconazole
c. Chloroquine
d. Tetracycline
e. Retinoids

Q539. Which of an antibiotic is the treatment of choice in pregnant patients with


Rocky Mountain spotted fever?
a. Amoxicillin
b. Doxycycline
c. Erythromycin
d. Chloramphenicol
e. Trimethoprim

Q540. Which off the following drugs is known to induce lichen planus-like eruptions?
a. Betablockers
b. Frusemide
c. NSAIDs
d. Phenothiazine
e. Mercury

Q541. Which of the biologic agent is infused intravenously?


a. Alefacept
b. Etanercept
c. Efalizumab
d. Infliximab
e. None of these answers are correct

Q542. Which of the biologic agent is given intramuscularly?

Q543. Which of the biologic agent is given subcutaneously?

Q544. Which of the following supplements is most likely to decrease hemolysis caused
by dapsone?
a. Vitamin A
b. Vitamin B6
c. Folic acid
d. Vitamin D
e. Vitamin E

Q545. Which of the following is a side effect of Clofazamine (lamprene)?


a. Dark hyperpigmented streaks in the nails
b. Pink to brownish-black hyperpigmentation of the skin
c. Blue-gray hyperpigmentation over the anterior shins, palate, ears
d. Slate gray-purple hyperpigmentation in a photo-exposed distribution
e. Yellow discoloration of the skin, sclera

93
Q546. What is the half-life of isotretinoin?
a. 7 hours
b. 20 hours
c. 50 hours
d. 30 days
e. 120 day

Q547. Concomitant use of methotrexate and what other drug is contraindicated


because of the potential increased risk of pancytopenia?
a. Folic acid
b. NSAIDs
c. Tetracycline
d. Psoralen
e. Systemic retinoids
Q548. Which of the following drug can induce Raynaud's phenomenon?
a. Estrogen
b. Betablockers (propranolol
c. Bleomycin
d. Nicotine
e. All the answer are correct.

Q549. Penile erosions, are a reported side effect associated with which medication?
a. Trimethoprim-sulfamethoxazole
b. Acyclovir
c. Foscarnet
d. Azaithioprine
e. Bleomycin

Q550. The most common side effect of azathioprine is?


a. Bone marrow suppression
b. Neuropathy
c. Hepatotoxicity
d. Nephrotoxicity
e. Myopathy

Q551. Which of the following biologic agents is pregnancy category C?


a. Alefacept
b. Infliximab
c. Efalizumab
d. Etanercept
e. All of these answers are correct

Q552. What is the name of the sigh when extension of the blister by application of
perpendicular pressure?
a. Fitzpatrick sign
b. Nikolsky’s sign
94
c. Asboe-Hansen's sign
d. Hutchinson's sign
e. Gorlin's sign

Q553. What drug can potentiate bone marrow suppression when used concomitantly
with azathioprine?
a. NSAIDS
b. Salicylates
c. Sulfonamides
d. Phenytoin
e. Allopurinol

Q554. A patient taking azathioprine for bullous pemphigoid develops a


hypersensitivity syndrome characterized by fever and shock. Approximately how long
ago did the patient start this medication?
a. 14 days
b. 24 hours
c. 3-4 days
d. 6 weeks
e. 1 week

Q555. Which of the following systemic agents has been shown to be the most effective
in the treatment of toenail onychomycosis?
a. Ketoconazole
b. Griseofulvin
c. Itraconazole
d. Fluconazole
e. Terbinafine

Q556. Which of the following is a side effect of hydroquinone cream?


a. Telangiectasia
b. Photosensitivity
c. Ochronosis
d. Tachyphylaxis
e. Atrophy

Q557. A 52-year-old male presents to the dermatology clinic complaining of pain ,


erythema, and a purulent discharge from the surgical site after excision of a basal
cell carcinoma on his left shin for 5 days duration,. He was given Cephalexin for a total
of 10 days. Two days later, he complains of worsening pain, redness, and discharge.
Which of the following is the best antibiotic choice?
a. Doxycycline
b. Amoxicillin
c. Azithromycin
d. Polymyxin B
95
e. Ciprofloxacin

Q558. What is the mechanism utilized by the co-administration of probenicid to raise


blood levels of penicillins in patients with infections that require high blood levels?
a. Inhibition of cytochrome P-450 hepatic bio- transformation system
b. Competitive inhibition of b-lactam binding sites
c. Displacement of plasma proteins
d. Synergistic effect of probenicid with penicillins
e. Prolongs the half-life of penicillin by decreasing the renal secretion.

Q559. What is the most likely congenital defect associated with isotretinoin therapy?
a. 1 Atrial septal defect
b. 2 Ventral septal defect
c. 3 Cranial synostosis
d. 4 Spina bifida
e. 5 Phocomelia

Q560. A patient takes oral clindamycin for a deep soft tissue infection due to a gram-
positive organism. The mechanism of action of clindamycin is most similar to which of
the following antibiotics?
a. Erythromycin
b. Levofloxacin
c. Doxycycline
d. Penicillin
e. Rifampin

Q561. What is the most common side effect of treatment with interferon-alpha?
a. Nausea
b. Weight loss
c. Liver toxicity
d. Flu-like symptoms
e. Spastic diplegia

Q562. Which of the following side effect of terfinadine (an antihistamine), is no longer
available in the USA?
a. Life-threatening cardiac arrhythmias
b. Thyroid cancer
c. Seizures
d. Cyanosis
e. Agranulocytosis

Q563. Which anti-viral agent used most often in cases of acyclovir-resistent HIV?
a. Valacyclovir
b. Gancyclovir
96
c. Foscarnet
d. Famciclovir
e. Pencyclovir

Q564. Which of the new biologic therapies for psoriasis, requires weekly CD4 T-cell
count monitoring?
a. Efalizumab
b. Alefacept
c. Infliximab
d. Etanercept
e. Adalimumab

Q565. What is the most common side effect of thalidomide?


a. Distal motor neuropathy
b. Oral ulceration
c. Photosensitivity
d. Hypothyroidism
e. Sensory neuropathy

Q566. Which of the following is the most common adverse effect of thalidomide
therapy?
a. Diarrhea
b. Sedation
c. Skin discoloration
d. Photosensitivity
e. Hypertension

Q567. In addition to minocycline, which of the following drugs has been associated
with drug-induced lupus erythematosus-like syndrome?
a. Amiodarone
b. Itraconazole
c. Rifampin
d. Doxycycline
e. Hydralazine

*Q568. Which medication is most likely responsible for a yellowish changes of his
sclera and yellowish discoloration of his dorsal hands and feet?
a. Quinacrine
b. Minocycline
c. Hydroxychloroquine
d. Terbinafine
e. Prednisone

Q569. How terbinafine exerts its antifungal activity?


a. Inhibition of 14-a demethylase
b. Inhibition of squalene epoxidase
97
c. Inhibition of epoxide hydroxylase
d. Interference with cell respiratory processes
e. Direct binding to membrane sterols, increasing permeability

Q570. Which of the following is the highest potency topical corticosteroid?


a. Halobetasol propionate cream 0.05%
b. Triamcinolone acetonide ointment 0.1%
c. Fluocinolone acetonide cream 0.025%
d. Hydrocortisone valerate ointment 0.2%
e. Mometasone furoate ointment 0.1%

Q571. Which of the following medications would be safe for use in pregnancy?
a. Methotrexate
b. Finasteride
c. Etretinate
d. Azeleic acid
e. Tetracycline

Q572. Which medication can cause Painful periungual pyogenic granulomas ?


a. Ketoconazole
b. Indinavir
c. Doxycycline
d. Tazarotene
e. Valacyclovir

Q573. Which type of tetracycline is not phototoxic?


a. Minocycline
b. Doxycycline
c. Oxytetracycline
d. Tetracycline
e. Demeclocycline

Q574. The anti-CD 20 antibody rituximab, is FDA-approved for treatment in which of


the following diseases?
a. Mycosis fungoid
b. Metastatic melanoma
c. Psoriasis
d. Paraneoplastic pemphigus
e. Non-Hodgkins lymphoma

Q575. Which of the following are reported cutaneous side effects of zidovudine?
a. Trichomegaly (long eyelashes)
b. Diffuse and oral hyperpigmented macules
c. Hyperpigmented streaks in nails
d. Periungual pyogenic granulomas
e. All of the above
98
*Q576. What is the target antigen for rituximab?
a. CD4
b. CD7
c. CD8
d. CD20
e. CD22

Q577. What is the most common cause of fixed drug eruption?


a. Tetracycline
b. NSAIDs
c. Erythromycin
d. Amoxicillin
e. Paracetamol

Q578. Which medication can cause gray-green discoloration of the mid-portion of


permanent teeth?
a. Doxycycline
b. Tetracycline
c. Minocycline
d. Erythromycin
e. Clindamycin

Q579. Which medication can cause hemorrhagic cystitis?


a. Chlorambucil
b. Cyclophosphamide
c. Vinca alkaloids
d. Hydroxyurea
e. Azaithioprine

Q580. Which of the following medications is abdominal cramping and watery diarrhea
a common side effect that may limit its use?
a. Colchicine
b. Gold
c. Potassium iodide
d. Thalidomide
e. Chlorambucil

Q581. What is the recommended period for contraception after stopping of acitretin
therapy in the United States?
a. 1 month
b. 3 months
c. 1 year
d. 2 years
99
e. 3 years

*Q582. 56 year-old man presents with blue-gray discoloration on his face, ears, and
dorsal hands. What is the most likely offending agent?
a. Minocycline
b. Amiodarone
c. Chloroquine
d. Quinacrine
e. Clofazimine
Q583. Which of the following medications is most likely to interefere with the efficacy
of oral contraceptives?
a. Rifampicin
b. Tetracycline
c. Doxycycline
d. Trimethoprim-sulfamethoxazole
e. Amoxicillin

*Q584. A patient taking daily prednisone is advised to switch to alternate day dosing
to decrease the risk of which of the following?
a. Glaucoma
b. Aseptic bone necrosis
c. Cataracts
d. Adrenal crisis
e. Osteoporosis

Q585. Which of the following is an adverse effect of oral retinoid therapy?


a. Hemolytic anemia
b. Headache and mild depression
c. Vascular calcification
d. Hypertension
e. Hypertrichosis

Q586. Which of the following retinoid medications is a second-generation synthetic


retinoid?
a. Acitretin
b. Isotretinoin
c. Tretinoin
d. Adapalene
e. Bexarotene

Q587. Which of the following has been associated with a lichenoid drug eruption?
a. Sulfasalazine
b. Hydrochlorothiazide
c. Augmentin
d. Erythromycin
e. Nicotinamide
100
Q588. Thyroid function tests should be checked before and during therapy with which
of the following medications?
a. Colchicine
b. Gold
c. Potassium iodide
d. Thalidomide
e. Azathioprine

Q589. At standard dosages, which of the following is fungicidal?


a. Terbinafine
b. Fluconazole
c. Ketoconazole
d. Itraconazole
e. Griseofulvin

Q590. What anti-fungal is known to cause gynecomastia and impotence?


a. Griseofulvin
b. Itraconazole
c. Terbinafine
d. Ketoconazole
e. Fluconazole

Q591. Which of the following agents when used prophylactically has demonstrated
efficacy in reducing skin cancer in organ transplant patients?
a. Tacrolimus
b. Systemic retinoid
c. Imiquimod
d. Interleukin-2
e. Cyclosporine

Q592. Which of the following drugs has been known to cause penile erosions?
a. Gancyclovir
b. Cidofovir
c. Foscarnet
d. Penciclovir
e. Abacavir.

Q593. The agent of choice used to acutely lower methemoglobin levels in patients
taking dapsone?
a. Cimetidine
b. Oral methylene blue
c. Homocysteine
d. Vitamin E
e. Vitamins C.
101
Q594. An 87-year-old female with chronic lymphocytic leukemia develops
disseminated varicella-zoster infection. She hospitalized for treatment.
Rapid intravenous infusion of acyclovir has been associated with what complication?
a. Reversible obstructive nephropathy
b. Disseminated intravascular coagulation
c. Thrombocytopenia
d. Pulmonary fibrosis
e. Serum sickness.
Q595. Which medication has strong H1 and H2 antihistamine blockade?
a. Chlorpheniramine
b. Cetirizine
c. Cimetidine
d. Cyproheptadine
e. Doxepin.

Q596. Methemoglobinemia, is an adverse reaction to which of the following drug?


a. Dapsone
b. Methotrexate
c. Plaquenil
d. Azathioprine
e. Cyclophosphamide.

*Q597. A 59 year-old woman is diagnosed with acute myelogenous leukemia.


Induction chemotherapy started. A few days later, the patient develops tender
erythematous plaques on her face. What is the most likely culprit drug?
a. Methotrexate
b. Cytarabine
c. Fluorouracil
d. Cyclophosphamide
e. Hydroxyurea.

Q598. Which of the following statements is not true regarding the tetracycline
antibiotics?
a. Ingestion of zinc salts may impair absorption of tetracycline
b. Tetracycline is the most common cause of fixed drug eruption
c. Tetracycline is more phototoxic than demeclocycline and doxycycline
d. These antibiotics are effective against mycoplasma infections
e. Tetracyclines are contraindicated in children less than 9 years of age.

Q599. Which cutaneous side effect is a common complication of topical nitrogen


mustard therapy?
a. Bullous drug eruption
b. Telangiectasia
c. Hyperpigmentation
d. Allergic contact dermatitis
102
e. Fixed drug eruption.

Q600. Which of the following side effects has not been reported in association with
intravenous immunoglobulin?
a. Headache
b. Flushing
c. Hypotension
d. Stevens-Johnson syndrome
e. Anaphylaxis.
*Q601. What is the serious side effect has been associated with the use of valacyclovir
in AIDS patients?
a. Stevens-Johnson syndrome
b. Acute renal failure
c. Jarisch-Herxheimer reaction
d. Disseminated intravascular coagulation
e. Thrombotic thrombocytopenia purpura/hemolytic uremic syndrome

*Q602. Which type of tetracycline can be use in patients with renal failure?
a. Doxycycline
b. Minocycline
c. Tetracycline
d. Demeclocycline
e. All are correct

*Q603. Alternate-day administration of oral steroids can reduce all of the following
side effects except?
a. Growth impairment
b. HPA axis suppression (hypothalamic-pituitary-adrenal axis)
c. Cataracts
d. Peptic ulcer disease
e. Opportunistic infection

Q604. Which of the following are the side effect of cyclosporine?


a. Hyperkalemia
b. Hyperuricemia
c. Hypomagnesemia
d. Hypertrichosis
e. All are correct

*Q605. The following retinoids are excreted in the urine except?


a. Bexarotene
b. Etretinate
c. Isotretinoin
d. Tretinoin
e. Acitretin

103
Q606. Which of the following are the specific marker for drug-induced lupus?
a. 1 Anti-histone antibody
b. 2 ANA
c. 3 Anti-Ro antibody
d. 4 Anti-La antibody
e. 5 Anti-ds DNA antibody

Q607. Drug induced lupus include the following except?


a. Griseofulvin
b. Hydralazine
c. Procainamide
d. Isonaizid (INH)
e. Penicillamine.

Q608. Which of the following anti parasitic agent is highly flammable?


a. Lindane
b. Permethrin
c. Precipitated sulfur
d. Thiabendazole
e. Malathion

Q609. Which of the following drugs can cause gynecomastia, impotence, and loss of
libido as side effects?
a. Doxepin
b. Cyproheptadine
c. Promethazine
d. Fexofenadine
e. Cimetidine

Q610. Which of the following can increase the risk of pseudo-tumor cerebri in patients
taking isotretinoin?
a. Dehydration
b. Concomitant use of tetracycline
c. Concomitant use of TMP-SMX
d. Doses higher than 1.0 mg/kg/day
e. All are correct.

Q611. Which of the following cell type is increased by glucocorticoids therapy?


a. Neutrophils
b. Monocytes
c. T-cells
d. B-cells
e. Eosinophils

104
Q612. Which of the following medications is associated with xerosis (dryness) of the
skin?
a. Sulfonylureas
b. Beta blockers
c. Calcium channel blockers
d. Loop diuretics
e. Cholesterol lowering agents.

Q613. Which of the following steroid has the least mineralocorticoid activity?
a. Hydrocortisone
b. Cortisone
c. Prednisone
d. Methylprednisolone
e. Prednisolone

Q614. Over use of which medication may lead to exogenous ochronosis?


a. Topical steroid
b. Topical antibiotic
c. Topical retinoid
d. Imiquimod (Aldara)
e. Hydroquinone

*Q615. What is histological feature which can distinguishes between lichen planus and
lichenoid drug eruption?
a. Segmental hypergranulosis
b. Parakeratosis
c. Spongiosis
d. Saw tooth pattern
e. None is correct

Q616. Which of the following is a potentially irreversible ocular side effects of


antimalarial agents?
a. Corneal deposition - causing halos, blurred vision, photophobia
b. Neuromuscular eye toxicity
c. Retinopathy
d. Cataracts
e. Pterygium

*Q617. A 15-year-old boy presents with pigmented bands on several fingernails and
toenails for 4-months duration. The most like etiological agent is?
a. Peutz-Jeghers syndrome
b. Chloroquine therapy
c. Minocycline therapy
d. Melanocytic nevi
105
e. Acral lentiginous melanoma

Q618. Which of the following drug has mucocutaneous side effects such as, stomatitis,
chelitis, lichen planus-like eruptions, and pityriasis rosea-like eruptions?
a. Colchicine
b. Gold
c. Potassium iodide
d. Thalidomide
e. None of the above.
Q619. Which of the following antifungal is fungicidal?
a. Itraconazole (sporanox)
b. Terbinafine (Lamisil)
c. Fluconazole (diflucan)
d. Clotrimazole
e. Griseofulvin (fulcin)

Q620. Which of the following oral agents has been effective in the treatment of
Norwegian scabies?
a. Thiabendazole
b. Ivermectin
c. Mebendazole
d. Griseofulvin
e. Metronidazole

***Q621. What testing can be performed to asses for methotrexate induced liver
fibrosis, instead of of liver biopsy?
a. AST/ALT ratio
b. Alkaline phosphatase level
c. Procollagen 3 testing
d. Procollagen 1 testing
e. Procollagen 4 testing

***Q622. Which of the following therapeutic agent can cause neutrophilic eccrine
hidradenitis is a side effect?
a. Cytarabine
b. Interferon type I
c. Intravenous immune globulin
d. Granulocyte colony stimulating factor
e. Dapsone.

Q623. Which of the following drug can reduces the formation of methemoglobin?
a. Dapsone
b. Cimetidine
c. Azathioprine
d. Cyclosporine
e. Rifampicin.
106
Q624. Which of the following medications can lead to hematologic toxicity when
combined with methotrexate?
a. Trimethoprim
b. Sulfonamides
c. Dapsone
d. All of these answers are correct
e. None of these answers are correct

Q625. Imiquimod (Aldara) has proven beneficial in the treatment of the following
lesions except?
a. Molluscum contagiosum
b. Superficial basal cell carcinoma
c. Actinic keratosis
d. Squamous cell carcinoma
e. Genital wart

***Q626. The following are true regarding water-soluble retinoids except?


a. They are undetectable in the serum after 1 month of stopping therapy
b. They include isotretinoin
c. They include etretinate
d. They include bexarotene
e. They have very little lipid deposition

***Q627. Which of the following medications used in dermatology is known to


increase the risk for bone fractures?
a. Dapsone
b. Isotretinoin
c. Acitretin
d. Prednisone
e. Doxycycline

***Q628. Which member of the tetracycline family is most photosensitizing?


a. Demeclocycline
b. Doxycycline
c. Minocycline
d. Oxytetracycline
e. Tetracycline

***Q629. Which of the following drug is known to cause an SCLE-like eruption?


a. Griseofulvin,
b. Hydrochlorothiazide,
c. Penicillamine and piroxicam
d. Terbinafine
e. All are correct.

107
Q630. A 66-year-old female presents with palpable purplish lesions and small necrotic
ulcers of the extremities. Additional workup reveals renal and hepatic involvement,
and a diagnosis of Wegener's granulomatosisis made. Which of the following cytotoxic
agents is the treatment of choice for Wegener's granulomatosis?
a. Cyclophosphamide
b. Chlorambucil
c. Methotrexate
d. Bleomycin
e. Mycophenolate mofetil (cellcept).

Q631. What is the possible metabolic abnormality with long-term systemic steroid
therapy?
a. Hyperlipidemia
b. Hypoglycemia
c. Hyperkalemia
d. Hypercalcemia
e. Hypokalemic acidosis

Q632. What is the treatment of choice for Wegner's granulomatosis?


a. Chlorambucil
b. Systemic steroid
c. Cyclophosphamide
d. Methotrexate
e. Azathioprine

Q633. Which of the following organisms increased colonization in the skin with
treatment with isotretinoin?
a. Streptococcus pyogenes
b. Pseudomonas aeruginosa
c. Staphylococcus aureus
d. Pityrosporum orbiculare
e. Demodex folliculorum

Q634. Which of the following substances is not found in Castellani's paint?


a. Resorcinol
b. Acetone
c. boric acid
d. Phenol
108
e. Lactic acid

Q635. Which of the following drug has been shown to be use in chemoprevention of
squamous cell carcinoma in recessive dystrophic epidermolysis bullosa?
a. Isotretinoin
b. Cyclosporine
c. Mycophenolate mofetil
d. Methotrexate
e. Topical tar

***Q636. Which of the following antibiotics is the only drug that is bactericidal to
Mycobacterium leprae?
a. Rifampicin
b. Levofloxacin
c. Minocycline
d. Amikacin
e. Pentavalent antimony

***Q637. Which of the following drug used in cases of methotrexate toxicity ?


a. Folinic acid ( Leukovorin)
b. Folic acid
c. Oral methylene blue
d. 4. Cimetidine
e. 5.Vitamin E

Q638. Which of the following statements regarding retinoids is true?


a. Tretinoin and isotretinoin are second-generation synthetic retinoids
b. Etretinate is a first-generation retinoid
c. Second-generation retinoids are polyaromatic retinoids
d. Tazarotene is a fourth-generation selective retinoid
e. Bexarotene is a third-generation retinoid

Q639. What is the treatment of choice for erythema nodosum leprosum (ENL)?
a. Thalidomide
b. Clofazimine
c. Rifampicin
d. Rifampin and clofazimine
e. Isoniazid, rifampin and clofazimine.

***Q640. Which of the following medication is the safest to use during pregnancy?
a. Penicillin
109
b. Erythromycin estolate
c. Doxycycline
d. Estrogens
e. NSAIDs.

***Q641. Which of the following medication should be avoided in pregnant women


with acne?
a. Isotretinoin
b. Azelaic acid
c. Clindamycin lotion
d. Erythromycin gel
e. Azithromycin

Q642. Which of the following locations has the highest percutaneous absorption of
topical corticosteroids?
a. Scrotum
b. Axilla
c. Back
d. Arm
e. Palms

***Q643. Which of the following sites has the highest percutaneous absorption of
topical drugs?
a. Scrotum
b. Eyelid
c. Chest
d. Acral area
e. Mucous membrane

Q644. Which of the following antifungal agents is contraindicated in patients with a


history of porphyria?
a. Griseofulvin (fulcin)
b. Ketoconazole (nizoral)
c. Fluconazole (diflucan)
d. Itraconazole (sporanox)
e. Terbinafine (Lamisil)

Q645. The following are the side effects of PUVA therapy except:
a. Headache
b. Pruritus
c. Neutropenia
d. Burning sensation
e. Nausea

110
***Q646. Which of the following drug may increase levels of digoxin?
a. Amoxicillin
b. Ciprofloxacin
c. Erythromycin
d. Minocycline
e. Cephalexin

***Q647. What is the treatment of choice for scabies in pregnant women?


a. Malathion
b. 1% permethrin
c. Thiabendazole
d. Precipitated sulfur
e. Ivermectin

***Q648. Which of the laboratory abnormality most associated with cyclosporine?


a. Hypermagnesemia
b. Hypotension
c. Hyperkalemia
d. Hypouricemia
e. Increased LDH

Q649. A full term neonate presented to the dermatology clinic with small pustules
with no underlying erythema which were present at delivery. On examination,
the pustules were easily removed with clearing of the vernix and a collarette appears.
A gram stain is done showing predominately neutrophils without bacteria. What is the
most likely diagnosis?
a. Miliaria
b. Erythema toxicum neonatorum
c. Transient neonatal pustular melanosis
d. Congenital candidiasis
e. Urticaria pigmentosa

***Q650. Which of the following is a potential complication of subcutaneous fat


necrosis of the newborn?
a. Infection
b. Hypercalcemia
c. Acute renal failure
d. Hepatitis
e. Elevated uric acid levels

Q651. What is the most likely diagnosis in a 16-year-old patient who developed
generalized papulosquamous skin eruption on the trunk for 2 weeks after onset of
sore throat due to strep infection?
a. Guttate Psoriasis
b. Pityriasis rosea
c. Cutaneous T-cell lymphoma
111
d. Contact dermatitis
e. Lichen planus

Q652. What is the best therapeutic option for eczema herpeticum ?


a. Oral cephalexin
b. Topical immune modulator
c. Oral acyclovir
d. Topical mupirocin
e. Oral prednisolone

Q653. A full term newborn develops erythematous, indurated plaques on the upper
back. Which of the following tests should be performed?
a. Calcium
b. Complete blood count
c. Blood culture
d. Thyroid function test
e. Alkaline phosphatase

***Q654. Which of the following is not true about Gianotti-Crosti?


a. It is associated with viral infections like enterovirus, EBV, and CMV
b. It typically affects children between the age of 3 months and 15 years
c. It is characterized by monomorphic symmetric flat topped papules in
acral areas
d. It may have associated fevers, lymphadenopathy, and diarrhea
e. Corticosteroids should be given to alleviate the pruritus of the lesion
s

Q655. What is the most likely etiology of Jacquet diaper dermatitis?


a. Candida
b. Trichophyton rubrum
c. Group A beta-hemolytic streptococcus
d. Herpes simplex virus, Type 2
e. Multifactorial

Q656. A newborn presents with a well-defined, shiny patch with complete alopecia
on the vertex of the scalp along the suture lines. Which of the following is the first
step in diagnosis?
a. MRI examination
b. Serum Calcium evaluation
c. Skin biopsy
d. Skull x-ray
e. Fungal culture
112
Q657. A child presents with high fever, strawberry tongue, lymphadenopathy, and a
polymorphous exanthem. Approximately how many of these patients will have
coronary artery aneurysms if left untreated?
a. Two percent
b. Five percent
c. Ten percent
d. Twenty-five percent
e. Sixty percent

Q658. A 10-year old girl presents with desquamation of the fingertips. Which
laboratory or clinical examination should be ordered?
a. Renal ultrasound
b. ASO titer
c. EEG
d. Eye examination
e. Chest x-ray

Q659. Which of the following is not a major criterion for Kawasaki disease?
a. Fever > 5 days
b. Palmo-plantar erythema > desquamation
c. Cardiac aneurysm
d. Strawberry tongue/ red lips
e. Cervical adenopathy
Q660. What is the most common tumor associated with nevus sebaceous?
a. Trichoblastoma
b. Squamous cell carcinoma
c. Sebaceous carcinoma
d. Basal cell carcinoma
e. Melanoma

Q661. What deficiency is responsible for acrodermatitis enteropathica?


a. Thiamine
b. Niacin
c. Zinc
d. Vitamin C
e. Vitamin D

Q662. Which of the following is the most common long-term sequelae from congenital
rubella syndrome?
a. Macrocephaly
113
b. Saber shins
c. Deafness
d. Nystagmus
e. Microcephaly

***Q663. In a child with acrodermatitis enteropathica but zinc level either normal or
near normal. What is the other test could be a valuable as an adjunctive?
a. Magnesium
b. Uric acid
c. Calcium
d. Alkaline phosphatase
e. Iron.

Q664. Which of the following condition leads to elevated serum CPK.


a. Dermatomyositis
b. Lupus erythematosus
c. Psoriasis
d. Atopic dermatitis
e. Lichen planus.

Q665. An 8 year-old boy presents with pink, flushed cheeks and a low-grade fever. A
week later, he developed lacy eruption appeared on the trunk and extrimities. What
is the most likely etiology?
a. Human papilloma virus
b. Pox virus
c. Group A streptococcus
d. Herpes simplex virus
e. Parvovirus.

***Q666. Which of the following condition may be associated with aplasia cutis
congenita?
a. Tuberous sclerosis
b. Neurofibromatosis
c. Albinism
d. Epidermolysis bullosa dystrophica
e. Perinatal trauma.

Q667. A healthy, full-term infant develops a pustular, erythematous eruption on her


face and trunk on the third day of life. A smear taken from one of these pustules, will
show which of the following?
a. Gram-positive bacteria
b. Predominantly neutrophils
c. Multi-nucleated giant cells
d. Predominantly eosinophils
e. Hyphae.
114
Q668. Which of the following disorders presents with rhinorrhea, snuffles, rhagades,
condylomata lata, and mucous patches?
a. Rubella
b. Toxoplasmosis
c. Neonatal herpes
d. Syphilis
e. Human papillomavirus infection

Q669. Which of the topical dressing should be avoided in a patient with cutaneous
mastocytosis?
a. Mupirocin ointment
b. Polymyxin B ointment
c. Petrolatum gel
d. Neomycin ointment
e. Silver sulfadiazine

Q670. Which of followings syndromes characterized by Seborrheic-like or exfoliative


dermatitis?
a. Leiner's disease
b. Hyper-immunoglobulinem E syndrome
c. Wiskott-Aldrich syndrome
d. Chronic granulomatous disease
e. X-linked ichthyosis

. Q671. Which of the following disease can present as collodion baby?


a. Ichthyosis vulgaris
b. X-linked ichthyosis
c. Lamellar ichthyosis
d. Bullous congenital ichthyosiform erythroderma
e. Sjogren-Larsson syndrome

Q672. What is the usual treatment for subcutaneous fat necrosis of the newborn?
a. Observation
b. Aspirin
c. Topical calcipitriol
d. Excision
e. Retinoids

Q673. A 10-year-old male presents with multiple, non-itchy, pink-red papules


distributed on, the face, extensor surface of the extremities, and buttocks. His mother
noted that he had suffered from low grade fevers over the past few days, but is
otherwise healthy. Which of the following is the most likely diagnosis?
a. Measles
b. Chicken pox
115
c. Rubella
d. Papular acrodermatitis of childhood
e. Dermatitis herpetiformis

Q674. A 2-year-old boy presented to the dermatology clinic having dark coarse
scales on the sides of the neck, trunk and extremities. On examination, the boy was
completely healthy with dark scales pasted on the entire trunk and the upper and
lower extrimities with sparing of the face, palm, soles and flexures (anticubital and
popliteal fossa). What is the most likely association?
a. Corneal opacities
b. Cryptorchidism
c. Ectropion
d. Prolonged maternal labor
e. Neurologic abnormality

Q675. Which of the following is the most common complication associated with
cutis marmorata telangectatica congenita?
a. Seizure disorder
b. Atrial septal defect
c. Systemic lupus erythematosus
d. Hypercalcemia
e. Limb hypertrophy or atrophy

Q676. Which of the following organism is associated with neonatal acne?


a. Staphylococcus
b. Propionibacterium
c. Candida
d. Malassezia
e. Streptococcus

Q677. What is the most common location of cutaneous lesions in neonatal lupus
erythematosus?
a. Cheeks
b. Nose
c. Scalp
d. Perioral
e. Periorbital

Q678. A 3-months old baby boy, presented to the dermatology clinic having an
infantile haemangioma on the tip of the nose, which may in the future will obstruct
breathing. Imaging reveals no underlying structural anomalies. Propranolol started.
What is the side effect do you need to monitor?
a. Hypertension
116
b. Hypercalcemia
c. Hypoglycemia
d. Tachycardia
e. Hypernatremia

Q679. An infant presents with red-purple, granulomatous nodules occurring in the


diaper area. The etiology is secondary to local irritation, maceration and Candida
albicans. What is the most likely diagnosis?
a. Granuloma gluteale infantum
b. Langerhans cell histiocytosis.
c. Seborrheic dermatitis
d. Atopic dermatitis
e. Flexural Psoriasis

Q680. In which of the following time, Koplik’s spots of measles is a typically appear?
a. Before the exanthema
b. At the same time as the exanthema
c. 1 week after the exanthema
d. 6 weeks after the exanthema
e. 8 weeks after the exanthema

Q681. Which of the following human papilloma virus is a cause of butcher's warts?
a. HPV7
b. HPV 2
c. HPV 3
d. HPV 5
e. HPV 13

Q682. Which of the following is a cause of tricomycosis axillaris?


a. Malassezia furfur
b. Candida albicans
c. Corynebacterium tenuis
d. Micrococcus sedentarius
e. Corynebacterium minutissimum

Q683. Which of the following disease is transmitted by Pediculus humanus?


a. Malaria
b. Epidemic typhus
c. Scrub typhus
d. Endemic typhus
e. Oriental sore.
Q684. A 4 year-old boy is diagnosed with Staphylococcal Scalded-Skin Syndrome.
Vesiculation in this disease is secondary to exfoliative toxins binding what
target protein?
a. Plectin
b. Bullous pemphigoid antigen2
117
c. Bullous Pemphigoid antigen 1
d. Desmoglein 3
e. Desmoglein 1

Q685. Which organism can pitted keratolysis?


a. Corynebacterium minutissimum
b. Corynebacterium tenuis
c. Micrococcus sedentarius
d. Staphylococcus aureus
e. 5 Streptococcus pyogens

Q686. Which type of porphyrins is responsible for the fluorescence under Wood's
lamp examination of patient with erythrasma?
a. Porphobilinogen
b. Protoporphyrin IX
c. Coproporphyrin III
d. Uroporphobilinogen
e. Coproporphyrinogen

Q687. What is the most likely diagnosis in A 62 year-old patient presents with tinnitus,
facial paralysis, and vesicles of the external ear?
a. Primary Herpes Simplex infection
b. Primary Varicella Infection
c. Herpes Zoster Infection
d. Cytomegalovirus infection
e. Coxsackie virus A16 infection

Q688. What is the treatment of choice of cat-scratch disease is:


a. Erythromycin
b. Ceftriaxone
c. Trimethoprim-sulfamethoxazole
d. Itraconazole
e. Mebendazole

Q689. What is the causative organism of erythema infectiosum?


a. Pox virus
b. Herpes virus
c. Coxsackie virus
d. Parvovirus B19
e. None of these answers are correct
Q690***. A 16-year-old female presents with a sore throat, painful cervical
lymphadenopathy, and an enlarged spleen. An empiric trial of amoxicillin leads to the
development of a diffuse morbilliform rash. What is the incubation period of the most
likely viral cause of this patient's symptoms?
a. Measles
118
b. Rubella
c. Gianotti-Crosti disease
d. Infectious mononucleosis
e. None of these are correct
Q691***. The following can cause of a false positive VDRL except:
a. Systemic Lupus Erythematosus (SLE)
b. Pregnancy
c. Malignant Melanoma
d. Lepromatous Leprosy
e. Malaria

Q692. Which type of lichen planus is most strongly associated with Hepatitis C
virus infection?
a. Mucosal ulcerative lichen planus
b. Hypertrophic lichen planus
c. Lichen planus pemphigoid
d. Lichen planus of the nails
e. Follicular lichen planus

Q693. An Indian patient presents to the emergency clinic with fever,


hepatosplenomegaly, and darkening of the skin. Which of the following is a cause
of visceral leishmaniasis?
a. L. donovani
b. L. major
c. L. Mexicana
d. L. aethiopica
e. L. tropicana

Q694. A 19-year-old sexually active male presents with a painful ulcer on the shaft
of his penis. A biopsy specimen stained with Giemsa reveals a "school of fish." Which
of the following is the causative organism?
a. Hemophilus ducreyi
b. Treponema pallidum
c. Calymmatobacterium granulomatis
d. Chlamydia trachomatis L1
e. Herpes simplex virus

Q695. Which of the following tests is the most sensitive serologic test in
primary syphilis?
a. FTA-ABS
b. VDRL
c. ELISA
d. RPR
e. MHA-TP
119
Q696. Which of the following is a cause of purpura fulminans?
a. S. aureus
b. P. aeruginosa
c. H. influenzae
d. E. coli
e. Group A streptococcus

Q697. What is the most common cause of perianal wart in 12 months old baby?
a. Sexual abuse
b. Contact with a wart-containing finger
c. Contact with a fomite
d. Aerosol transmission
e. Vertical transmission at the time of delivery

Q698. What is the other name of Orf virus infection?


a. Ecthyma
b. Ecthyma contagiosum
c. Ecthyma gangrenosum
d. Scarlet fever
e. Milker's nodule

Q699. A 52-year-old butcher presents with a tender lesion on his left hand. Diagnosed
as erysipeloid. What is the treatment of choice of this condition?
a. Penicillin
b. Azithromycin
c. Ciprofloxacin
d. Tetracycline
e. Chloramphenicol

Q700. Which of the following skin manifestations of M. tuberculosis infection is a


tuberculids?
a. Erythema induratum of Bazin
b. Lupus vulgaris
c. Miliary TB
d. Scrofuloderma
e. Tuberculous gumma
Q701. A young male patient presents with a painful ulcer with a ragged edge after a
new sexual encounter. What is the treatment of choice?
a. Penicillin
b. Doxycycline
c. Azithromycin
d. Ciprofloxacin
e. Cephalexin

Q702. What is the causative organism of granuloma Inguinale?


120
a. Chlamydia trachomatis types I, II & III
b. Hemophilis ducreyi
c. Calymmatobacterium granulomatis
d. Treponema pertenue
e. Treponema pallidum

Q703. What is the most common cause of erythema multiforme?


a. Herpes simplex virus
b. Mycoplasma pneumonia
c. Staphylococcus aureus
d. 4 Drugs
e. 5 Cytomegalovirus

Q704. Which of the following cytokines is most closely associated with The Jarisch-
Herxheimer reaction?
a. IFN-alpha
b. IL-12
c. TNF-alpha
d. IL-4
e. IL-10

Q705***. Which of the following is the treatment of choice in a pregnant woman


presented with erythema migrans?
a. Doxycycline
b. Erythromycin
c. Chloramphenicol
d. Clindamycin
e. Amoxicillin

Q706***. What is the number of CD4 cell count when disseminated lesion of
molluscum contagiosum are first seen in HIV/AIDS patients?
a. Below 100
b. 200
c. 400
d. 800
e. 1000

121
Q707. What is the organism most responsible for hemorrhagic bullae and
erythematous nodules with dusky gray centers in a bone marrow transplant patient
has has a fever, neutropenia?
a. Streptococcus pyogenes
b. Mycobacterium tuberculosis
c. Candida albicans
d. Pseudomonas aeruginosa
e. Staphylococcus aureus

Q708. Which of the following is caused by a pox virus?


a. Molluscum contagiosum
b. Herpes simplex virus
c. Pediculosis capitis
d. Sarcoptes scabeii
e. Human immunodeficiency virus

Q709***. Which of the following tests will be helpful in determining a streptococcal


infection in a patient with pharyngitis and guttate psoriasis?
a. A complete blood count
b. FTA-ABS
c. U1-RNP
d. DNAse B
e. Serum calcium

Q710. Which of the following techniques can confirm the diagnosis of cutaneous
leishmaniasis?
a. Weil-Felix test
b. VDRL
c. Culture in Sabouraud’s glucose agar
d. Culture in Novy-MacNeal-Nicolle (NNN) medium
e. Culture in Michel's medium

Q711. A child presents with fever, cutaneous tenderness and erythema of flexural and
periorificial areas. Within 24 hours, the erythema progresses to flaccid blisters and
erosions. Nikolsky's sign is positive. What is First line therapy for SSSS?
a. IV penicillin
b. Aspirin
c. IVIG
d. Tetracycline
e. Cephalexin

122
Q712. Which of the following forms of syphilis is characterized by ham-colored
macules on the palms and soles, condyloma lata, and \"moth-eaten\" alopecia?
a. Primary syphilis
b. Secondary syphilis
c. Latent syphilis
d. Tertiary syphilis
e. Congenital syphilis

Q713. What is the clearance rate of untreated common warts in children at 2 years
after diagnosis?
a. 25-30%
b. 50-55%
c. 70-75%
d. 1%
e. 5-10%

Q714. Which of the following virus is a cause of roseola infantum?


a. Human herpes virus 6
b. Parvovirus B19
c. Epstein-Barr virus
d. Pox virus
e. Coxsackie virus A16

Q715***. Hutchinsons teeth are a feature of which stage of syphilis?


a. Early congenital syphilis
b. Late congenital syphilis
c. Primary syphilis
d. Secondary syphilis
e. Tertiary syphilis

Q716***. Which of the following statements describes the pathogenesis of


scrofuloderma?
a. Hematogenous spread of M. tuberculosis Autoinoculation of M. tuber
culosis from advanced pulmonary tuberculosis
b. Contiguous spread from an underlying focus of tuberculous lymphad
enitis
c. Primary incoculation of M. tuberculosis in a sensitized host
d. An cutaneous manifestation of military tuberculosis

123
e. direct extension of tuberculosis into the skin from underlying
structures such as lymph nodes, bone

Q717***. Scrofuloderma, is most commonly associated with an underlying infection


of what organ system?
a. Lymph nodes
b. Spine
c. Lungs
d. Gastrointestinal tract
e. ovaries

Q718. Which virus is most commonly associated with oral hairy leukoplakia?
a. Pox virus
b. Herpes virus
c. Epstein-Barr virus
d. Human papilloma virus
e. Parvovirus

Q719. Measles, is a caused by which of the following viruses?


a. Paramyxovirus
b. Herpes virus
c. Human papilloma virus
d. Varicella virus
e. Adenovirus

Q720. A 45-year old female, admitted to the hospital because of symmetric, large
ecchymotic patches on the extremities, ears, and nose. Laboratory testing reveals the
patient has disseminated intravascular coagulation. Which of the following is the
most common underlying infection associated with this condition?
a. Group A streptococcus
b. Meningococcus
c. Staphylococcus
d. Pseudomonas aeroginosa
e. Mycobacterium leprae

Q721***. Where do you see Gamma-Favre bodies?


a. Granuloma inguinale
b. Lymphogranuloma venereum (LGV)
c. Syphilis
d. Chancroid
e. Herpes virus

Q722. Which of the following is the best choice for treatment of herpes zoster?
124
a. Gancyclovir
b. Foscarnet
c. Valacyclovir
d. Gabapentin
e. Cidofovir

Q723. A 35-years-old patient with primary syphilis, treated with Benzathine penicillin
G. Soon after treatment, he develops fever, headache, myalgias, and elevated white
blood cell count, consistent with the Jarisch-Herxheimer Reaction. This reaction is
primarily mediated through which the following inflammatory cytokine?
a. TNF-alpha
b. IFN-gamma
c. IL-6
d. IL-8
e. IL-10

Q724. What is the most common trigger of recurrent oro-labial herpes simplex virus
infection?
a. UVB radiation
b. UVA radiation
c. Visible light radiation
d. Emotional stress
e. Drugs

Q725. What is the name of the nerve involved in the formation of Hutchinson's sign
(vesicles at the nasal tip) in herpes zoester?
a. Ophthalmic nerve
b. Maxillary nerve
c. Nasociliary branch
d. Infratrochlear nerve
e. Labial nerve

Q726. Which HLA type is most commonly associated with herpes-associated erythema
multiforme?
a. HLA B27
b. HLA DR2
c. HLA DR3
d. HLA B15
e. HLA B8

125
Q727. Which type of leprosy in a patient presented to the dermatology clinic with claw
hands, hypoesthesia of the distal extremities, madarosis and leonine face?
a. Lepromatous leprosy
b. Tuberculoid leprosy
c. Borderline leprosy
d. Erythema nodosum leprosum
e. Lucio phenomenon

Q728. What laboratory finding is commonly seen in children with measles virus
infection?
a. Lymphopenia
b. Thrombocytopenia
c. Polycythemia
d. Anemia
e. Lymphocytosis

Q729. What is the treatment of choice in a patient presented with a 2-days history of
fever and a well demarcated erythematous plaque on the right cheek?
a. Prednisone
b. Intravenous penicillin
c. Cephalexin
d. Fluconazole
e. Potent topical corticosteroid

Q730. Which of the following organisms is most likely colonizing a patient with atopic
dermatitis presented with multiple infected lichenoid plaques?
a. Candida albicans
b. Staphylococcus epidermidis
c. Staphylococcus aureus
d. Propionibacterium avidum
e. E. choli.

Q731. What is the annual risk of developing herpes zoster in an HIV infected patient?
a. 3% per month
b. 10% per month
c. 3% per year
d. 10% per year
e. 25% per year

126
Q732. What is the treatment of choice for acyclovir-resistant herpes simplex virus
infection?
a. Valacyclovir
b. Famciclovir
c. Indinivir
d. Saquinivir
e. Foscarnet

Q733. Which of the following virus is a cause of Milker’s nodule?


a. Human herpesvirus-8 (HHV-8)
b. Paravaccinia virus
c. Human papilloma virus
d. Orf virus
e. Human herpesvirus-6 (HHV-6)

Q734. What is the most likely diagnosis in a female patient presents with multiple
painful vaginal ulcers and tender left-sided inguinal lymphadenopathy. Gram stain
reveals clusters of cocco-bacilli in a school of fish pattern?
a. Primary syphilis
b. Lymphogranuloma venereum
c. Granuloma inguinale
d. Chancroid
e. Secondary syphilis

Q735. Which HPV (human papilloma virus) type is the most commonly cause of
condyloma accuminatum or genital wart?
a. HPV 1 & 3
b. HPV 4 & 7
c. HPV 6 & 11
d. HPV 31 & 33
e. HPV 48 & 67

Q736. What is the most common single nerve involved with herpes zoster?
a. Facial nerve
b. Trigeminal nerve
c. Spinal nerve C7
d. Spinal nerve T4
e. Spinal nerve T10

Q737. What is the characteristic cell type found on histopathology of A 27 year-old


HIV positive presents with pruritic eruption eosinophilic pustular folliculitis?
a. Neutrophil
b. Plasma Cell
c. Basophils
d. Eosinophils
e. Lymphocyte
127
Q738***. Which of the following infections is the most likely cause of an extensive
undermined, non-healing ulceration on the right lower leg in a patient returns from
Africa?
a. Mycobacterium marinum
b. Mycobacterium kansasii
c. Mycobacterium chelonei
d. Mycobacterium ulcerans
e. Leishmania donovani

Q739. What is the most likely time between the appearance of the genital lesion and
diffuse papulosquamous eruption appear on the trunk and genital area resembling
pityriasis rosea, in a patient with a history of a healed genital ulcer?
a. 1 - 2 weeks
b. 3 - 12 weeks
c. 3 - 5 months
d. 6 -12 months
e. 1 - 2 years

Q740. What is the first-line recommended therapy for orf?


a. Penicillin
b. Erythromycin
c. Doxycycline
d. Bactrim
e. No therapy is needed since the condition is self-resolving

Q741***. What is the most likely diagnosis in a homeless man presented with a
tender suppurative nodule on the mandible. Sulfur granules were present on
microscopy?
a. Actinomycetoma
b. Actinomycosis
c. Anthrax
d. Acne Conglobata
e. Aspergillosis

Q742***. What is the most common subtype of molluscum contagiosum virus


infection seen in HIV patients?
a. MCV-1
b. MCV-2
c. MCV-3
d. MCV-4
e. MCV-5

128
Q743***. What is the most common causative agent of impetigo in a 5-year old child?
a. Staphylococcus aureus
b. Group A B-hemolytic streptococci
c. Pseudomonas aeroginosa
d. Candida albicans
e. H. influenza
.

Q744. What is the most likely diagnosis in a patient presented to the Dermatology
Clinic with a non-tender penile ulcer which has An indurated border and associated
with non-tender bilateral inguinal lymphadenopathy?
a. Chancroid
b. Herpes simplex
c. Lymphogranuloma venereum
d. HIV
e. Primary syphilis

Q745. What anatomic sites are favored for molluscum contagiosum lesion in patients
with HIV/AIDS patient?
a. Face and genitalia
b. Head and neck
c. Abdomen and back
d. Upper and lower extremities
e. Chest and shoulders

Q746. Which of the following diseases is caused by Coxsackie virus A16?


a. ORF infection
b. Viral warts
c. Erythema infectiosum
d. Hand-Foot-and-Mouth disease
e. Molluscum contagiosum.

Q747. Which of the following is not a DNA virus?


a. Herpes virus
b. Pox virus
c. Parvovirus
d. HIV virus
e. Adenovirus

Q748. What is the most common causative agent for ecthyma in an immune-
competent patient?
a. Candida albicans
b. Group A Beta hemolytic streptococcus
c. Pseudomonas aeruginosa
129
d. Klebsiella
e. 5.Proteus

Q749. Which of the following organisms is spread by non-sexual person-to-person


contact?
a. Treponema carateum
b. Chlamydia trachomatis types I, II & III
c. Hemophilis ducreyi
d. Calymmatobacterium granulomatis
e. Treponemal pallidum
Q750. A known case of chronic Hepatitis C virus infection presents with hyperkeratotic
plaques on both feet (necrolytic acral erythema). What type of mineral has been
shown to improve the appearance and symptoms of this cutaneous finding?
a. Zinc
b. Biotin
c. Magnesium
d. Calcium
e. Chromium

Q751. What is the most likely causative organism in a 45-year-old poorly oral hygiene
patient presented with suppurative nodules and sinus tracts over the mandible?
a. Nocardia brasilensis
b. Streptococcus somaliensis
c. Actinomyces israelii
d. Staphylococcus aureus
e. Bartonella Henselae

Q752. What is the most common extra-cutaneous complications of varicella zoster


virus?
a. Lympho-reticular system
b. Musculoskeletal system
c. Cardiovascular system
d. Central nervous system
e. Genitourinary system

Q753. What is the name of the syndrome defined by gonorrhea, peri-hepatitis and
liver adhesions?
a. KID Syndrome
b. Turner Syndrome
c. Fitz-Hugh-Curtis Syndrome
d. Klinfelter syndrome
e. Clutton's Syndrome

Q754.Which of the following is true regarding lymphogranuloma venereum?


a. The primary stage presents as a painful herpetiformis ulcer
b. Proctocolitis can be seen in the late stages of this disease
130
c. It is caused by Chlamydia Psittaci
d. Buboes should be incised and drained
e. Transmission does not occur via skin-to-skin contact

Q755***. Which of the following disease is transmitted by pediculus corporis (human


body louse)?
a. Epidemic typhus
b. Trench fever
c. Relapsing fever
d. All are correct
e. None are correct

Q756. Which culture medium is used for growth of mycobacteria?


a. Chocolate agar in 10% CO2
b. Lowenstein-Jensen
c. New York City medium
d. Sheep blood agar
e. Agar supplemented with heme and nicotinamide

Q757. Which of the following antiviral agents, high doses has been associated with
thrombotic thrombocytopenic purpura in immunosuppressed patients?
a. Valacyclovir
b. Acyclovir
c. Foscarnet
d. Famciclovir
e. Cidofovir

Q758***How long is the life cycle for the mite sarcoptes scabiei that causes scabies?
a. One day
b. One Week
c. One month
d. Sex months
e. One year

Q759. The following is true regarding leishmaniasis except?


a. Arthropod vector is the female sand fly
b. Mazzottis test is diagnostic
c. Pentavalent antimony used for both cutaneous and visceral disease
d. Cutaneous disease is the most common form of leishmaniasis
e. Espundia is seen mucocutaneous disease

131
Q760. What is the most likely diagnosis in a female patient presents with a large,
vegetating ulcer involving her left labia majora and groin for over 1 year. A Giemsa
stained preparation reveals bipolar, safety pin-shaped intra-cytoplasmic inclusions?
a. Lymphogranuloma venereum
b. Granuloma Inguinale
c. Chancroid
d. Primary herpes simplex
e. Gonorrhea

Q761. An 18 year-old girl who was hospitalized last month after a serious car
accident is noted to have white transverse grooves on her fingernails and toenails.
What is the most likely diagnosis?
a. Twenty-nail dystrophy
b. Beaus lines
c. Half-and-half nails
d. Mees Lines
e. Terrys nails

Q762. Which of the following disease can lead to triangular lunulae of the nails?
a. Dyskeratosis congenita
b. Neurofibromatosis
c. Darier disease
d. Nail-patella syndrome
e. Tuberous sclerosis

Q763. Which of the following diseases can cause blue discoloration of the lunulae?
a. Yellow nail syndrome
b. Wilson's disease
c. Hemochromatosis
d. Hypertrophic pulmonary osteoarthropathy
e. neurofibromatosis

Q764. Which of the following conditions can cause diffuse severe sudden hair loss
developing three months after hospitalization for septicemia?
a. Anagen effluvium
b. Catagen effluvium
c. Telogen effluvium
d. Loose anagen syndrome
e. Uncombable hair syndrome

132
Q765. Which of the following organism is the most common cause of distal subungual
onychomycosis?
a. Trichophyton mentagrophytes
b. Epidermophyton floccosum
c. Trichophyton schoenleinii
d. Trichophyton rubrum
e. Trichophyton megninii

Q766. Which of the following organism can cause proximal subungual


onychomycosis?
a. Trichophyton mentagrophytes
b. Epidermophyton floccosum
c. Trichophyton rubrum
d. All of the above
e. None of the above

Q767. Which of the following organism is the most common cause of white superficial
onychomycosis?
a. Trichophyton mentagrophytes
b. Epidermophyton floccosum
c. Trichophyton schoenleinii
d. Trichophyton rubrum
e. Microsporum canis.

Q768. What is Mee’s lines?


a. Brownish macules beneath the nail plate
b. Local or diffuse hyperkeratotic tissue that develops on the lateral or
proximal nail folds
c. Transverse white lines that affect all nails, grow out with nail growth
d. Vertical black lines on a single or multiple nails
e. Double white transverse lines from abnormal vascular bed

Q769. Which of the following is a relative contraindication to beginning etanercept


(Enbrel) therapy?
a. Family history of retinitis pigmentosa
b. History of chronic renal failure
c. History of congestive heart failure (stage III)
d. History of positive PPD test and treatment with isoniazid(
e. History of unexplained anemia

Q770. The following medications are the most likely to cause acute
generalized exanthematous pustulosis except?
a. Penicillin
b. Sulphonamide
c. Quinolones
d. Hydroxycholoroquine
133
e. Griseofulvin

Q771.The following laboratory tests are important before beginning intravenous


immunoglobulin (IVIG) therapy for pemphigus vulgaris except?
a. Complete blood cell count with differential
b. Liver function test
c. Renal function test
d. Serum IgA level
e. Urinalysis

Q772. The following drugs may cause bullous pemphigoid except?


a. Etanercept (Enbrel)
b. Sulfasalazine
c. Itraconazole
d. Frusemide
e. Penicillin

Q773. Which of the following skin diseases, circulating antibodies to type VII collagen are
reported in most of the cases?
a. Bullous pemphigoid
b. Chronic bullous disease of childhood
c. Dermatitis herpetiformis
d. Bullous lupus erythematosus
e. Subacute cutaneous lupus erythematosus

Q774. A 35-year-old woman has the onset of proximal muscle weakness and
a heliotrope rash. Serum creatine kinase activity is greater than 3000 U/L. The following is
the most common cancer described in cancer-associated dermatomyositis except?
a. Breast cancer
b. Lung cancer
c. Ovarian cancer
d. cancer of colon
e. Skin cancer

Q775. A 33-year-old man is being evaluated because of multiple skin tags and flesh-
colored papules found on the face, ears, neck and upper trunk. Histopathological
examination shows fibrofolliculomas (benign tumour of the hair follicle). Which of the
following disorders the patient has the greatest risk?
a. Atherosclerosis
134
b. Cushing disease
c. Lymphoma
d. Kidney cancer
e. Testicular carcinoma

Q776. What does not mean by ecthyma?


a. Crusted impetigo
b. Deep, ulcerative impetigo
c. Circinate impetigo
d. Bullous impetigo
e. None of the above

Q777. The following can be used in the treated of viral warts except?
a. Laser
b. Cryotherapy
c. Electrocautery
d. Intralesional steroids.
e. surgery

Q778. What does it mean by vulgaris?


a. Serious.
b. Common
c. Easily treated
d. All of the above
e. None of the above

Q779. Which of the following the drug eruptions may be look like?
a. An erythematous rash
b. Morbilliform rash
c. Lichenoid rash
d. Pustular rash
e. All of the above.

Q780. The following are features of pityriasis rubra pilaris except?


a. Scaly scalp
b. Nail involvement
c. Psoriasiform patches
d. Herald patch
e. Follicular hyperkeratosis.

Q781. The following body sites are involved in scabies in adults patient except?
a. Wrist
b. upper back

135
c. Genitalia
d. Finger webs
e. Axilla.

Q782. Which of the following skin disorders may be treated by electrocautary?


a. Verruca vulgaris
b. Psoriasis vulgaris
c. Acne vulgaris
d. Pemphigus vulgaris
e. None of the above

Q783. Which of the following skin diseases are contagious?


a. Atopic dermatitis
b. Acne vulgaris
c. Erythema multiforme
d. Lepromatous leprosy
e. Acute urticaria

Q784. Which of the followings are the characteristic lesions in acne patient?
a. Comedones, vesicles and cysts
b. Burrows, papules and nodules
c. Comedones, papules and nodules
d. Comedones, bullae and pustules
e. Papules, pustules and vesicles

Q785. How long will stay, the urticarial wheals in food induced acute urticaria?
a. Few hours
b. Few days
c. Few weeks
d. Few months
e. None of the above

Q786. In discoid lupus erythematosus. Which of the following is true?


a. It is more common in males than females
b. Clinically it appears as erythematous nodules on the trunk
c. The skin lesion is covered by loose scales
d. It may causes diffuse non-scarring alopecia of the scalp hair
e. It is treated by potent topical steroid and antimalarial

Q787. In psoriasis. Which of the following is true?


a. It is a skin disease caused by staphylococcus or streptococcus bacteria
b. It is a congenital disease which usually appear at birth

136
c. It can be induced or exacerbated by drugs such as beta-blockers
d. it is a curable disease after a few months of treatment by systemic
steroid
e. It is highly infectious.

Q788. In allergic contact dermatitis. Which of the following is false?


a. It is a form of contact dermatitis and less common than irritant contact dermatitis
b. It is due to type IV hypersensitivity reaction
c. It affects any individual
d. It can be caused by nickel and rubber
e. It is an allergic response caused by contact with a substance

Q789. In pityriasis rosea. What is the primary lesion?


a. Vesicle and bullae
b. Papule covered by silvery scale
c. Wheals
d. Herald patch
e. Comedones

Q790. In lichen planus. Which of the following is false?


a. It is highly pruritic
b. It is highly contagious
c. It may cause cicatritial alopecia of the scalp hair
d. The primary lesion is a poly-angular papule
e. Nail involvement can occur

Q791. A 25-years-old male visited a town and stayed in a hotel, 4 weeks later, he
started to have papules on the genitals and between the hand fingers. Lesions are itchy and more
at night. What is the possible diagnosis?
a. Eczema
b. Urticaria
c. Scabies
d. Papular urticaria
e. Phthirus pubis

Q792. A seven-years- old baby presents with a whitish discoloration of the tongue and oral mucosa
after a prolonged course of systemic antibiotics. What is your diagnosis?
a. Oral candidiasis
b. Herpes simplex infection
c. Lichen planus
d. Chicken pox
e. folliculitis
137
Q793. A 24 year-old female, presented to the Dermatology outpatient clinic by itching, erythema
and vesiculation at the outer aspect of the upper arm. The lesion occurred exactly on the site of a
henna tattoo that she did two weeks before. What is your diagnosis?
a. Allergic contact dermatitis
b. Irritant contact dermatitis
c. Eczema herpeticum
d. Impetigo contagiosum
e. Herpes zoster

Q794. Which of the following diseases may be associated with multiple


dermatofibroma?
a. Bullous pemphigoid
b. Incontinentia pigmenti
c. Systemic Lupus erythematosus
d. Histiocytosis
e. Neurofibromatosis

Q795. A 6-month old child presents with a symmetrical eczematous eruption on the
cheeks, elbows and anterior aspects on the knees. The rash responds to a mild
topical steroid cream but flares whenever the cream is stopped. What is the most
likely cause of the rash?
a. Seborrhoeic dermatitis
b. Contact dermatitis to steroid cream
c. Atopic eczema
d. Food intolerance
e. Acrodermatitis enteropathica.

Q796. What is the most likely diagnosis in an eight-year-old boy of Indian descent
presents to your clinic with ill-defned hypopigmented patches on his cheeks. He
has a history of moderate atopic eczema controlled with 1% hydrocortisone
ointment and a simple emollient?
a. Melasma
b. Pityriasis alba
c. Steroid induced hypopigmentation
d. Vitiligo
e. Lepromatous leprosy.

Q797. An 88-year-old presented to the dermatology clinic with a moderately itchy


rash on the lower legs. On examination, an eczematous rash with extreme xerosis
and ‘river-bed’ cracking over the shins. The rash does not improve despite on using

138
copious amounts of greasy emollients. Which of these tests is likely to be the most
useful for this patient?
a. Patch testing to emollients
b. Full blood count with a blood film examination
c. Skin biopsy
d. Skin scrapings for mycology
e. Thyroid function test.

Q798. An 82-year-old woman, presented to the dermatology clinic with an


eczematous rash on the legs, which is associated with hemosiderin deposition and
varicosities. The rash was controlled with a combination of regular
emollients, support stockings and betamethasone/neomycin ointment (Betnovate-
N). Few months later, she developed , she developed a widespread eczematous
eruption covering much of her body. What is most likely diagnosis?
a. A disseminated eczema with allergic contact dermatitis to neomycin
b. A disseminated eczema with allergic contact dermatitis to
betamethasone
c. Secondary asteatotic eczema
d. Superimposed zoster infection with koebnerization
e. Development of nummular eczema.

Q799. An eight-year-old boy, known case of asthma, presents to the dermatology clinic with
skin rash on the soles of both feet for sex months duration. The rash has not
responded to a number of topical steroid preparations prescribed by his
general practitioner. On examination, the skin of the balls and toe-pads is dry, scaly and
fissured with a glazed appearance. What treatment is most appropriate?
a. Regular emollients only
b. Super-potent topical steroid
c. Wear shoes less and use leather shoes rather than trainers
d. A short course of oral terbinafine
e. Topical miconazole.

Q800. A 60-year-old male patient cardiac patient on aspirin, metoprolol, ramipril and
simvastatin (Zocor). After few months, he developed a skin rash on the extrimities. .
On examination, he has multiple small beefy red plaques with silvery scale, most
prominent on the extensor surfaces. What is the most likely diagnosis?
a. Patient caught psoriasis from another patient on the ward
b. Patient develop psoriasis precipitated or induced by beta-blocker
c. Psoriasiform drug reaction to aspirin
139
d. Lichenoid drug eruption
e. Acute urticaria due to aspirin

Q801. A 12-year-old boy attends to the dermatology clinic with widespread rash on
the trunk two weeks shortly after a severe sore throat treated by systemic
antibiotics. On examination, there are a widespread rash consisting of multiple,
small, deep red papules and plaques with some overlying white scale on the trunk.
What is the initial treatment is most appropriate?

a. Admission to hospital and treatment with a potent topical steroid


b. Start systemic steroid (1 mg/kg/day oral prednisolone)
c. Work up for cyclosporine
d. Start dithranol therapy
e. Consideration for UVB phototherapy (NB-UVB).

Q802. A pregnant woman in the third trimester become unwell developed extensive
patches of erythema and pustules. No history of drug intake or any contact with a
local irritant. The initial swabs from the pustules showed no organisms. On
examination, there are widespread pustules on an erythematous background, on the
trunk. What is the most likely diagnosis?
a. Generalized pustular psoriasis
b. Staphylococcal scalded skin syndrome
c. Toxic epidermal necrolysis
d. Eczema herpeticum
e. Pemphigoid of pregnancy.

Q803. A 24-year-old nurse recently married presents to the dermatology


department complaining of numerous small flat pustules, whitish to yellowish in
colour, on an erythematous base on hands and feet for 2-months duration. No
history of drug intake or any contact with local irritants. The cultural test showed
sterile pustules, confirming diagnosis of palmo-planter pustular psoriasis. She has
previously maintained reasonable control of her condition with super potent topical
steroids and vitamin D analogues. What would be the next reasonable step in
treatment?
a. Methotrexate
b. Infliximab
c. Acitretin
d. Hand and foot PUVA
e. Hydroxycholoroquine.

Q804. A 16-year-old boy presented to the dermatology clinic with crops of numerous
reddish-brown crusted papules and widespread hypopigmented macules on the
trunk, arms and legs, which had been present for 2 months. On examination, there
were multiple erythematous to purple crusty papules with some small ulcers,

140
vesicles and pustules. In some areas where lesions have resolved varioliformis
scarring has been left behind. What is the most likely diagnosis?
a. Pityriasis lichenoides et varioliformis acuta (PLEVA)
b. Pityriasis lichenoides chronic (PLC)
c. Mycosis Fungoides
d. Guttate psoriasis
e. Small plaque parapsoriasis.

Q805. A 62-year-old man presented with diffuse erythroderma of gradual onset. He


was systemically well. On examination, there were follicular hyperkeratosis on an
erythematous base and there were large orange-red patches with distinctive islands
of sparing. The palms and soles showed an orange-red waxy keratoderma and there
was fine diffuse scale on the scalp. The nails showed a yellow-brown thickened nail
plate with subungual debris. Which of the following treatments would you not
consider for this patient?
a. Hydroxycholoroquine
b. Methotrexate
c. Acitretin
d. Isotretinoin
e. Etanercept (Enbrel)

Q806. A 15-year-old girl presented to the dermatology clinic with skin rash on the
trunk for 2 weeks duration. The rash started as a single lesion appearing on her back
that gradually enlarged over a few days, then multiple lesions appeared over the
trunk and upper arms. The lesions were oval shaped, skin-coloured and have a
slightly raised margin. They vary from 2–4 cm in size, have central fine scale and a
collarette of scale at the free edge. The lesions are asymptomatic and the rash does
not distress the patient. What is the appropriate course of action?
a. Start UVB phototherapy
b. Start a course of erythromycin
c. Reassure the patient and advise a little sun exposure
d. Start a topical steroid
e. Start a course of oral prednisolone.

Q807. A 52-year-old man was seen in clinic as an urgent referral. He gave a 2-week
history of a spreading rash that covered the whole body. The patient feels generally
unwell, lethargic and thirsty. On examination, patient was shivering and had difficulty in
standing. He was erythrodermic with over 95% of his skin showing a non-specific

141
confluent erythema. He had no history of drug intake or any skin disease. What is
the initial step should be done?
a. Admit the patient to the dermatology ward for assessment and
stabilization of his medical condition
b. Admit the patent in the dermatology department and start
systemic steroid (1 mg/kg)
c. Give the patient hydrocortisone injection and send the patient
home
d. Start dressing with emollient and send the patient home
e. Work up for cyclosporine therapy

Q808. A 12-year-old boy presented to the dermatology department, complaining of


bizarre configurations of erythema, oedema, vesicle and bullae, on her exposed arms
and legs. Three days previously, she had been playing in a field, full of plants.
What is the most likely diagnosis?
a. Allergic contact dermatitis
b. Phytophotodermatitis
c. Pemphigus vulgaris
d. Irritant contact dermatitis
e. Erythema multiforme

Q809. An 18-year-old girl has a henna tattoo on her right forearm by a beach vendor. Three
days later her tattoo becomes progressively more inflamed and sore to the point of
developing bullae. When she sees you three months later, the reaction and henna
have faded but an area of post inflammatory pigmentation remains. What important
information should you give the patient?
a. The reaction has settled and it is safe to get another henna tattoo
b. It will be safe to get another henna tattoo in six months time
c. She needs to carry an adrenalin containing pen as she is at
risk of anaphylaxis
d. She must not use permanent hair dyes in the future
e. She must avoid all henna containing products in the future.

Q810. A 32-year-old female pharmacist presented with hand dermatitis. On


examination, her hands are dry and cracked with erythema and mild paronychia. At
work she wears vinyl gloves whenever handling medicines and washes her hands regularly.
She has no particular hobbies, as she is busy with her three young children. What is the
likely diagnosis?
a. Irritant hand dermatitis
b. allergic hand dermatitis
142
c. Atopic dermatitis
d. Contact dermatitis to vinyl gloves
e. Pompholyx eczema

Q811. A 52-year-old man was seen in clinic as an urgent referral. He gave a 2-week
history of a spreading rash that covered the whole body. The patient feels generally
unwell, lethargic and thirsty. On examination, patient was shivering and had difficulty in
standing. He was erythrodermic with over 95% of his skin showing a non-specific
confluent erythema. He had no history of drug intake or any skin disease. Which of
the following is not causing this condition?
a. Psoriasis.
b. Pityriasis rubra pilaris.
c. Cutaneous lymphoma
d. Drugs
e. Pityriasis rosea

Q812. Desmoglein-1. The antigen in the skin in which of the following autoimmune
skin diseases?
a. Pemphigus foliaceus
b. Bullous impetigo
c. Dermatitis herpetiformis
d. Bullous pemphigoid
e. Pemphigoid gestationis

Q813. What is the major component of the anchoring fibril?


a. Type I Collagen
b. Type III Collagen
c. Type IV Collagen
d. Type VII Collagen
e. Laminin

Q814. Which type of pigment is present in red/blond hair?


a. The absence of melanin
b. The presence of pheomelanin
c. The reduced activity of tyrosinase
d. The reduced activity of DOPA dehydroxylase
e. The presence of eumelanin

Q815. What is the Darier's sign?


a. When an intact epidermis tears away from the underlying dermis,
leaving a moist surface
b. Spreading bulla phenomenon with pressure on an intact bulla
c. Swollen, itchy and or red after stroking the skin
d. Central depression within a lesion when squeezed along its margin
e. Disappearance of color when the lesion is pressed.
143
Q816. What is the Nikolsky sign?
a. When an intact epidermis tears away from the underlying dermis
, leaving a moist surface
b. Spreading bulla phenomenon with pressure on an intact bulla
c. Swollen, itchy and or red after stroking the skin
d. Central depression within a lesion when squeezed along its margin
e. Disappearance of color when the lesion is pressed.

Q817. What is the Asboe-Hansen sign?


a. When an intact epidermis tears away from the underlying dermis,
leaving a moist surface
b. Spreading bulla phenomenon with pressure on an intact bulla
c. Swollen, itchy and or red after stroking the skin
d. Central depression within a lesion when squeezed along its margin
e. Disappearance of color when the lesion is pressed.

Q818. What is the Diascopy test?


a. When an intact epidermis tears away from the underlying dermis,
leaving a moist surface
b. Spreading bulla phenomenon with pressure on an intact bulla
c. Swollen, itchy and or red after stroking the skin
d. Central depression within a lesion when squeezed along its margin
e. Blanching of the skin when a pressure is applied with a finger or
glass slide.

Q819. A 68-year-old cardiac patient with ischemic dilated cardiomyopathy two years
back, presented to dermatology department with an asymptomatic blue-
gray discoloration of the centro-facial region. Physical examination revealed a blue-
gray hyperpigmentation of the forehead, cheeks, nose, malar regions and chin.
Results of routine laboratory tests were normal. A3-mm punch biopsy specimen showed
normal epidermis and a perivascular deposition of yellow-brown pigment within
microphages in the dermis. Which one of the following is the correct diagnosis, given
the patient's history, the physical and skin biopsy findings?
a. Actinic lichen planus
b. Hemochromatosis
c. Amiodarone-induced hyperpigmentation
d. Ashy dermatosis (erythema dyschromicum perstans)
e. Argyria

Q820.Which of the following malignancy is associated with alopecia neoplastica?


a. Lymphoma
b. Squamous cell carcinoma
c. Breast carcinoma
d. Colon carcinoma
e. Melanoma
144
Q821.What is the term used to describe the parallel nature of coherent light waves?
a. Monochromicity
b. Coherence
c. Collimation
d. Energy
e. Power

Q822.The role of imiquimod in the treatment of superficial basal cell carcinoma is


based on, the induction of which cytokine?
a. Interleukin-2
b. Tumor growth factor-alpha
c. Interleukin-4
d. Interferon-gamma
e. Tumor necrosis factor-beta

Q823. Which of the following cosmetic injectable, can be seen on routine dental x-
rays?
a. Botox
b. Zyplast
c. Radiesse
d. Cosmoderm
e. Restylane

Q824. Which of the following suture materials is most likely to cause infection?
a. Silk
b. Vicryl (polyglactin )
c. Prolene (polypropylene)
d. Nylon
e. Catgut

Q825. What is the most important function of a postsurgical wound dressing?


a. Immobilize the wound
b. Prevent wound infection
c. Prevent scar formation
d. Reduce pain
e. Evert wound edges

Q826. Which laser would not be a good choice for the treatment of a red cosmetic
tattoo around the lips?
a. Carbon dioxide laser
b. Pulsed dye laser
c. Q-switched laser

145
d. Alexandrite laser
e. Nd:YAG laser

Q827. Which of the following cosmetic injectables binds water to create volume, has
a low allergic reaction profile, and lasts 6 to 12 months?
a. Botulinum toxin
b. Calcium hydroxylapatite
c. Human-derived collagen
d. Hyaluronic acid
e. Silicone

Q828. Which of the following complications has been reported with infra-orbital
injections of Botox?
a. Nystagmus
b. Blindness
c. Festooning
d. Astigmatism
e. Photophobia

Q829. A 78-year-old man with an infiltrative basal cell carcinoma of the left nasal ala.
What is the best treatment option?
a. MOHS surgery
b. 5-fluorouracil cream
c. Electrodessication and curettage
d. Standard excision
e. Imiquimod

Q830. Which of the following demonstrates the highest risk of metastasis?


a. Squamous cell carcinoma on the ear
b. Squamous cell carcinoma within a scar
c. Squamous cell carcinoma on the lip
d. Squamous cell carcinoma on the scalp
e. Squamous cell carcinoma on the nose

Q831. How long the maximal vasoconstriction effect of epinephrine, after cutaneous
infiltration with lidocaine be achieved?
a. 1 minute
b. 7 minutes
c. 15 minutes
146
d. 30 minutes
e. 1 hour.

Q832. Which of the following determines the wavelength of a laser?


a. Fluence
b. Pulse width
c. Medium
d. Spot size
e. Pulse duration

Q833. Which ocular structure is at most risk of injury from an erbium: YAG laser?

a. Lens
b. Retina
c. Cornea
d. Vitreous humor
e. Iris

Q834. What part of the eye may be damaged by exposure to irradiation from the
carbon dioxide laser?
a. Retina
b. Lens
c. Sclera
d. Cornea
e. Iris

Q835. When can a patient who undergoes follicular-unit hair transplantation


expect hair growth of the grafted follicles to begin?
a. 1 month
b. 2 to 3 months
c. 6 months
d. 9 months
e. 12 months

Q 836. Which of the following cosmetic injectables has the longest duration of
action?
a. Botox
b. Sculptra (poly L-lactic acid)
c. Restylane (hyaluronic acid)
d. Cosmoderm (collagen)
e. Juvederm

Q 837. On electron microscope, which cell demonstrate cytoplasmic projection and


secretory granules?
a. Langerhans cell
b. Keratinocyte
c. Mast cell
147
d. Macrophage
e. Melanocyte

Q 838. When do melanocyte begin to synthesize melanin?


a. 2nd month of gestation
b. 3rd month of gestation
c. 4th month og gestation
d. 5th month of gestation
e. 6th month of gestation

Q 839. Which of the following disease, direct immunofluorescence is of no value in the


diagnosis?
a. Neonatal lupus erythematosus
b. Lichen planus
c. Mixed connective tissue disease
d. Systemic lupus erythematosus
e. Erythema multiforme
Q 840. Which of the following is true about hair grow?
a. Hair grows 0.004 mm/day
b. Hair grows 0.04 mm/day
c. Hair grows 0.4 mm/day
d. Hair grows 4 mm/day
e. Hair grows 10 mm/day.

Q 841. Sebaceous glands, secrete sebum through which of the following mechanisms?
a. Holocrine
b. Apocrine
c. Merocrine
d. Holocrine and apocrine
e. Holocrine and merocrine.

Q 842. Red or blonde hair pigmentation, primarily result from which of the following?
a. The presence of eumelanin
b. The presence of pheomelanin
c. The presence of melanin
d. The reduced activity of tyrosinase
e. The reduced activity of DOPA decarboxylase.

Q 843. The microflora of the pilosebaceous unit, consist of which of the following?
a. Pityrosporum ovale
b. Staph aureus
c. Escherichia coli
d. Pseudomonas aeruginosa
e. Streptococcus pyogenes

Q 844. Apocrine glands are found in all of the following areas of the body except?
148
a. Palms
b. Breast
c. Axilla
d. Eyelid
e. Perineum

Q 845. Which of the following skin disease is characterized by, annular lesions?
a. Acne vulgaris
b. Tinea corporis
c. Urticaria
d. Erythema multiforme
e. Viral warts

Q846. Which of the following is Infectious origin


a. Lichen planus
b. Seborrhoeic keratitis
c. Melanoma
d. Hairy leukoplakia of tongue
e. Bacillary angiomatosis

Q 847. The following disease is premalignant lesion except?


a. Arsenic keratosis
b. Seborrheic keratosis
c. Bowen's disease
d. Naevus sebaceous
e. Actinic keratosis

Q 848. Which of the following skin disease, the topical steroids is useful?
a. Dermatitis artifacta
b. Perioral dermatitis
c. Inflammatory acne vulgaris
d. Rosacea
e. Hypertrophic scars

Q 849. Which of the following disease is transmitted sexually?


a. Granuloma inguinale
b. Erythroplasia of queyrat
c. Pearly papules of penis
d. Lichen sclerosis
e. Fordyce spots

149
Q 850. Blood serology is useful in the diagnosis which of the following disease?
a. Latent syphilis
b. Genital herpes
c. Trichomoniasis
d. Granuloma inguinale
e. Gonorrhea

Q 851. Which of the following treatment should be avoided in a one-year old child
who presents with monomorphous, nonpruritic, flat-topped papules or papulo-
vesicular lesion, on the face, buttocks, extremities, palms and soles (Gianotti-Crosti
or papular acrodermatitis of childhood)?
a. Mild topical steroid
b. Topical Fusidic acid (fucidin)
c. Tacrolimus (Elidel)
d. Observation
e. Emollient

Q 852. Which of the following malignancy is commonly associated with multiple


Juvenile xanthogranuloma?
a. Acute myelogenous leukemia
b. Chronic myelogenous leukemia
c. Acute lymphocytic leukemia
d. Chronic lymphocytic leukemia
e.

Q 853. A newborn has a nodule over his lumbar spine. Skin biopsy reveals a lipoma.
What is the next appropriate step?
a. Observation
b. Excision of the lesion
c. Genetic testing
d. Imaging study
e. Malignancy work up

Q 854. A full term neonate presented to the dermatology clinic having small pustules
with no underlying erythema present at delivery. The pustules are easily removed
with clearing of the vernix and a collarette appears. A gram stain is done showing
predominately neutrophils without bacteria. What is the most likely diagnosis?
a. Miliaria
b. Erythema toxicum neonatorum
c. Transient neonatal pustular melanosis
d. Congenital candidiasis
150
e. Urticaria pigmentosa

Q 855. Which of the following is a potential complication of subcutaneous fat


necrosis of the newborn?
a. Skin necrosis
b. Hypercalcemia
c. Acute renal failure
d. Hepatitis
e. Elevated uric acid levels

Q 856. Which of the following is the first symptom of ataxia telangiectasia?


a. Conjunctival telangiectasia
b. Facial telangiectasia
c. Hematologic abnormalities
d. Cerebellar ataxia
e. Breast cancer

Q 857. In Mal de Meleda. Which of the following is false?


a. Palmo-planter keratoderma
b. Hyperhidrosis
c. Keratotic plaque on the knee and elbow
d. Subungual hyperkeratosis
e. Alopecia.

Q 858. Ichthyosis hystrix is characterized by, which of the following gene defects?
a. Keratins 1 and 9
b. Keratins 1 and 10
c. Keratins 5 and 14
d. Keratins 6 and 16
e. None of these answers are correct.

Q 859. In Netherton syndrome. Which location of the body would most likely have
hairs demonstrating trichorrhexis nodosa?
a. Scalp
b. Eyebrows
c. Eyelashes
d. All of these answers are correct
e. None of these answers are correct.

Q 860. Dysplastic nevus syndrome. It is linked with which of the following


malignancy?
a. Breast malignancy
b. Thyroid malignancy
c. Pancreatic malignancy
d. Renal cell carcinoma
e. Colon carcinoma.
151
Q 861. Which of the following is true regarding tuberous sclerosis?
a. Confetti-like macules are typically present at birth
b. Facial angiofibromas are the most common cutaneous
manifestation
c. Hypomelanotic macules (ash leaf spots), typically presenting at
birth or early infancy
d. Periungual fibromas are usually present in the early childhood
e. Multiple neurofibroma.

Q 862. In Sturge-Weber syndrome. Which ophthalmologic complication is associated


with this disorder?
a. Glaucoma
b. Ectopic lens
c. Cataracts
d. Posterior sub-capsular lenticular opacity
e. Retinitis pigmentosa.

Q 863. What medicine taken in pregnancy has been most closely associated with
aplasia cutis congenita?
a. Methimazole
b. Levothyroxine
c. Magnesium
d. Isotretinoin
e. Trimethoprim/sulfamethoxazole.

Q 864. Epidermolysis bullosa simplex is caused by blistering in which structure?


a. Granular layer keratinocyte
b. Spinous layer keratinocyte
c. Basal layer keratinocyte
d. Lamina densa
e. Sublamina densa.

Q 865. Which ocular finding may be seen in a patient with pseudoxanthoma


elasticum?
a. Comma shaped corneal opacities
b. Retinitis pigmentosa
c. Glaucoma
d. Angioid streaks
e. Cataract.

152
Q 866. What is the treatment of acrodermatitis enteropathica?
a. Zinc supplementation
b. Iron supplementation
c. Vitamin B1 supplementation
d. Vitamin B12 supplementation
e. Phlebotomy.

Q 867. Which of the following conditions is inherited in an X-linked recessive


manner?
a. Epidermolysis bullosa simplex
b. Ichthyosis vulgaris
c. Sjogren-Larsson syndrome
d. Wiskott - Aldrich syndrome (WAS)
e. Netherton syndrome

Q 868. Which one of the following is the most common oncogenic virus in patients
with epidermodysplasia verruciformis?
a. HPV-5
b. HPV-8
c. HPV-13
d. HPV-16
e. HPV-33

Q 869. The following are true in KID syndrome except?


a. Autosomal recessive trait
b. Keratitis
c. Red, rough thickened plaque on the skin (erythrokeratoderma) on
the face, extrimities, trunk and palms and soles
d. Sensorineural deafness
e. Sparse hair or alopecia

Q 870. In X-linked ichthyosis. Which of the following is false?


a. Hereditary deficiency of the steroid sulfatase enzyme
b. Scaling of the skin, particularly on the neck, trunk and extrimities
(extensor surfaces)
c. Corneal opacities may be present but do not affect vision
d. Cryptorchidism
e. Confetti-like macules on the trunk.

153
Q 871. Which of the following is the main cause of death in patients with
dyskeratosis congenita?
a. Oral squamous cell carcinoma
b. Leukemia
c. Renal cell carcinoma
d. Pancytopenia
e. Atherosclerotic heart disease

Q 872. Which of the following systemic complications in Patients with Hermansky-


Pudlak syndrome (oculocutaneous albinism)?
a. Arteriovenous malformations
b. Pulmonary fibrosis
c. Gastroesophageal reflux disease
d. Aortic stenosis
e. Rectal abscesses

Q 873. What is the inheritance pattern of chronic granulomatous disease?


a. Autosomal recessive
b. Autosomal dominant
c. X-linked recessive
d. X-linked dominant
e. Sporadic

Q 874. Which of the following cosmetic injectables has the longest duration of
action?
a. Botox
b. Sculptra (poly L-lactic acid)
c. Restylane (hyaluronic acid)
d. Cosmoderm (collagen)
e. Myobloc (botulinum toxin-B)

Q 875. Which of the following lasers uses a solid as its medium?


a. Argon laser
b. Pulsed dye laser
c. Alexandrite laser
d. Krypton
e. Carbon dioxide laser

Q 876. Which of the following chemical peels does not need to be neutralized?
a. Salicylic acid
b. TCA 40%
c. Glycolic acid
d. Lactic acid
e. Phenol

154
Q 877. What is the most appropriate suture for closing a wound on the buccal
mucosa?
a. Prolene
b. Silk
c. Monocryl
d. Braided nylon
e. PDS

Q 878. Which of the following lasers has the greatest depth of penetration in the
skin?
a. Pulsed dye laser (585 nm)
b. Diode laser (800 nm)
c. Nd: YAG laser (1064 nm)
d. Erbium: YAG laser (2940 nm)
e. CO2 laser (10,600 nm)

Q 879. What is the most common adverse reaction seen with betadine?
a. Ototoxicity
b. Teratogenicity
c. Allergic contact dermatitis
d. Seizures
e. Corneal damage

Q 880. Which needle shape is most commonly used in cutaneous surgery?


a. 1/4 circle
b. 3/8 circle
c. 1/2 circle
d. 5/8 circle
e. Compound curve

Q 881. Which of the following skin disease, imiquimod (Aldara) can be used in the
treatment?
a. Genital wart
b. Actinic keratosis
c. Superficial basal cell carcinoma
d. All of the above
e. None of the above

Q 882. What is the type of laser is the most commonly used in the treatment of acne
scar?
a. Pulsed dye laser
b. Intense pulsed light
c. Photodynamic therapy
d. Radiofrequency
e. Q-switched Nd:YAG laser
155
Q 883. The following thrombotic complications have been reported after
discontinuing of aspirin therapy prior to surgery except?
a. Stroke
b. Myocardial infarction
c. Pulmonary embolism
d. Transient ischemic attack
e. Cerebral embolism

Q 884. What is the common use of topical vitamin K?


a. Decrease the appearance of infra-orbital pigmentation
b. Minimize the appearance of telangiectasia
c. Reduce the severity of laser-induced purpura
d. Increase collagen production
e. Induce keratinocyte differentiation

Q 885. Which of the following parameters determines the wavelength of a laser?


a. Medium
b. Fluence
c. Spot size
d. Pulse duration
e. Lens length

Q 886. How long after surgery, the use of dermabrasion to improve the appearance
of a scar?
a. One week
b. Three weeks
c. Sex weeks
d. Sex months
e. One year

Q 887.Which of the following laser is effective in treatment of psoriasis?


a. Carbon dioxide laser
b. Excimer laser
c. Ruby Laser
d. Pulsed dye laser
e. Alexandrite laser

Q888. Which of the following criteria carries the worst prognosis for a patient with a
squamous cell carcinoma?
a. Size of tumor > 1 cm
b. Depth of invasion > 4 mm
c. Perineural invasion
d. Anatomic location
e. Immunosuppression

156
Q889. Which of the following lasers would be effective in the treatment of
rhinophyma?
a. Pulsed dye laser
b. Ruby laser
c. Alexandrite laser
d. KTP laser
e. Carbon dioxide laser

Q890. What is the tattoo pigment responsible for most lichenoid reactions?
a. Titanium dioxide
b. Carbon
c. Mercuric sulfide
d. Chromates
e. Iron oxide

Q891. Which of the following treatment modalities for basal cell carcinoma has the
highest long-term cure rate?
a. MOHS surgery
b. Radiation
c. Cryotherapy
d. Electrodessication and curettage
e. Surgical excision

Q892. Which of the following cosmetic injectable is a contraindication in a patient


with a history of anaphylaxis?
a. Myobloc
b. Radiance
c. Cosmoderm
d. Restylane
e. Sculptra
Q893. Which of the following lasers may cause milia formation as a post-procedure
complication?
a. Excimer
b. KTP
c. Pulsed Dye
d. ND-Yag
e. Erbium

Q894. Which of the following topical antibacterial agents may cause neutropenia?
a. Mupirocin
b. Silver sulfadiazine
c. Polymyxin
d. Bacitracin
e. ovodine-iodine

157
Q895. Which of the following type of laser is the most appropriate laser to treat a
port wine stain (capillary haemangioma) on an infant’s cheek?
a. Excimer laser
b. KTP laser
c. Pulsed Dye
d. ND-Yag laser
e. Erbium laser

Q896. The following complications should be aware using a carbon dioxide laser to
treat verruca except?
a. Ocular damage
b. Transmission of viral disease
c. Purpura
d. Recurrence of lesion
e. Scarring

Q897. The following factors increase susceptibility to the development of skin


cancer in organ transplant recipients except?
a. Fair skin (Fitzpatrick types I-III)
b. History of chronic sun exposure
c. Duration of immunosuppression
d. History of HPV infection
e. CD8 lymphocytopenia.

Q898. What is the safety dose of lidocaine with epinephrine when used in tumescent
anesthesia?
a. 3 mg/kg
b. 4.5 mg/kg
c. 7 mg/kg
d. 20 mg/kg
e. 50 mg/kg
Q899. Which of the following is true regarding lidocaine local anesthetic?
a. 1% lidocaine is equal to 1g/10ml
b. Duration with no epinephrine is 4-6 hours
c. Maximum dose with no epinephrine is 7mg/kg
d. Beta blockers increase lidocaine levels
e. Allergy most commonly occurs to propylene glycol preservatives

Q900. Which of the following is the formation of granulomas is a potential


complication?
a. Isotretinoin
b. Autologous fat
c. CO2 resurfacing
d. Silicone
e. Hydroxychloroquine

158
Q901. Which of the following medicines has a newly reported side effect of eruptive
epidermoid cysts?
a. Minocycline
b. Retinoids
c. Imiquimod
d. Erythromycin
e. Potassium iodide

Q902. Which part of the eye is most likely to be damaged to exposure to a pulsed
dye laser?
a. Retina
b. Cornea
c. Lens
d. Conjunctiva
e. Sclera

Q903. Which ocular structure is at most risk of injury from an erbium:YAG laser?
a. Lens
b. Retina
c. Cornea
d. Iris
e. Pupil

Q904. Which of the following diseases is contraindicated the use of EMLA cream?
a. Atopic dermatitis
b. Psoriasis vulgaris
c. Sickle cell anemia
d. Methemoglobinemia
e. Peripheral neuropathy

Q905. What is the common use of topical vitamin K?


a. Increases epidermal differentiation
b. Reduce severity of postoperative purpura
c. Decrease epidermal pigmentation
d. Improve fine winkling
e. Improve skin hydration

Q906. The use of imiquimod (Aldara) for the treatment of superficial basal cell
carcinoma is advocated at which treatment regimen?
a. Five times per week for 6 weeks
b. Five times per week for 4 weeks
c. Three times a week for 6 weeks
d. Three times per week for 4 weeks
e. Three times a week for 10 weeks

159
Q907. Which of the following sutures is the first to be absorbed?
a. Catgut
b. Vicryl
c. Silk
d. Nylon
e. Prolene

Q908. What is the effect of activation of the procerus muscle?


a. Peri-ocular wrinkles
b. Wrinkles at the nasal root
c. Wrinkles on the forehead
d. Perioral wrinkles
e. Accentuation of the nasolabial folds

Q909. Which potential complications is seen exclusively with phenol peels?


a. Milia
b. Hypopigmentation
c. Toxic shock syndrome
d. Prolonged erythema
e. Laryngeal edema

Q910. Which of the following is the earliest symptom of lidocaine toxicity?


a. Tachycardia
b. Perioral tingling
c. Nystagmus
d. Cyanosis
e. Seizure

Which of the following cosmetic injectables is FDA-approved for the treatment of


lipoatrophy?
a. Radiance
b. Silicone
c. Botox
d. Sculptra
e. Isolagen

Sculptra (called New-Fill outside of the US) is a biodegradable filler composed of


poly-L-lactic acid, the same material used in absorbable sutures. It is biocompatible
and non-allergenic and was approved by the FDA in August 2004 for the treatment
of HIV-associated lipoatrophy.

The following are the signs and symptoms of lidocaine toxicity except?
a. perioral numbness
160
b. Ototoxicity
c. Slurred speech
d. Nystagmus
e. Seizure
The first signs of lidocaine toxicity are CNS symtpoms that resemble inebriation with
alcohol. These symptoms include stupor, dysarthria, circumoral numbenss and
dizziness. Further increases in toxicity leads to nausea, metallic taste, twitching, and
seizures.. Without epinephrine the maximum dose of lidocaine 4 mg/kg. For a 70 kg
individual, this is 300 mg or 30 ml of a 1% lidocaine solution. For a preparation of
lidocaine with epinephrine the maximum dose is 7 mg/kg. For a 70 kg individual, this
is 500 mg or 50 ml of a 1%

True statements regarding skin cancer in organ transplant recipients include all of
the following except:
65 fold increase in development of SCC compared with general population
Mohs micrographic surgery indicated for in-transit metastases
Cutaneous malignancies develop 3-5 years after organ transplantation
Extent of tumor development related to degree of immunosuppression
Skin cancer is the most common cancer in transplant patients

Mohs micrographic surgery indicated for in-transit metastases In-transit metastases


is a common manifestation of metastatic disease

The use of EMLA cream is contraindicated in patients with which of the following?
1 Atopic dermatitis
2 2 Neomycin allergy
3 3 Sickle cell anemia
4 4 Methemoglobinemia
5 5 Peripheral neuropathy

EMLA is a topical anesthetic composed of a eutectic mixture of 2.5% lidocaine and


2.5% prilocaine. The major concern when using EMLA is the potential risk of
methemoglobinemia. Patients with glucose-6-phosphate deficiency and patients
taking methemoglobin-inducing agents (dapsone, phenytoin, sulfonamides) are
more susceptible to developing methemoglobinemia. In addition, cares hould be
taken when using EMLA in infants less than 3 months of age because of the
incomplete maturation of their NADH-methemoglobinemia reductase system

Which type of collagen is the first to be deposited in a healing wound?


1 Type I collagen
2 Type II collagen
3 3 Type III collagen
4 4 Type IV collagen
5 5 Type VII collagen

161
Collagen type III is the predominant collagen in early wound healing

Which of the following sutures is best used for mucous membranes?


Catgut
Silk
Polyglactin 9, 10
Nylon
Polydioxanone
Silk is a natural non-absorbable suture used mainly for eyelid, lip, or intraoral
surgeries as they are more comfortable to the patient on these surfaces. This suture
has excellent handling as it is less stiff and flows easily through soft tissue. However,
it has high tissue reactivity, low tensile strength, and is associated with a higher risk
of infection

Reticulate eythema is a side effect seen with which treatment?


1 Mesotherapy
2 Sclerotherapy
3 3 Diode laser
4 4 Cryotherapy
5 5 Pulsed dye laser
Reticulate erythema is a recently-reported side effect of diode laser treatment. High
energy Fluence and a history of chilblains are believed to be predisposing risk
factors. Laser treatment should be discontinued at the first sign of this complication.

All of the following are alpha-hydroxy acids


except:1 Lactic acid2 Citric acid3 Glycolic acid4 Tartaric acid5 Salicylic acid
Alpha-hydroxy acids are naturally occurring carboxylic acids found in many foods.
The alpha-hydroxyacids include glycolic, lactic, malic, citric, and tartaric acids. Factors
that determine the intensity of the peel include the concentration of the acid, pH,
degree of buffering, vehicle formulation, frequency ofapplication, conditions of
delivery, and the length of time the acid is placed on the skin. Salicylic acid isa type
of beta-hydroxy acid

The pain associated with Botulinum A Exotoxin injection is attributed to


the:1 Needle gauge2 PH3 Preservative-free saline4 Exotoxin5 Temperature
Botulinum A exotoxin is used for multiple reasons, most often for the treatment of
dynamic facial lines.In a double-blind, randomized controlled study,
investigators found that botulinum A exotoxinreconstituted with preservative-
containing saline less painful than with preservative-free saline

Which structual component of local anesthetics (e.g. lidocaine) is responsible for the
onset of
activity?1 Aromatic ring2 Intermediate chain3 Amine end4 Length of the carbon chai
n5 None of the above

162
Local anesthetics like lidocaine contain 3 principle structural elements. The aromatic
ring determines theonset of activity, the intermediate chain defines the class (amide
vs. ester), and the amine end isresponsible for the duration of action.

Which is the following is true regarding


cellulite?1 It is caused by lipoatrophy of fat.2 Adipocytes in the gluteofemoral region
are more responsive to lipolysis.3 Norepinephrine is the only hormone that affects li
polysis.4 It occurs in up to 50% of postpubertal females.5 Caucasian women tend to
get cellulite more than Asian women
According to the JAAD CME article in March 2010, cellulite is caused by the
herniation ofsubcutaneous fat, not lipoatrophy, within fibrous connective tissue. It is
prevalent in almost all postpubertal females. It is most notable in the pelvic region,
lower limbs, and abdomen. The adipocytesin the gluteofemoral region are larger and
are influenced by female sex hormones. They aremetabolically more stable and
resistant to lipolysis. The hormones that acutely affect lipolysis inadipocytes are
catecholamines (epinephrine and norepinephrine) and insulin

Which cosmetic filler substance is contraindicated in the


glabella?1 Bovine collagen2 Human collagen3 Hyaluronic acid4 Calcium hydroxyapati
te5 Poly-L-lactic acid
Bovine collagenAs placement of bovine collagen is typically deeper in the tissues
compared to other filler, its use hasresulted in vascular occlusion and necrosis in the
glabellar area, and is therefore contraindicated

Which of the following is a potential side effect of ambulatory tumescent


liposuction?1 Muscle atrophy2 Breast enlargement3 Decreased appetite4 Abdominal
perforation5 Pulmonary embolus
Breast enlargement is a relatively common and unexpected side effect of tumescent
liposuction. Sincethe majority of these patients report increased breast size in the
absence of weight gain, some authors postulate shifting hormone ratios as the
etiology of this paradoxical breast augmentation. Abdominal perforation, respiratory
failure and pulmonary embolus are complications that are seen almostexclusively in
liposuction patients that receive general anesthesia or intravenous sedation

young African American patient presents with anemia and spontaneously

appearing leg ulcers over both lateral and medial malleoli. The most likely diagnosis
is:1 Atherosclerotic disease2 Sickle cell anemia3 Factitial dermatitis4 Lupus erythema
tosus5 Trauma

163
anemiaSickle cell anemia should be considered most likely in any young African
American with spontaneousleg ulcers. The ulcers are more common in people
with severe anemia

What is the most common internal cause of intractable pruritus?1 Hepatitis C2


Hypothyroidism3 Hyperthyroidism4 Chronic renal failure5 Internal malignancy
Chronic renal failure is the most common internal systemic cause of pruritus. Up to
49% of patientswith chronic renal failure have pruritus. Other systemic causes of
pruritus include liver disease,hepatitis C, hypo and hyperthryoidism, iron deficiency
anemia, polycyhtemia vera, Hodkin'slymphoma, leukemia, carcinoid, internal
malignancy, AIDS, and internal parasites.

Regarding eruptive xanthomas, which of the following is


true?1 They occur in the setting of familial hyperlipidemia types I, IV, and V2 They oc
cur in the setting of familial hyperlipidemia types II and III3 Triglyceride levels are usu
ally below 500mg/dl4 They are most commonly found on the eyelids5 They are not r
elated to alcohol consumption
Eruptive xanthomas generally occur in patients with triglyceride levels of 2000mg/dl
or greater.Associations include poorly-controlled diabetes mellitus, retinoids,
estrogens, excessive alcoholconsumption (leading to pancreatitis) and familial
hyperlipidemias types I, IV and V. Clinically, theyappear as crops of firm, non-tender
yellowish papules with an erythematous border. Most commonly,they occur on the
extensor surfaces, but they can be diffuse. A reduction in triglycerides and/or
tightglucose control usually results in a reduction in the number of lesions

Which of the following skin findings is most closely linked to hepatocellular


carcinoma as a paraneoplastic
syndrome?1 Pityriasis lichenoides2 Pityriasis alba3 Pityriasis amiantacea4 Pityriasis r
otunda5 Pityriasis rosea
Pityriasis rotunda is an dermatosis that features characteristic discrete, circular,
scaly, brown patches onthe trunk and extremities. Pityriasis rotunda may be
associated with systemic diseases in certain racially predisposed groups (blacks), and
has been linked to hepatocellular carcinoma

All of the following are true regarding incontinentia pigmenti


EXCEPT:1 It is caused by a mutation in the NEMO gene2 It is an X-
linked recessive disorder3 Inflammation and blistering may be followed by hyperkera
totic, verrucous lesions4 It is associated with cerebellar ataxia5 It is associated with c
oloboma and retinal detachment

Incontinentia pigmenti is an X-linked dominant disorder caused by a mutation in the


NEMO gene.Females only present at birth with linear lesions of inflammation and
blistering (stage 1), followed byhyperkeratotic verrucous areas (stage 2),
hyperpigmentation (stage 3), and then atrophy (stage 4).Systemic findings include
psychomotor retardation, microcephaly, seizures, cerebellar ataxia,coloboma, and
retinal detachment
164
A patient on hemodialysis presents with indurated plaques having a peau d

orange texture on the bilateral lower legs. Which of the following statements is
TRUE?1 The diagnostic histopathological findings include acanthosis and hyperkerato
sis2 Serum protein electrophoresis should be performed3 Dapsone will likely be effe
ctive treatment4 The face is usually is affected5 The palms and soles are usually affec
ted
This patient likely has nephrogenic fibrosing dermopathy (NFD). NFD is an acquired,
idiopathicdisorder that occurs in renal disease patients. It resembles scleroderma or
eosinophilic fasciitisclinically and scleromyxedema histopathologically. Large areas of
indurated skin with fibrotic nodulesand plaques develop. The extremities are most
commonly involved, followed by the trunk. The face, palms and soles are almost
never involved. Histopathologically, NFD displays a proliferation of dermalfibroblasts
and dendritic cells, thickened collagen bundles, increased elastic fibers, and
mucindeposition. Serum protein electrophoresis and immunoelectrophoresis results
are negative unlikescleromyxedema, and may be helpful to distinguish the two
diseases. NFD is usually a chronic, progressive condition, and favorable responses to
medical intervention are anecdotal

A 2 year old child is evaluated for suspected diagnosis of neurofibromatosis. Which


of the followingdiagnostic findings is typically absent on exam in this age group?1
neurofibromas2 macrocephaly3 pigmented iris hamartomas4
seizures5 cafe au lait macules
Cutaneous neurofibromas typically appear after puberty. The absence of
neurofibromas in youngchildren does not rule out this diagnosis. Pigmented iris
hamartomas are more likely finding in youngchildren than neurofibromas.

A 16-year-old girl known to suffer from acne vulgaris has been started
on isotretinoin.Which of the following statements best applies to treatment with
isotretinoin?
It is contraindicated in patients with renal artery stenosis

It can cause hirsutism

It can cause hyperkalaemia and hence electrolytes should be checked everymonth

Pregnancy should be avoided during and 1 month after treatment


Your answer
It may cause haemoptysis
sotretinoin is indicated for the treatment of severe inflammatory acne. However, it
causesmarked dryness of the skin and mucous membranes, especially the lips, and
can result inminor nosebleeds. Due to its teratogenicity, pregnancy must be
excluded prior to itsinitiation and during treatment as well as for 1 month after
treatment. Other side-effectsare paronychia, meatitis in males and contact lens
165
problems due to dryness of the eyes.Abnormalities of serum lipids and liver
function tests should be excluded before treatmentand sought after 4, and
perhaps 8, weeks of treatment.

A62-year-old female is referred to dermatology due to a lesion over her shin. It


initially started as a small red papule which later became a deep, red,necrotic ulcers
with a violaceous border. What is the likely diagnosis?A. Necrobiosis lipoidica
diabeticorumB. SyphilisC. Erythema nodosumD. Pretibial myxoedemaE. Pyoderma
gangrenosum
E. Pyoderma gangrenosumThis is a classic description of pyoderma gangrenosum

The differential diagnosis of shin lesions includes the following


conditions:•erythema nodosum•pretibial myxoedema•pyoderma
gangrenosum•necrobiosis lipoidica diabeticorumBelow are the characteristic
features:Erythema nodosum•symmetrical, erythematous, tender, nodules which
heal without scarring• m o s t c o m m o n c a u s e s a r e s t r e p t o c o c c a l
i n f e c t i o n s , s a r c o i d o s i s , inflammatory bowel disease and drugs (penicillins,
sulphonamides, oralcontraceptive pill)Pretibial myxoedema•symmetrical,
erythematous lesions seen in Graves' disease•shiny, orange peel skinPyoderma
gangrenosum•initially small red papule•later deep, red, necrotic ulcers with a
violaceous border •idiopathic in 50%, may also be seen in inflammatory
bowel disease,connective tissue disorders and myeloproliferative
disorders Necrobiosis lipoidica diabeticorum•shiny, painless areas of yellow/red skin
typically on the shin of diabetics•often associated with telangiectasia

Question 2 of 121
A30-year-old female in her third trimester of pregnancy mentions during
anantenatal appointment that she has noticed an itchy rash around her umbilicus.
This is her second pregnancy and she had no similar problems inher first
pregnancy.Examination reveals blistering lesions in the peri-umbilical region and
onher arms. What is the likely diagnosis?A. Seborrhoeic dermatitisB. PompholyxC.
Polymorphic eruption of pregnancyD. Lichen planusE. Pemphigoid gestationis-------------
----------------------------------E. Pemphigoid gestationisPolymorphic eruption of pregnancy is
not associated with blisteringPemphigoid gestationis is the correct answer.
Polymorphic eruption of pregnancy is not associated with blistering

Question 2 of 121
A30-year-old female in her third trimester of pregnancy mentions during
anantenatal appointment that she has noticed an itchy rash around her umbilicus.
This is her second pregnancy and she had no similar problems inher first
pregnancy.Examination reveals blistering lesions in the peri-umbilical region and

166
onher arms. What is the likely diagnosis?A. Seborrhoeic dermatitisB. PompholyxC.
Polymorphic eruption of pregnancyD. Lichen planusE. Pemphigoid gestationis

E. Pemphigoid gestationisPolymorphic eruption of pregnancy is not associated with


blisteringPemphigoid gestationis is the correct answer. Polymorphic eruption
of pregnancy is not associated with blistering

Polymorphic eruption of pregnancy•pruritic condition associated with last


trimester •lesions often first appear in abdominal striae•management depends on
severity: emollients, mild potency topical steroidsand oral steroids may be used
Pemphigoid gestationis•pruritic blistering lesions•often develop in peri-
umbilical region, later spreading to the trunk, back, buttocks and arms•usually
presents 2nd or 3rd trimester and is rarely seen in the first pregnancy•oral
corticosteroids are usually required

Question 3 of 121
A25-year-old man presents with a widespread rash over his body. The torsoand
limbs are covered with multiple erythematous lesions less than 1 cm indiameter
which in parts are covered by a fine scale. You note that two weeksearlier he was
seen with to a sore throat when it was noted that he hadexudative tonsillitis. Other
than a history of asthma he is normally fit andwell. What is the most likely diagnosis?A.
Pityriasis RoseaB. Pityriasis versicolor C. SyphilisD. Discoid eczemaE. Guttate psoriasis

Guttate psoriasis is more common in children and adolescents. It may be precipitated by a


streptococcal infection 2-4 weeks prior to the lesionsappearing
Guttate psoriasis is more common in children and adolescents. It may be precipitated by a
streptococcal infection 2-4 weeks prior to the lesionsappearingFeatures•tear drop
papules on the trunk and limbsManagement•most cases resolve spontaneously
within 2-3 months•there is no firm evidence to support the use of antibiotics to
eradicatestreptococcal infection•topical agents as per
psoriasis•UVB phototherapy•tonsillectomy may be necessary with
recurrent episodes

Dermoscopic features suggestive of malignant


melanoma include:1 Presence of 2 or more colors within the lesion2 Blue-
whitish veil3 Asymmetric radial streaming
Abrupt interruption of pigment network in the periphery5 All of these answers are c
orrect
All of these answers are correctAsymmetry, multicomponent pattern, blue-whitish
veil, parallel-ridge pattern, atypical pigment network,uneven radial streaming,
localized irregular and diffuse pigmentation, irregularly distributed globules,
andregression structures are all dermoscopic features suggestive of malignant
melanoma

All of the following are true of thick melanomas (>3 mm)


except:1 Predominantly nodular type2 Women affected more than men3 Predilectio
167
n for the head and neck4 Mainly in older patients (>50 years)5 Associated with fewer
nevi
Women affected more than menAccording to a study performed by Chamberlain,
et.al., thick melanomas (> 3 mm) were predominantlynodular in type. They occurred
in older men, mostly on the head and neck and were associated with fewernevi

Capsaicin causes which of the


following?1 Decreased firing of nerve fibers transmitting pain2 Increased substance
P release3 Decreased substance P release4 Blocking the influx of sodium ions into th
e cell5 Creating a barrier between nerves and their stimuli
I ncreased substance P releaseRepeated substance P release leads to eventual
depletion of this agent which causes pain transmission.

The majority of phytophotodermatitis is caused by plants from which family?


Apiaceae
Liliaceae
Alliaceae
Compositae
None of the above
The Apiaceae family (formerly Umbelliferae) includes parsley, celery, parsnip,
hogweed, and fennel, andcauses the majority of phytophotodermatitis

The majority of naturally-occurring cases of anthrax:1 Are extra-


cutaneous2 Are acquired through ingestion of spores3 Are oropharyngeal4 Are pulm
onary5 Are cutaneous
The majority (95%) of naturally-occurring cases of anthrax are of the cutaneous
form, acquired fromdirect contact with the carcasses of dead sheep, cows, goats,
and horses. Naturally-occurring pulmonary,gastrointestinal (acquired by ingestion of
spores) and oropharyngeal infection with anthrax is lesscommon.

Which of the following concerning leg ulcers is correct?


1) Diuretics have been shown to be of benefit in the treatment of associated
oedema.
2) In diabetic ulcers the dressing should be left in situ for no more than 1 week
3) Large gravitational ulcers are always painful
4) Treating superficial infection with antibiotics has been shown to be beneficial
5) ulcers caused by arterial disease are typically treated by compression bandaging
Answers-2
Diuretics are only of value if ulceration is associated with oedema. Gravitational
ulcers are not usually painful. If there is no obvious features of surrounding cellulitis,
antibiotic therapy is usually unnecessary and has not been shown to improve healing
in superficial infection which is common in ulceration.

168
Exposure to sunlight aggravates:
1) Pellagra
2) Acne vulgaris
3) Psoriasis
4) Acute Intermittent porphyria
5) Xeroderma Elasticum
Answers-1
Exacerbation or localization of other dermatoses is characteristic of pellagra,
Hartnup's disease, lupus erythematosus, Darier's disease, rosacea, scleroderma,
actinic lichen planus, and lymphocytoma.

3- A 22 year old woman complains of haemoptysis, abdominal pains and pyrexia for
a month. She is admitted to hospital and found to be apyrexial and not distressed.
There are numerous crusted, linear lesions on her forearms.
What is the most likely diagnosis?
1) Acute intermittent Porphyria
2) Factitious disorder
3) Systemic lupus erythematosus
4) TB
5) Wegener's granulomatosis
Answers-2
The history is very vague and the patient has no clinical features other than a rash
which sounds typical of dermatitis artifacta.

4- Which of the following concerning Pityriasis rosea is correct?


1) It is due to a fungal infection
2) It is characterised by flat scaly patches
3) It is frequently associated with oro-genital itching
4) May be preceded by intense itching
5) Tends to recur after apparent cure
Answers-2

5- A young woman has acne and is taking oral medication. She develops polyarthritis
and raised liver enzyme tests. Investigations show
AST 95
ALT 170
bilirubin 16
antinuclear antibodies strongly positive at 1/640, negative at 1/20
Which of the following drugs is she most likely to have been prescribed?
1) erythromycin
2) isotretinoin
3) minocycline
4) oxytetracycline
5) trimethoprim
Answers-3

169
6- Which of the following suggests a diagnosis of molluscum contagiosum rather
than chickenpox?
1) Presence of macules and papules
2) Absence of erythema surrounding lesions
3) Lesions disappearing within a month
4) Presence of pruritis
5) Positive contact history
Answers-2

7- An 18-year-old woman presents with red, tender lumps on her shins and arthralgia. Chest
X-ray shows bilateral hilar lymphadenopathy and clear lung fields. A clinical diagnosis of
sarcoidosis is made.
Which one of the following is the most appropriate management plan?
1) 24 hour urinary calcium measurement
2) follow up appointment with chest X-ray in three months
3) mediastinoscopy and lymph node biopsy
4) skin biopsy
5) thoracic CT scan
Answers-2

8- Which of the following is true regarding diabetic foot ulceration?


1) Autonomic neuropathy results in increased resting blood flow
2) Callus formation at pressure areas is an important predictor of ulceration
3) Plantar ulceration is most commonly due to atherosclerosis.
4) Skin infection is the most common initiating event in ulceration.
5) Radiography can readily distinguish between Charcot’s joint and osteomyelitis.
Answers-2

9- A 75-year-old female presents with chronic leg ulceration which is a consequence of


venous insufficiency.
Which one of the following is the most appropriate management?
1) Appropriate systemic antibiotic in preparation for skin grafting
2) Compression bandaging
3) Improve the venous return by limb elevation
4) Skin biopsy to exclude neoplasm
5) Vein surgery Exclusion of neoplasm by skin biopsy
Answers-2

10- A 68-year-old woman presents with a 2 month history of a widespread pruritic rash.
Examination reveals widespread erythema with several small blisters containing straw-
coloured fluid and one or two larger serosanguinous blisters.
What is the most likely diagnosis?
1) bullous impetigo
2) bullous pemphigoid
3) Insect bite
4) scabies
5) urticarial vasculitis
Answers-2
The causes of a vesicular eruption are rather few but include pemphigoid, Erythema
Multiforme and Herpes. This is a classic description of pemphigoid.

11- A 50-year-old man presented in the summer complaining of itching and blistering of his
hands and forehead. On examination there were small areas of excoriation on the backs of
his hands. What is the most likely diagnosis?
1) dermatitis herpetiformis
2) lupus erythematosus
3) pemphigoid
4) pemphigus

170
5) porphyria cutanea tarda
Answers-5
The distribution of the lesions suggests a photosensitive element. Both lupus erythematosus
and PCT are associated with a photosensitive elements, however
this is more typical of PCT. PCT causes blistering of the hands and the forehead which
usually heal with small scar and milia formation. It is also associated with an
excessive alcohol intake.

12- A 40-year-old female presents with a six month history of pruritic papules, vesicles and
excoriations on the elbows, knees, buttocks and scalp. Her GP has prescribed topical
betamethasone therapy which has been unhelpful. What is the most likely diagnosis?
1) Atopic dermatitis (Eczema)
2) Dermatitis herpetiformis
3) Hennoch-Schonlein purpura
4) Psoriasis
5) Scabies
Answers-2
Answer-1

13- A 26-year-old man is noted to have cyanosis of the lower limbs and clubbing of the toes
but not the fingers. Which of the following statements is true?
1) He has Eisenmenger's syndrome.
2) He has coarctation of the aorta.
3) He is likely to have a loud continuous 'machinery' murmur below the left clavicle.
4) He is likely to need urgent surgery.
5) He has had a Blalock shunt operation.

14- A previously fit, 30-year-old female presents with a four day history of intractable pruritus
and urticaria. What is the most appropriate initial
management?
1) Chlorpheniramine
2) Prednisolone
3) Ranitidine
4) Topical hydrocortisone
5) topical mepyramine
Answers-1

15- Which of the following is a true of cutaneous anthrax?


1) causes a black eschar which overlies pus
2) lesions are usually painful and tender
3) lesions are associated with marked oedema
4) Mortality is approximately 20% despite antibiotic therapy
5) Is very likely to occur in subjects exposed to anthrax spores
Answers-3
Anthrax is caused by B Ahtracis a gram positive rod. Cutaneous anthrax is associated with a
black eschar without pus, tend to be painless and have widespread oedema. Without
antibiotics mortality is of the order of 20%, but with antibiotics, mortality is low, which
contrasts with pulmonary anthrax.

16- A 22 year old male presents with generalised pruritus of six weeks duration. Examination
reveals little except for erythematous papules between the fingers. Which ofthe following
therapies would be most appropriate for this patient?
1) Astemizole
2) Calamine lotion
3) Chlorpromazine
4) Ciprofloxacin
5) Permethrin cream
Answers-5
This patient has scabies, a highly contagious disease caused by the mite, Sarcoptes Scabiei.
171
Appropriate treatment includes Permethrin cream topical Benzyl Benzoate or malathion.

17- A 40-year-old man presented with pityriasis versicolor. What is the most appropriate
treatment?
1) methotrexate
2) oral terbinafine
3) psoralen with ultraviolet light (PUVA) therapy
4) topical selenium sulphide
5) phototherapy with ultraviolet light (UVB)
Answers-4
Pityriasis versicolor (also called tinea versicolor) is a skin lesion cause by a fungus called
Malassezia furfur. The treatment is topical selenium sulphide. Oral
Itraconazole is also effective.

18- Which of the following is a feature of hereditary haemorrhagic telangiectasia?


1) a good response to oestrogen therapy
2) cerebral arteriovenous malformations
3) GI haemorrhage as the usual presenting feature
4) telangiectasia of the mucous membranes, but not the skin
5) tendency of lesions to become less obvious with age
Answers-2
In hereditary haemorrhagic telangiectasia there may also be pulmonary AV
malformations.Epistaxis, not GI haemorrhage, is the usual presenting feature. Lesions
become more obvious with age and affect mucuous membranes as well as skin. Oestrogen
therapy is sometimes advocated but the effect, if any, is small.

19- Concerning Neurofibromatosis Type 1 (NF1), which one of the following statements is
true?
1) Bilateral acoustic neuromas are common
2) Clinical severity in individuals is similar in a given family
3) New mutations occur rarely
4) Pigmented spots on the iris are a characteristic feature
5) The diagnosis is likely if two café-au-lait patches are present
Answers-4

20- A 30 year old woman presents with a skin rash. On applying pressure to an unaffected
area of skin it was relatively easy to induce trauma.
Increased fragility of the skin is characteristic of which of the following conditions?
1) acute intermittent porphyria
2) epidermolysis bullosa
3) neurofibromatosis
4) pseudo-xanthoma elasticum
5) tuberous sclerosis
Answers-2

21- A 50-year-old man presented in the summer complaining of itching and blistering of his
hands and forehead. On examination there were small areas of excoriation on the backs of
his hands. What is the most likely diagnosis?
1) dermatitis herpetiformis
2) lupus erythematosus
3) pemphigoid
4) pemphigus
5) porphyria cutanea tarda
Answers-5

22- What is the most common presenting feature of porphyria cutanea tarda?
1) acute blistering crises affecting the trunk and limbs
2) acute redness and swelling following sun exposure
3) erythroderma
172
4) generalised hypertrichosis
5) skin fragility and blistering affecting the hands, face and scalp
Answers-2
Development of vesicles and bullae on sun exposed areas like the face, dorsa of the hand,
feet, forearm and legs is the commonest feature.

23- A 24 year old female presents with vague frontal headaches and visual disturbance. She
has a past history of acne for which she is receiving treatment. Examination reveals her to be
obese with a blood pressure of 110/70 mmHg. There is absence of the central retinal vein
pulsation on fundoscopic examination. Which of the following drugs account for these
findings?
1) Isotretinoin
2) Ampicillin
3) Topical tetracycline
4) Dianette
5) Erythromycin
Answers-4
Dianette, like any oral contraceptive may be associated with Benign Intracranial
Hypertension. Topical tetracycline is not associated with BIH. Rarely BIH has been
associated with isotretinoin but usually in combination with a tetracycline.

24- A 74-year-old man with a thirty year history of psoriasis presented with generalised
erythroderma of 3 days duration. Examination reveals him to be shivering but otherwise is
well. He was treated as an inpatient with emollients and attention to fluid replacement and
temperature control but failed to improve after five days. What is the most appropriate next
treatment?
1) Oral hydroxychloroquine
2) Oral methotrexate
3) Oral prednisolone
4) Topical coal tar
5) Topical dithranol
Answers-2

25- A 43 year old woman with atopic dermatitis (atopic eczema) presented with an acute
generalized exacerbation of her disease. She was admitted to hospital but failed to improve
with emollients, topical betamethasone-17-valerate and oral antihistamine. Which one of the
following drugs is the most appropriate treatment?
1) Acitretin
2) Amoxycillin
3) Ciclosporin
4) Colchicine
5) Dapsone
Answers-3

26- Which of the following is a recognised feature of psoriasis?


1) Angular stomatitis
2) Iridocyclitis
3) Koebner Phenomenon
4) Loss of hair
5) Response to chloroquine
Answers-3

27- Which is true regarding Eczema Herpeticum?


1) Is invariably fatal if untreated.
2) Usually has an indolent onset.
3) Only a single crop of vesicles usually appear.
4) Is typically associated with a high fever for over a week.
5) Is more severe in reactivation disease.
Answers-4
173
28- A 16-year-old boy presentes with erythema nodusum. Which of the following should be
considered?
1) Reiter's Disease
2) Ulcerative colitis
3) Cytomegalovirus infection
4) Toxoplasmosis
5) Kawasaki Disease
Answers-2

29- A 58-year-old man has a history of obesity, gastro-oesophageal reflux disease, low back
pain and IHD. He presents with large, itchy wheals over the trunk and limbs and a sensation
of tightness in the throat. Which one of the following drugs is the most likely to have triggered
this skin eruption?
1) aspirin
2) GTN (nitrate) spray
3) omeprazole
4) paracetamol
5) simvastatin
Answers-1
In hypersensitive patients aspirin can cause angioedema, bronchospasm and urticaria(skin
rashes).

30- Which statement regarding tinea capitis is correct?


1) It is most commonly caused by the fungus microsporum canis.
2) Its presence should suggest immunological deficiency.
3) It often results in permanent alopecia.
4) It causes patches that fluoresce dull green under Wood's lamp.
5) It is effectively treated with topical Nystatin ointment.
Answers-4

31- Which of the following may be responsible for an acute relapse of Systemic Lupus
Erythematosus in a 38 year old female?
1) hydralazine therapy
2) Pregnancy
3) Progesterone only contraceptive pill
4) Salmeterol therapy
5) Winter holiday in Lapland
Answers-2

32- A 38 year old female presents with red target lesions confined to the hands and is
diagnosed with erythema multiforme. Which of the following could be the cause?
1) Cytomegalovirus infection
2) Ureaplasma urealyticum
3) Group B Streptococci
4) Langerhan's cells histiocytosis
5) Penicillin V
Answers-5

174
Key answers

Q1. b Q30. e Q59. d Q88. c


Q2. c Q31. b Q60. c Q89. d
Q3. b Q32. c Q61. c Q90. a
Q4. b Q33. d Q62. c Q91. e
Q5. e Q34. c Q63. c Q92. c
Q6. e Q35. c Q64. d Q93. c
Q7. c Q36. e Q65. c Q94. b
175
Q8. c Q37. c Q66. c Q95. d
Q9. d Q38. d Q67. d Q96. e
Q10. e Q39. d Q68. e Q97. a
Q11. e Q40. b Q69. a Q98. c
Q12. c Q41. e Q70. a Q99. b
Q13. c Q42. e Q71. a Q100. b
Q14. e Q43. e Q72. d Q101. e
Q15. c Q44. d Q73. e Q102. b
Q16. e Q45. b Q74. e Q103. a
Q17. c Q46. a Q75. e Q104. a
Q18. d Q47. b Q76. c Q105. d
Q19. e Q48. c Q77. b Q106. d
Q20. e Q49. c Q78. a Q107. a
Q21. c Q50. d Q79. d Q108. e
Q22. b Q51. b Q80. d Q109. d
Q23. b Q52. a Q81. c Q110. e
Q24. b Q53. e Q82. b Q111. d
Q25. e Q54. e Q83. e Q112. a
Q26. d Q55. e Q84. a Q113. a
Q27. c Q56. d Q85. c Q114. e
Q28. d Q57. d Q86. e Q115.
Q29. e Q58. e Q87. d Q116. e

Q 117. c Q146. e Q175. d Q204. c


Q 118. b Q147.c Q176. e Q205. d
Q119. d Q148.b Q177. e Q206. b
Q120. c Q149.a Q178. e Q207. c
Q121.d Q150.e Q179. b Q208. b
Q122.b Q151.e Q180.c Q209. d
Q123.e Q152.c Q181. d Q210. c
Q124.a Q153.e Q182. a Q211. b
Q125.b Q154.e Q183. c Q212. c
Q126.b Q155.e Q184. d Q213. c
Q127.d Q156.b Q185. e Q214. e
Q128.a Q157.d Q186.a Q215. c
Q129. e Q158.c Q187.a Q216. b
Q130. Q159. Q188 Q217.
176
Q131.a Q160.b Q189.d Q218. b
Q132. c Q161.e Q190. d Q219. d
Q133. d Q162.a Q191.e Q220. e
Q134. b Q163.b Q192.c Q221. e
Q135. c Q164.c Q193.c Q222. d
Q136.c Q165.a Q194.b Q223. e
Q137.d Q166.c Q195.b Q224. c
Q138. e Q167.a Q196.b Q225. b
Q139. b Q168.e Q197.b Q226. e
Q140. b Q169.b Q198.d Q227. c
Q141. c Q170.a Q199.d Q228. e
Q142. e Q171.a Q200.c Q229. a
Q143. b Q172.b Q201.b Q230. e
Q144.a Q173.c Q202.c Q231. e
Q145.a Q174.e Q203.a Q232. a

Q233. b Q263. d Q293. d


Q234. b Q264. c Q294. c
Q235. c Q265. d Q295. e
Q236. e Q266. d Q296. e
Q237.d Q267. d Q297. c
Q238.b Q268. c Q298. e
Q239. d Q269. b Q299. c
Q240. d Q270. d Q300. b
Q241. c Q271. b Q301. b
Q242. c Q272. a Q302. b
Q243. a Q273. e Q303. a
Q244. a Q274. b Q304.
Q245. b Q275. e Q305. d
Q246. c Q276. b Q306.
Q247. c Q277. d Q307.
Q248. d Q278. Q308.
Q249. c Q279. C Q309.
Q250. b Q280. e
Q251. d Q281. e
Q252. c Q282. c
Q253. e Q283. d
Q254. b Q284. e
Q255. a Q285. b
Q256. c Q286. a
Q257. d Q287. d
Q258. b Q288. b
Q259. e Q289. e
Q260. e Q290. a
177
Q261. b Q291. d
Q262. b Q292.

Melanocytic nevus

Melanocytic nevus

Herpes simplex infection

178
Herpes zoster

Viral warts (filiform type) (By courtesy of Dr. Shafi)

Impetigo

179
Peri-oral dermatitis

Acne vulgaris

Acne vulgaris

180
Acne (nodulocystic type)

Acne keloidalis nuchae

Acne excoree

181
Infected eczema

Atopic dermatitis

Squamous cell carcinoma

182
Squamous cell carcinoma

Tinea pedis

Tinea corporis

183
Erythrasma

Folliculitis

Exfoliative dermatitis

184
Contact dermatitis (house wife dermatitis)

Lichen planus

Lichen planus

185
Herald patch of pityriasis rosea

Pityriasis rosea

Psoriasis vulgaris

186
Psoriasis vulgaris

Psoriasis vulgaris

Psoriasis vulgaris

187
Psoriasis

Erythrodermic psoriasis

Pubic lice

188
Genital warts (condyloma accuminatum)

Cutaneous leishmaniasis

Cutaneous leishmaniasis (By courtesy of Dr. Shafi)

189
Cutaneous leishmaniasis (By courtesy of Dr. Shafi)

Leishmania parasite inside the tissue macrophage

Tinea ungium (By courtesy of Dr. Shafi)

190
Tinea capitis (inflammatory type)

Tinea corporis (By courtesy of Dr. Shafi)

Tinea corporis (By courtesy of Dr. Shafi)

191
Pityriasis versicolor

Candidal intertrigo

Neurofibromatosis (multiple neurofibroma)

192
Xeroderma pigmentosum ‫اللهم اغفر له وارحمه‬

Periungual fibromas (tuberous Sclerosis)

Trichotillomania (By courtesy of Dr. Shafi)

193
Traction alopecia (By courtesy of Dr. Shafi)

Alopecia areata (ophiasis)

Lipoma

194
REFERENCES:

1. ROOK / WILKINSON / EBLING (Textbook of Dermatology)


2. ANDREWS (Diseases of the Skin) Clinical Dermatology
3. MOSCHELLA and HURLEY (Dermatology)
4. W. MITCHELL SAMS, JR / PETER J LYNCH (Principles and Practice of
Dermatology)
5. ANTHONY DU VIVIER (Atlas of Clinical Dermatology)
6. THOMAS P HABIF ( Clinical Dermatology)

‫ عـ ــل سـ ـ ـ ـ ــليم األشهب‬/ ‫د‬

‫ كلية الطب – جامعة الم ــرقب‬/ ‫أستاد مشارك‬

Dr. Ali Saleim Lashhab


Associated professor
Medical School, University of Al-Mergib

E mail; aslashhab2007@yahoo.com

195
‫عل األشهب‬
‫نتمن للجميع التوفيق والنجاح ‪ ............................‬دانيا ي‬

‫‪196‬‬
Q340. Azelaic acid is the only medication listed which falls under category B. The
others listed are category C, except for tazarotene, which is category X.

Q341. Sarcoidosis is granulomatous autoimmune condition characterized by the


formation of non-caseating granulomas, which may affect any organ system.
Lacrimal gland involvement occurs in 15-28% of patients.

Q403. Polymorphous light eruption is the most common photo-dermatosis. An


idiopathic disease usually appears in the first three decades. Pathogenesis is unclear
but it may be related to a type IV hypersensitivity reaction. Most lesions are
erythematous pruritic papules. The plaque form, vesicles and an eczematous
dermatitis are rare. It may improve as the summer progresses. It may occur through
window glass, which filters out UVB.

Q404. Ultraviolet C does not reach the earth's surface. It is filtered out by the ozone
layer. It extends from 200-290 nm.

Q405. Skin type I always burns and never tans


Skin type II usually burns and rarely tans
Skin type III rarely burns and usually tans
Skin type IV never burns and always tans
Skin types V,VI are highly pigmented individuals

Q420. Doxycycline is the tetracycline derivative most likely to cause photosensitivity.

Q421. Chlorpromazine is associated with blue-gray pigmentation on sun-exposed


areas and is not associated with photo-onycholysis. Quinolones, tetracyclines,
psoralens and quinine can cause photo-onycholysis.

Q427. Psoralens in certain plants, fruits, and vegetables can cause phyto-photo-
dermatitis. The most common ones are; limes, figs, parsley, parsnip, bergamot
oranges, and celery.

Q431. UVB in natural sunlight is the main contributor to erythema. UVB erythema
reaches a maximum in 6-24 hours. UVA accounts for 15-20% of sunlight erythema
despite that there is much more UVA than UVB in sunlight.

Q432. Solar urticaria is an idiopathic, type I photosensitivity disorder. Mediator


release during widespread whealing may result in headache, nausea, wheezing,
faintness, and syncope. It usually lasts for many years. Treatment includes sun
avoidance and protection, and H-1 antihistamines may be of partial benefit.

Q434. UVA can be divided into UVA II (320-340 nm) and UVA I (340-400 nm)

197
Q436. Polymorphous light eruption is the most common photo-dermatosis. It is a
idiopathic disease that usually appears in the first three decades of life and is more
common in fair-skinned females. The pathogenesis is unclear, but is believed to be
related to a type IV hypersensitivity reaction. Most lesions are erythematous pruritic
papules, with the plaque form being less common. Lesions appear symmetrically on
exposed areas after a delay of several hours to several days. Patients with mild disease
are treated with sun avoidance and a broad-spectrum sunscreen. In more severe
cases, antimalarials , and a short course of prednisone can be helpful

Q437. Solar urticaria is an idiopathic, type I photosensitivity disorder. Rare cases have
been associated with erythropoietic protoporphyria (EPP) and lupus erythematosus
(LE). Blood tests for LE and appropriate screening for EPP should be performed
including ANA, anti Ro/La, urine and stool porphyrins.

Q438. Psoriasis is improved by phototherapy, especially at the wavelengths of 311-


312 nm in most cases.

Q439. Amiodarone, chlorpromazine and tricyclic antidepressants all are capable of


causing blue-gray pigmentation on sun-exposed areas without involvement of the
sclerae, lunulae of the nails or mucous membranes

Q444 UVB converts 7-dehydrocholesterol in the skin to previtamin D3, which then
thermally isomerizes to form vitamin D3. It is hydroxylated in the liver and then in the
kidney to form 25-hydroxyvitamin D and 1,25-dihydroxyvitamin D, respectively

Q446. The UVB band extends from 290 to 320 nm. The UVB spectrum is recognized as
the primary cause of sunburn, skin cancer, and other harmful effects on human skin.
The UVA band extends from 320 to 400 nm. This spectrum is further subdivided into
UVA-2 (320 to 340 nm) and UVA-1 (340 to 400 nm).The UVA spectrum is recognized
as a cause of immediate and delayed tanning reaction of skin, and several other
effects including photoaging, skin photosensitization, and immunosuppression. UVC
radiation comprises wavelengths shorter than 290 nm (from 200 to 290 nm). Notably,
window glass filters-out ultraviolet wavelengths shorter than 320 nm, so both UVB
and UVC are effectively filtered by car window glass. UVB radiation is more intense
during summer months compared to winter months and peaks during midday hours.
It has been postulated that physical factors such as high temperature, high humidity,
and wind can all increase susceptibility to UV-induced carcinogenesis

Q447. Furosemide (Lasix) is not a cause of lichenoid drug reactions, but can cause
pseudoporphyria. The others listed are causes of lichenoid drug reactions. Others are
antimalarials, demethylchlortetracycline and quinine.

198
Q449. Actinic prurigo is an idiopathic photosensitivity disorder. Lesions are excoriated
papules and nodules that begin in childhood and remit in puberty. They can last for
several months and may occur on non-sun-exposed areas. Thalidomide has been very
effective for treating the majority of patients with actinic prurigo

Q452. Hours to days after exposure. The history of a delay of several hours to several
days after exposure is important to the diagnosis of polymorphic light eruption.

Q453. Patients with chronic actinic dermatitis are usually middle-aged to elderly males
who present with a chronic eczematous dermatitis in a photo-distribution without
history of current exposure to a photosensitizer.

Q455. UVR 290-320nm (UVB range) has been shown to be most efficient in inducing
neoplasia in mice. Long-wave UVA, when added to UVB may accelerate
carcinogenesis.

Q456. Most patients have mild disease that can be treated by sun avoidance, especially
between 11 am and 3pm; a broad spectrum sunscreen and clothing with a tight weave.
In more severe cases, UV hardening, antimalarials or prednisone can be used.
Cyclophosphamide is not used in PMLE.

Q457. Pellagra is characterized by the triad of diarrhea, dermatitis and dementia. The
dermatitis begins as a burning erythema in sun-exposed areas. There may be bullae
and erosions. This is followed by a dry, brittle, scaling and hyperpigmented phase.
Pellagra is due to a deficiency of niacin and tryptophan.

Q458. A phototoxic reaction appears as an exaggerated sunburn with erythema and


sometimes blistering resolving with hyperpigmentation. It is a non-immunologic
reaction that could occur in all individuals given enough of the chemical and enough
UVR. It can occur on the first exposure to the chemical and the UVR.

Q464. UVR causes an increase of circulating cytokines (IL-1, IL-6, TNF-alpha)

Q465. UVA light is found between 320 and 40 nm and is broken up into UVAI (340-
400nm) and UVAII (320-340nm). UVB light is found between 290 and 320nm. UVA
radiation is 100 times greater than UVB during midday hours and sunlight early in the
morning and late in the day contains relatively more UVA. UVB radiation is 1000 times
more erythrogenic than UVA. Cloud cover is a poor UV absorber

Q468. Photography solutions often contain para-pheneylene-diamine, or PPD.


Mercaptobenzothiazole is a rubber accelerator found in shoes, rodenticides, and
insecticides, and is the most common allergen of healthcare workers. Lanolin is a
moisturizer found in cosmetics. Epoxy resin is found in glues and marine cement.
Colophony is found in pine rosin, violins, soaps, and adhesives.

Q470. Talcum powder application causes fiberglass spicules to slide off skin
199
Q471. Leiner's disease is associated with deficient C5 and possibly C3. Babies with this
disease are prone to diarrhea, infections (sepsis), anemia, and a generalized
seborrheic dermatitis-like rash.
Q472. Pemphigus foliaceus is a form of pemphigus with superficial blisters. The
autoantibodies are against Desmoglein-1. Medications associated with the
exacerbation of pemphigus foliaceus is sulfhydryl groups such as captopril,
penicillamine and piroxicam.
Q474. Nickel is the most common contact allergen, found in costume jewelry, alloys,
pigments, scissors, razors and many other metal coated objects. Bacitracin and
neomycin are commonly used topical antibiotic agents that many are allergic to.
Neomycin is the most common topical antibiotic allergen found in testing.
Quaternium-15 is a formaldehyde-releasing preservative found in many topical
products. It can cross react with formaldehyde, but not all patients will react to
formaldehyde on testing.

Q476. Dermatitis herpetiformis is an uncommon chronic, pruritic papulo-vesicular


dermatitis occurring most commonly in young to middle-aged adults. Common sites
of predilection include the buttocks, elbows, knees, scapula and scalp. Typical
histologic features include accumulation of neutrophils at the tips of dermal papillae,
sometimes admixed with eosinophils. Direct immunofloresence (DIF) reveals granular
deposits of IgA within the dermal papillae. As the name implies, linear IgA disease is
characterized by linear IgA deposition along the basement membrane zone with DIF
in 100% of cases. The pattern of direct immunoflorescence in bullous pemphigoid is
linear C3 deposition at the dermoepidermal junction in nearly 100% of cases and IgG
in 65-95% of cases.

Q477. Gold is a known to cause a delayed patch test reading. Patients allergic to gold
often also react to nickel and cobalt. Bacitracin is a frequent contact allergen that
often coexists with an allergy to neomycin. Rosin (colophony) is found in adhesive
tape, cosmetics, glossy papers and chewing gum.

Q479. Methyl methacrylate is found in synthetic resins, dentures, artificial, nail


adhesives, and acrylic bone cement. This allergen may penetrate through the gloves
to the fingertips and most often affects the first three fingers. In addition to dermatitis,
it also causes a peripheral neuropathy

Q480. All of the above have been associated with para-neoplastic pemphigus with
non-Hodgkin's lymphoma being the most common. Castlemans's disease is most
common in children with para-neoplastic pemphigus

Q481. Burning secondary to capsaicin can be relieved by vinegar (acetic acid 5%) as
the capsaicin is soluble in vinegar (but not water)

Q482. Paraneoplastic pemphigus can be caused by lymphoma as well as other


malignancies. It can also be secondary to benign thymomas and Castleman's disease.

200
Paraneoplastic pemphigus is associated with various benign as well as malignant
internal tumors, with the most common being non-Hodgkin's lymphoma

Q484. Senear-Usher syndrome, also known as pemphigus erythematosus, is thought


to be an overlap between lupus erythematosus and pemphigus foliaceus. The
histology resembles that of pemphigus foliaceus with linear IgG at the
dermoepidermal junction. 80% of patients have a positive lupus band.

Q488. In bullous pemphigoid, the antigenic targets are believed to be BPAg1 and
BPAg2-.These proteins are located in the hemidesmosome. Direct immunofluorescent
studies reveal linear basement membrane of C3 in approximately 95% of patients, and
IgG4 in approximately 80%. Linear IgA is found in linear IgA and chronic bullous disease
of childhood. Granular IgA and C3 in the dermal papillae is found in dermatitis
herpetiformis. Intercellular IgG4 throughout the epidermis is found in pemphigus
vulgaris.

Q489. Gluten-sensitive enteropathy or Celiac disease is demonstrated on small bowel


biopsy of all patients with dermatitis herpetiformis. However, most of these patients
do not demonstrate symptoms of gastrointestinal disease. Dermatitis herpetiformis
(DH) is not associated with inflammatory bowel disease, autoimmune hepatitis,
herpes labialis, or rheumatoid arthritis

Q490. Junctional EB is an autosomal recessive disorder with a defect in the lamina


lujcida. The Herlitz variant is due to mutations in laminin 5 and clinically presents as
generalized bullae (which are non-scarring), non-healing granulation tissue periorally,
absent nails, dysplastic teeth and enamel defects, anemia, growth retardation and is
often fatal by age 3-4 due to low protein, anemia and infection. The non-Herlitz variant
of Junctional EB presents with bullae especially on the extremities which heal with
atrophic scarring; patient will improve with age and typically have a normal lifespan.
Weber-cockayne is a type of EB simplex and presents with palmo-plantar bullae and
hyperhidrosis. Hyperplastic cockayne-touraine is a form of dominant dystrophic EB
which presents with bullae localized to extremities. Albapapuloid Pasini variant of
dystropohic EB presents with widespread bullae, atrophy, hypopigmented scar-like
white papules on trunk, no milia or scarring, and mild oral involvement

Q491. Propylene glycol is a widely used solvent and humectant found in a variety of
products such as cosmetics, lotions, corticosteroids, antiperspirants, and K-Y jelly.
Lanolin is an emollient, which comes from wool wax and is found in adhesives,
cosmetics, and topical emollients such as aquaphor. Budesonide is a steroid, alpha
tocopherol is topical vitamin E, and triclosan is a topical antibiotic

201
Q496. Penicillamine is the most common cause of drug-induced pemphigus, and the
split is more often sub-corneal (pemphigus foliaceus-like) than supra-basal
(pemphigus vulgaris-like).

Q498. Herpes gestationis (HG) typically occurs in the second or third trimester, and
clinically presents as urticarial papules and plaques around the umbilicus which
progress to involve the rest of the body. HG has been associated with Grave's
disease

Q500. Staphylococcal scalded skin syndrome is caused by Staphylococci group 2,


phage type 71, which produces an exfoliative toxin that cleaves desmoglein 1.
Clinically, this results in a subcorneal separation of keratinocytes, leading to skin
tenderness, erythema, and superficial desquamation of skin

Q501. The most effective drug for dermatitis herpetiformis is dapsone. The dose
varies between 50 and 300 mg daily. Side effects include hemolytic anemia,
leukopenia, methemoglobinemia, and rarely agranulocytosis or peripheral
neuropathy. Sulfapyridine is also a very effective treatment for dermatitis
herpetiformis

Paraneoplastic pemphigus. It is an uncommon blistering disease with a variety of


clinical patterns and target antigens associated with underlying malignancy. It is most
likely associated with lymphoma, leukemia, Castleman tumor or thymoma

Q511. The Asboe-Hansen sign as applies to blister formation refers to lateral


dissection of the blister when pressure is applied directly to the blister. This is also
called, "bulla-spread phenomenon", because pressure on an intact bulla, gently forces
fluid to wander under the skin away from the pressure site.

Q512. Pemphigus erythematosus is also known as Senear-Usher syndrome. It is


an uncommon variant of pemphigus foliaceus with additional features of lupus
erythematosus.

Q514 / Type VII collagen is present in the basement membrane of stratified


squamous epithelia in the anchoring fibrils. It is the target antigen in several
blistering disease including epidermolysis bullosa acquisita, bullous lupus
erythematosus, dominant and recessive dystrophic epidermolysis bullosa.

Q515. Herpes gestationis (HG) typically occurs in the second or third trimester, and
clinically presents as urticarial papules and plaques around the umbilicus which
progress to involve the rest of the body. HG has been associated with Grave's disease.
Hormonal factors influence the disease manifestation. This condition can be seen in
pregnant women, menstruating women, and women taking oral contraceptives.

202
Q516. Azelaic Acid is pregnancy category B. The other choices are in less safe
pregnancy categories. Epinephrine is category C, Benzoyl peroxide is category C, and
Methotrexate and Isotretinoin are category X.

Q520. Many medications are associated with acneiform eruptions, including halogens
(bromide and iodide), androgenic hormones such as testosterone, ACTH,
corticosteroids, isoniazid (INH), lithium, phenytoin, and vitamins B2, B6 and B12.

Q522. Systemic corticosteroids are commonly used in dermatology. Short acting


steroids, cortisone and hydrocortisone, have the greatest mineralocorticoid activity,
while cortisone has the lowest glucocorticoid activity. Intermediate and long-acting
steroids, methylprednisolone, triamcinolone, dexamethasone, and betamethasone,
have virtually no mineralocorticoid activity. Dexamethasone and betamethasone have
the highest glucocorticoid activity

Q524. Isotretinoin, acitretin, and bexarotene are water-soluble, with very little lipid
deposition. Water-soluble retinoids are undetectable in the serum 1 month after
stopping therapy. Etretinate is 50 times more lipophilic than acitretin. It can last
several years in fatty tissues.
Q525. The estolate form of erythromycin has been associated with cholestatic
hepatitis

Q528. A woman should wait one month before trying to conceive after taking
isotretinoin to prevent birth defects. After taking acitretin, a woman should wait three
years before trying to conceive

Q530. Cetirizine is a second-generation H1 antihistamine that is a low sedation


metabolite of hydroxyzine.

Q531. Imiquimod is FDA approved for the treatment of actinic keratosis, superficial
basal carcinomas, and condyloma acuminate. It is not used to treat infiltrative basal
cell carcinomas, squamous cell carcinoma, superficial spreading melanoma, or
psoriasis. Off label uses include common warts, molluscum contagiosum, Bowen
disease (SCCIS), and keloids

Q532. Ketoconazole can produce impotence and gynecomastia by interfering with


androgen synthesis. Ketoconazole inhibits cytochrome (CYP) P450 and most
concerning, can rarely cause fulminant hepatitis. Griseofulvin induces CYP P450, not
inhibits it. Terbinafine is fungicidal along with amphotericin B. Ketoconazole is
fungistatic.

Q533. Regional penetration of topical therapy is ranged from highest penetration to


less penetration as following: Mucous membrane > scrotum > eyelids > face > chest
and back > upper arms and legs > dorsal hands and feet > nails

203
Q537. Oral iron supplements markedly reduce absorption of mycophenolate mofetil
(Cell-Cept). It is recommended that iron be taken four to six hours before, or two
hours after mycophenolate mofetil

Q538. Tetracycline has been associated with onycholysis and photo-onycholysis.


Chloroquine may cause a blue-brown discoloration of the nail bed and retinoids are
associated with paronychia

Q539. Chloramphenicol is the treatment of choice for pregnant patients with Rocky
Mountain Spotted Fever. In non-pregnant patients, the treatment of choice is
Doxycycline.

Q544. Adverse effects from dapsone are both pharmacologic and idiosyncratic and
include hemolytic anemia, methemoglobinemia, agranulocytosis, hypersensitivity
syndrome and neuropathy. Methemoglobinemia is the formation of methemoglobin
in the blood, which has a decreased oxygen-carrying capacity compared with
hemoglobin and can result in cyanosis. Vitamin E (800 IU/day) has been shown to
provide a small amount of protection against methemoglobinemia and hemolysis.

Q547. Drugs that simultaneously inhibit the folate metabolic pathway, such as NSAIDS,
dapsone, or trimethoprim-sulfamethoxazole, can increase hematologic toxicity when
combined with methotrexate

Efalizumab is pregnancy in category C. The other drugs listed are pregnancy category
B

Q554. Azathioprine is a purine analog which is used to treat multiple inflammatory


disorders, including autoimmune bullous disease, vasculitis, and severe dermatitis. A
hypersensitivity syndrome with fever and shock can occur at around 14 days

Q557. This patient presents with a surgical wound infection unresponsive to


cephalexin. This is likely secondary to community acquired MRSA infection.
Doxycycline is the best choice for community acquired MRSA infection. Bactrim, not
listed, is another option. Amoxicillin, Azithromycin, and Ciprofloxacin are not typically
used to treat MRSA infection as resistance is high with these medications. Polymyxin
B is a topical antibiotic with anti-pseudomonas properties.

Q558. Probenicid is co-administered with penicillin to prolong its half-life through


decreased renal tubular secretion when higher blood levels are wanted.

Q560. Clindamycin is particularly effective against anaerobic and gram-positive


organisms, particularly those causing deep tissue infections
204
Q561. The most common side effect of treatment with interferon-alpha is
flu-like symptoms of fever, chills, myalgia, headache and arthralgias. Prophylactic
administration of non steroidal anti-inflammatory medications may alleviate some of
these symptoms.

Q562. Terfenadine is an anti-histamine that was one of the first non-sedating


antihistamines. It is no longer available in the USA because of its propensity to cause
life-threatening cardiac arrhythmias.

Q568. Quinacrine frequently produces a yellow discoloration of the sclera and skin,
especially over the dorsal hands and feet. Minocycline and hydroxychloroquine can
cause bluish-gray hyperpigmentation. Terbinafine and prednisone are not associated
with alteration of cutaneous pigmentation

Q574. Rituximab (brand name Rituxan) is a monoclonal antibody is approved for the
treatment of CD20 non-Hodgkin’s lymphoma. Rituximab is a monoclonal antibody
directed against B lymphocytes which are CD20.

Q576. Rituximab is an anti-CD20 monoclonal antibody. CD20 is a B-cell marker and is


used to treat Non-Hodgkins B-cell lymphoma. It is also approved for the treatment of
rheumatoid arthritis.

Q582. The patient presents with blue-gray discoloration in sun-exposed areas. The
most likely offending agent is amiodarone. Blue-gray discoloration from minocycline
usually occurs on legs. Chloroquine usually causes blue-gray discoloration in the
sclerae, teeth, buccal mucosa, nail beds, and pretibial areas. Quinacrine causes yellow
discoloration of skin and conjunctiva. CLofazimine usually causes a red-brown
discoloration

Q584. Long-term therapy with oral corticosteroids can result in numerous adverse
effects, including elevated risks of glaucoma, cataracts, hypertension, diabetes,
osteoporosis, adrenal axis suppression, and aseptic bone necrosis. Alternate-day
dosing of oral corticosteroids lowers the rate of adrenal axis suppression

Q585. Adverse effects of oral retinoid therapy:


Cutaneous: Xerosis, retinoid dermatitis, photosensitivity, pyogenic granulomas,
stickiness sensation.
Hair: Telogen effluvian and abnormal hair texture
Nails: Fragility with nail softening, paronychia and onycholysis
Occular: Dry eyes w/visual blurriness, Blepharo-conjunctivitis, Photophobia
Oral: Cheilitis and dry mouth
Nasal: Nasal mucosal, dryness, decreased mucous secretion, epistaxis
Musculoskeletal: Arthralgia, myalgia, fatigue and muscle weakness.
Neurological: Headache and mild depression
205
Gastrointestinal: Nausea /Vomiting /Diarrhea and abdominal pain

Q590. Ketoconazole is known to cause gynecomastia and impotence, by interfering


with androgen and glucocorticoid synthesis

Q591. Systemic retinoids like acitretin have been used in the chemoprophylaxis of skin
cancers. Several studies have suggested a beneficial effect of retinoids with lowered
incidences of skin cancer

Q593. Cimetidine and vitamin E have both been known to provide prophylaxis against
methemoglobin formation. G6PD-deficient individuals are at greater risk of
hematologic toxicity from dapsone

Q595 Doxepin, a tricyclic antidepressant, has H1 and H2 antihistamine activity.

Q596. Dapsone is a lipid-soluble sulfone drug that is used widely in dermatology in a


variety of conditions including dermatitis herpetiformis, leprosy, and neutrophilic
dermatosis. Adverse effects from dapsone are both pharmacologic and idiosyncratic
and include hemolytic anemia, methemoglobinemia, agranulocytosis, hypersensitivity
syndrome and neuropathy.

G6PD-deficient individuals are more susceptible to oxidative stresses, including those


from dapsone metabolites, and a baseline G6PD level is recommended prior to
initiation of dapsone therapy. In the event of emergent methemoglobinemia, oral
methylene blue (100-300mg/day) can be used to decrease methemoglobin levels.
However, if the patient is G6PD deficient, this strategy is ineffective

Q597. The most likely diagnosis is neutrophilic eccrine hidradenititis. Cytarabine is the
usual offending agent

Q598. Demeclocycline and doxycycline are the most phototoxic of all the tetracyclines

Q600. IVIG is used to treat several diseases including graft versus host disease,
connective tissue disease, and autoimmune bullous dermatoses. Adverse effects
include infusion reactions (headache, flushing, chills, myalgia, wheezing, tachycardia,
lower back pain, nausea, or hypotension). Anaphylaxis occurs rarely. Disseminated
intravascular coagulation, transient neutropenia, and aseptic meningitis syndrome
has been reported. Cutaneous adverse effects include eczematous eruptions and
alopecia.

Q601. Valacyclovir is a pro-drug of acyclovir, and has enhanced bioavailability and


converts rapidly and completely to acyclovir. Severe and even fatal cases of
thrombotic thrombocytopenia purpura/ hemolytic-uremic syndrome have been
reported in AIDS and transplant patients taking high doses of valacyclovir.

206
Q602. Tetracycline antibiotics are effective against gram positive and negative
organisms, Mycoplasma, Chlamydia, Rickettsia, and others. Doxycycline, which is
excreted by the gastro-intestinal tract, is the only tetracycline for use in patients with
renal failure

Q603. Alternate-day corticosteroid dosing regimens does not decrease the risks of
posterior sub-capsular cataracts, osteoporosis, and possibly osteonecrosis.

Q605. Bexarotene is excreted via hepato-biliary excretion. The others are excreted in
bile and urine

Q611. Glucocorticoids alter the balance of circulating leukocytes, causing an increase


in the number of polymorphonuclear leukocytes and diminishing the numbers of
lymphocytes, eosinophils, and monocytes.

Q613. Triamcinolone, dexamethasone, and betamethasone also have low


mineralocorticoid activity.

Q615. Lichenoid drug reaction and lichen planus are best distinguished clinically.
However, there are clues to lichenoid drug, including parakeratosis and eosinophils.
The presence of parakeratosis has been shown to be more sensitive than eosinophils
in the diagnosis of lichenoid drug reaction

Q616. Three types of ocular adverse effects may develop from antimalarials: corneal
deposits, neuro-muscular eye toxicity and retinopathy. Retinopathy is potentially
irreversible. It is recommended that a patient be evaluated for retinopathy at
baseline, then every 6 months by an ophthalmologist. Testing visual acuity, visual
fields and performing a funduscopic examination are considered acceptable for
screening purposes

Q617. Melanonychia occurring simultaneously on several nails is most likely to be due


to minocycline therapy. Blue-black pigmentation may be present in nails, skin, scars
and sclerae.

Q620. Ivermectin (Stromectol) is an anti-helminthic agent currently FDA-approved for


the treatment of strongyloides and onchocerciasis. Ivermectin is pregnancy category
C. There are virtually no associated drug interactions with oral ivermectin therapy.

Q621. Procollagen 3 has been shown to be a sensitive marker for methotrexate


induced liver fibrosis.

Q622. Neutrophilic eccrine hidradenitis most commonly occurs in the setting of a


patient with acute myelogenous leukemia being treated with cytarabine

207
Q623. Cimetidine and vitamin E can reduces the formation of methemoglobin, thereby
increasing tolerance to dapsone, which can lead to methemoglobinemia. Azathioprine
can cause hepatitis, lympho-proliferative malignancy, infections, and hypersensitivity
syndrome. Cyclosporine can lead to renal dysfunction, hypertension, and gingival
hyperplasia. Rifampin can cause decreased effectiveness of oral contraceptives.

Q626. Isotretinoin, acitretin, and bexarotene are water-soluble, with very little lipid
deposition. Etretinate is 50 times more lipophilic than acitretin, with increased storage
in adipose tissue

Q627. Long term use of oral corticosteroids has been shown to decrease bone mineral
density (BMD). This decrease in BMD leads to an increased risk for fractures. It was
originally thought that Vitamin A analogues such as isotretinoin and acitretin may also
increase the risk of fracture but in a large case-controlled study published in the May
2010 Archives illustrates that risk of fracture is not associated with Vitamin A
analogues. Vitamin A analogues have been associated with hyperostosis

Q628. The tetracycline family of antibiotics are bacteriostatic and act by inhibiting
protein synthesis. Each member of the family may cause photosensitivity, but
demeclocycline is the most photosensitizing.

Q629. SCLE-like reaction are known to occur most likely from glyburide, griseofulvin,
hydrochlorothiazide, penicillamine, piroxicam, and terbinafine.

Q631. Long-term systemic steroid therapy can result in hyperlipidemia, especially


Hypertriglyceridemia. Hyperglycemia, not hypoglycemia may occur with steroid
therapy. Hypokalemic alkalosis may result from long-term steroid use. Uncommonly,
hypocalcemia may develop with resultant tetany.

Q632. Wegener granulomatosis is an idiopathic disorder characterized by necrotizing


granulomatous vasculitis of the upper and lower respiratory tract, glomerulonephritis,
and a variable degree of small-vessel vasculitis. Testing for c-ANCA is 90% sensitive for
the diagnosis when the presentation is classic. Cyclophosphamide has been used very
effectively and now is the usual drug of choice for induction of remission.

Q634. Castellani's paint is a fungicidal and bactericidal agent. It contains boric acid,
resorcinol, acetone, water, industrial methylated spirit, magenta, and phenol

Q636. Rifampin inhibits RNA synthesis by blocking DNA-dependent RNA polymerase.


It is effective in tuberculosis and atypical mycobacterial infections. It is the only drug
that is bactericidal to Mycobacterium leprae.

Q637. Leukovorin is also available as an oral form, and may be administered as a


continual part of methotrexate therapy, instead of folic acid.

208
Q638. First-generation retinoids are tretinoin (all-trans RA) and isotretinoin (13-cis
RA). Second-generation synthetic retinoids are etretinate, which was replaced by its
metabolite acitretin. Third-generation polyaromatic retinoids) include the arotinoids,
tazarotene, adapalene, and bexarotene.
Q639. Erythema Nodosum Leprosum is a type 2 reaction of leprosy. It most commonly
ocurrs in lepromatous leprosy. Clinically, it presents as painful nodules and is
associated with fever, malaise, anorexia, and arthralgias. The treatment of choice is
thalidomide.

Q640. Erythromycin estolate is contraindicated in pregnancy because of the risk of


cholestatic hepatitis. Other forms of erythromycin are safer for use in pregnancy.

Q641. Isotretinoin is absolutely contraindicated in pregnancy and is category X. All


other alternatives are safe in pregnancy.

Q643. Mucous membranes have the highest absorption of topical medicines.


Percutaneous absorption, from highest to lowest, is as follows: mucous membrane >
scrotum > eyelids > face > chest and back > upper extremity > lower extremity > acral

Q646. Erythromycin inhibits the cytochrome P-450 system, which may result in
increased levels of digoxin among many other drugs.

Q647. Many authors consider precipitated sulfur to be the drug of choice for the
treatment of scabies in pregnant women. Permethrin of note is category B in
pregnancy.

Q648. The laboratory abnormalities associated with cyclosporine are magnesium,


increased potassium, and increased uric acid. Renal function and blood pressure
must also be monitored closely in patients using cyclosporine.

Q649. Transient neonatal pustular melanosis typically begins with sterile pustules that
leave a characteristic collarette when ruptured. The lesions heal with hyperpigmented
macules

650. Hypercalcemia is an infrequent complication of subcutaneous fat necrosis of the


newborn. Associated symptoms can include irritability, weight loss, vomiting and
failure to thrive. Repeated serum calcium tests are advised until one month after all
the cutaneous lesions have resolved.

Q651. There is a well-known correlation between a history of upper respiratory


infection secondary to group A beta-hemolytic streptococci and the subsequent
development of guttate psoriasis. The eruption may resolve spontaneously or with
treatment (topical steroids or ultraviolet therapy). Some portion of these patients
eventually develop chronic, plaque-type psoriasis.

209
Q653. The most likely diagnosis is subcutaneous fat necrosis of the newborn.
Hypercalcemia may result and serum calcium levels should be monitored up to 4
weeks after resolution of the skin

Q654. Gianotti-Crosti, or papular acrodermatitis of childhood, is typically nonpruritic


and corticosteroids should be avoided as they may have adverse effect. The
condition typically resolves after 2-3 weeks.

Q655. Jacquet diaper dermatitis is a multifactorial process. Yeast, irritants and


moisture all contribute to the occurrence of this eruption.

Q656. The newborn most likely has aplasia cutis congenita. A skull x-ray would be the
simplest, most cost effective means of identifying any underlying bony abnormality

Q657. This child has Kawasaki's disease, or mucocutaneous lymph node syndrome.
Other features include conjunctival injection and extremity erythema and edema.
Twenty-five percent of untreated patients go onto develop coronary artery
aneurysms. Treatment is with IVIG and aspirin

Q659. Desquamation of the fingertips is commonly associated with infections with


group A beta-hemolytic strep and staph aureus, including scarlet fever, staph scalded
skin syndrome, and toxic shock syndrome. Thus ASO titer would be indicated in this
case. Fingertip desquamation is also a manifestation of Kawasaki disease.

Q661. Acrodermatitis enteropathica is a rare inherited disorder transmitted in an


autosomal recessive fashion. The disorder is caused by an inability to absorb zinc. The
clinical syndrome is characterized by acral dermatitis, alopecia, and diarrhea. Zinc
deficiency from other causes (including chronic wasting, poor oral intake, lack of
supplementation in total parenteral nutrition) can cause similar clinical changes

Q662. Deafness may occur in up to 50% of infants with congenital rubella syndrome

Q663. The diagnosis of zinc deficiency should be considered in at-risk individuals with
acral or periorificial dermatitis. Chronic diaper dermatitis in an infant should lead to
the evaluation for zinc deficiency. A low serum zinc level can usually confirm the
diagnosis. If normal or near normal a low serum alkaline phosphatase, a zinc-
dependent enzyme, may be a valuable adjunctive test

Q665. Fifth disease is caused by Parvovirus B19. Most cases start with fever, malaise,
headache and rhinorrhea. Cutaneous reaction follows approximately 5-7 days later
with erythema of the cheeks ("slapped cheeks") and reticulate rash of the trunk and
extremities.

Q666. Aplasia cutis congenita is characterized by the absence of a portion of skin, most
commonly presenting as a solitary defect on the scalp, but sometimes it may occur as
210
multiple lesions. The lesions are non-inflammatory and well demarcated appear as an
atrophic, membranous, ulcerated area with alopecia. The condition may be associated
with other physical anomlies

Q667. Erythema toxicum neonatorum is a very common eruption in healthy


newborns. A gram stain reveals sterile pustules containing eosinophils Erythema
toxicum usually develops on the second or third day of life and resolves by day 10. A
smear taken from one of the pustules will demonstrate eosinophils, which is sufficient
to make the diagnosis

Q669. Mastocytosis comprises a group of diseases characterized by increased number


of mast cells in the skin and other organs. Seventy-five percent of cases occur before
the age of 2 years. Patients with mastocytosis should avoid potential mast
degranulators including aspirin, codeine, opiates, procaine, spicy foods, cheese,
alcohol, and polymyxin B.

Q670. Leiner's disease or erythroderma desquamativum is a complication of


seborrheic dermatitis in infants. There is usually a sudden confluence of lesions,
leading to a universal scaling redness of the skin (erythroderma). The young patients
are severely ill with anemia, diarrhea, and vomiting. These patients respond to
antibiotics and infusions of fresh frozen plasma or whole blood.

Q672. Subcutaneous fat necrosis of the newborn is rare and self-limited. It is thought
to result from perinatal stress and resolves in 3-6 months. Treatment is usually not
necessary unless Hypercalcemia develops

Q673. Papular acrodermatitis of childhood (Gianotti-Crosti syndrome) typically


affects children 6 months to 10-years of age, and is characterized by symmetric
papules ( sometimes papulo-vesicles) involving the face and buttocks, and extensor
surfaces of the upper and lower extremities. Lesions tend to be asymptomatic.
Multiple infectious etiologies have been implicated, including Epstein-Barr virus
(most common in the U.S.) and hepatitis B virus (most common in Europe). However,
the eruption is most often self-limited, resolving in 4 to 8 weeks, and thus further
work-up (e.g. testing for hepatitis) is indicated only if additional clinical symptoms
are present

Q674. X-linked ichthyosis is characterized by small, dark, firmly adherent scales


accentuated on the sides of the neck and trunk. The face, palms, soles, ante-cubital
and popliteal flexures are generally spared. Associated extracutaneous findings
include corneal opacities (50%), undescended testes (20%), and prolonged maternal
labor (usual). Neurological or mental retardation are rare

Q676. Acne which develops within the first 30 days of life is termed neonatal acne.
Neonatal acne has a predilection for the face, chest, back and groin appearing as small,
discrete papules at 2 to 4 weeks of age, and persisting for up to 8 months. As these
211
lesions are self-resolving, no treatment is necessary, though 2.5% benzoyl peroxide
may hasten resolution. Neonatal acne is quite common and is postulated to occur as
a result of hyperplasia of premature sebaceous glands coupled with transient
increases in circulating androgens. More recent data suggests that Malassezia species
may be implicated at etiologic factors in neonatal acne.

Q677. Neonatal lupus erythematosus (NLE) is a form of sub-acute cutaneous lupus


erythematosus (SCLE) that occur in infants whose mothers have anti-Ro
autoantibodies. Unlike SCLE in adults, lesions of NLE have a predilection for the face
especially per-orbital region. Other sites include scalp, arms, legs, trunk and groin

Q678. In the treatment of infantile haemangioma with propranolol, the most


common serious adverse effects are bradycardia and hypotension. Other potential
adverse effects include bronchospasm and hypoglycemia

Q682. Corynebacterium tenuis is the causative organism associated with most cases.
Although up to 33% of adults have colonization by this bacterium in the inguinal or
axillary regions, factors such as hyperhidrosis predispose to more extensive growth
and resultant clinical manifestation

Q686. Erythrasma is caused by corynebacterium minutissimum. The presence of


coprophyrinogen III created by the bacteria is responsible for the characteristic coral-
red fluorescence under Wood's lamp.

Q689. Erythema infectiosum is caused by parvovirus B19 which is a single stranded


DNA virus

Q690. Infectious mononucleosis is characterized by a sore throat,


painful lymphadenopathy, fatigue, and splenomegaly. The formation of a morbilliform
rash upon challenge with ampicillin or related antibiotics is a classic finding. The most
common virus associated with infectious mononucleosis is Epstein-Barr virus. The
incubation period is relatively long, 3-7 weeks

Q691. Non-treponemal tests for syphilis measure antibodies against phospholipids


released from treponemas and damaged host cells. False positive non-treponemal
tests can occur in the setting of pregnancy, autoimmune diseases, other spirochete
infections, and infectious diseases such as leprosy and malaria. It does not occur in
cases of melanoma.

Q694. Chancroid is a sexually transmitted disease caused by the bacteria Hemophilus


ducreyi. It is one of many organisms that can cause a genital ulcer. Chancroid
is characterized by a soft, painful chancre with ragged edges. Giemsa stained sections
reveal a "school of fish" configuration which is typical. Treatment is with azithromycin,
ceftriaxone, ciprofloxacin, or erythromycin

212
Q697. In any child with peri-anal warts, the possibility of sexual contact must be
excluded. Fortunately, in infants up to 12 months of age, the most common route of
acquisition of the human papilloma virus is vertical transmission at the time of delivery
Q699. The clinical description and lesion shown in the image suggest a diagnosis
of erysipeloid, caused by Erysipelothrix rhusiopathiae. The treatment of choice is
penicillin.

Q700. A tuberculid is a cutaneous immunologic reaction to tuberculosis elsewhere in


the body. By definition, cultures and stains for M. tuberculosis are negative. Erythema
induratum of Bazin/Nodular vasculitis present as erythematous or cyanotic nodules
on the posterior calves. 85% of cases occur in women

Q701. Ulcers in the genital area in sexually active patient generally fall into
two groups: painful and painless. Painless ulcers include syphilllis, lymphogranuloma
venereum, and granuloma inguinale. The primary cause of painful erosion or ulcer is
H. ducreyi or herpes simplex.

Q702. Granuloma inguinale is caused by an infection of calymmatobacterium


granulomatis. It is a granulomatous skin disease of the genitals caused by sexual
transmission. It is a gram negative rod. It is a small, raised papule or subcutaneous
nodule that rapidly ulcerates, rarely with lymphadenopathy. Hemophilis ducreyi is
the causative organism of chancroid. Chlamydia trachomatis
causes lymphogranuloma venereum. Treponema pertenue is the causative organism
of Yaws and T. pallidum is the causative organism of syphilis

Q703. The most common cause of erythema multiforme (EM) is herpes simplex virus,
which may not be active at the time of the EM eruption. Patients with recurrent EM
are typically treated with acyclovir or valacyclovir

Q705. In the setting of pregnancy, amoxicillin is the treatment of choice for Lyme
Disease since doxycycline is contraindicated

Q706. Molluscum contagiosum is usually not seen until a patient\'s CD4 count drops
below 100

Q708. Ecthyma gangrenosum is an infection of critically ill or immunocompromised


individuals by Pseudomonas aeruginosa. Clinically, patients develop opalescent, tense
vesicles or pustules surrounded by a narrow pink to violaceous halo. The lesions
rapidly become hemorrhagic, then violaceous and necrotic leaving ulcers. The most
common location is on the buttocks and lower extremities. Treatment is
with aminoglycosides.

Q709. The DNAse B, hyaluronidase or streptolysin O (ASLO) antibodies in serum can


be helpful for confirming streptococcal infection in situations where streptococci
cannot be isolated. The FTA-ABS is used for detecting syphilis, and is the only test that

213
is positive in the first 14 days of infection. U1-RNP is an extractable nuclear antibody
which is a marker for Mixed Connective Tissue Disease

Q714. Roseola infantum may be cause be either human herpes virus types 6 or 7. It is
the sixth of the traditional exanthema of childhood

Q715. Hutchinson’s teeth (widely-spaced, peg-shaped upper incisors) are a seen


in Late Congenital Syphilis (> 2 years of age).

Q716, Q717. Scrofuloderma is a tuberculous or non-tuberculous mycobacterial


infection affecting children and young adults, representing direct extension of
tuberculosis into the skin from underlying structures such as lymph nodes, bone.
It most commonly presents as a neck abscess secondary to a tuberculosis infection of
the cervical lymph nodes. It is manifested by the development of painless
subcutaneous swellings that evolve into cold abscesses, multiple ulcers, and draining
sinus tracts. The diagnosis is based on positive Mantoux test/PPD test, histologic
features of caseating granulomatous inflammation, culture, and favorable response
to anti-tuberculosis therapy.

Q718. Oral hairy leukoplakia is an oral mucosal infection most often caused by the
Epstein-Barr virus, which occurs in immunocompromised patients. Clinically, thick,
white plaques are noted on the lateral sides of the tongue.

Q720. Purpura fulminans is an often times dramatic presentation of large ecchymotic


areas of the skin, favoring acral sites including the extremities, ears, and nose. The
underlying pathophysiology is hemorrhagic infarction of the skin, due to disseminated
intravascular coagulation. Group A streptococcal infection is the leading underlying
infection associated with this condition

Q721. LGV is a sexually transmitted disease that is characterized by suppurative


inguinal adenitis with matted lymph nodes, inguinal bubo with secondary ulceration,
and constitutional symptoms. It is caused by Chlamydia trachomatis serotypes L1, L2,
and L3. First line treatment is doxycycline 100 mg bid for 3weeks as well as treating the
sexual partners. Gamma-Favre bodies are found in histiocytes in LGV

Q722. First line treatment of herpes zoster (shingles) is with valacyclovir, acyclovir,
or famciclovir. Gabapentin may be used for post-herpetic neuralgia. Cidofovir
and ganciclovir are treatment options for cytomegalovirus. Foscarnet is used to treat
acyclovir resistant herpes simplex infections

Q723. The Jarisch-Herxheimer Reaction characterized by fever, headache,


lymphadenopathy, myalgia, and elevated white blood cell count. It caused by the
release of inflammatory cytokines, particularly TNF-alpha, due to phagocytosis of
spirochetes following antibiotic administration

214
Q724. Recurrent oro-labial herpes simplex virus infection is a very common problem.
In many cases, the cause of the recurrences is idiopathic. Identifiable triggers include
light exposure and emotional stress, among others. UVB is the most commonly
implicated factor

Q725. Hutchinson's sign is defined by vesicles at the nasal tip in a patient with
facial herpes zoster. It is caused by VZV invovlement of the nasociliary nerve. Patients
suffering with this sign should be screened for ophthalmologic herpes zoster
invovlement.

Q726. Recurrent erythema multiforme (EM minor) is usually caused by recurrent


herpes simplex, most commonly HSV-1 . This is more correctly now called herpes-
associated erythema multiforme(HAEM) and has been more commonly associated
with people with HLA type B15.

Q727. Hansen's disease ( Leprosy) is a chronic granulomatous disease principally


affecting the skin and peripheral nervous system, caused by Mycobacterium
leprae. The parts of the body most commonly affected are the superficial peripheral
nerves, skin, mucous membranes of the upper respiratory tract, anterior chamber of
the eyes, and testes. These areas tend to be cool parts of the body.

Q731. The annual risk of developing herpes zoster in an HIV infected individual,
regardless of the CD-4 count, is approximately 3%

Q734. Chancroid classically presents with a soft, tender chancre with ragged edges
and unilateral, tender inguinal adenopathy (bubo). A school of fish pattern on Gram
or Giemsa stain is diagnostic

Q735. Many subtypes of HPV are associated with genital warts but HPV type 6 and
11 are the most common.

Q737. Eosinophilic pustular folliculitis is a pruritic disorder that may be seen in


HIV positive patients.

Q738. The most likely cause of a non-healing ulceration in someone who has been in
a tropical location is M. ulcerans. Mycobacterium marinum causes inflammatory
nodules following injury in an aquatic environment. Mycobacterium chelonei is a
rapidly growing organism most commonly found in soil, water, dust and animals
which presents as single/multiple erythematous SC nodules on an extremity or
following a surgical procedure. Mycobacterium kansasii rarely causes skin lesions.
Leishmania donovani is a cause of visceral leishmaniasis in India and Kenya (kala-
azar)

215
Q739. Papulo-squamous lesions are characteristic, and the presence of
constitutional symptoms such as fever, fatigue, headaches, and bone pain may help
distinguish secondary syphilis from pityriasis rosea. Other lesions of secondary
syphilis include a moth-eaten alopecia, mucous patches and condyloma lata. The
Muco-cutaneous manifestations of secondary syphilis typically occur 3-12 weeks
after the appearance of the chancre and last 4-12 weeks

Q740. No therapy is needed, since Orf is usually a benign, self-resolving infection. A


parapox virus, often associated with sheep and goats, causes Orf. The infection
undergoes sex different stages of evolution: the papular, target, acute, regenerative,
papillomatous, and regressive stages

Q741. The presentation of a suppurative nodule discharging sulfur granules on


the jawline of a patient with poor oral hygiene is characteristic of actinomycosis –
which caused by the anaerobic gram-positive rod, actinomyces israelii. This differs
from actinomycetoma, which is a slowly progressive sub cutaneous infection
characterized by tumefaction, draining sinuses, and an exudate containing grains on
microscopy. Unlike Eumycetoma - caused by fungi - Actinomycetoma are bacterial
infections.

Q742. The most common subtype of molluscum contagiosum virus is MCV-1. In


HIV patients, however, MCV-2 is more common.

Q743. Impetigo, is a common, contagious, superficial skin infection caused most


commonly by staphylococci. The disease is most common in infants and children.
Lesions may involve any body surface but occur most frequently on the exposed parts
of the body, especially the face, hands, neck, and extremities. There are two classic
forms of impetigo, bullous and non-bullous. Non-bullous impetigo accounts for more
than 70% of cases

Q745. Molluscum contagiosum is a common opportunistic infection of HIV patients.


Lesions can be seen anywhere, but the most common locations are on the face and
genitalia

Q748. Ecthyma is a deep or ulcerative type of pyoderma commonly seen on the


lower extremities and buttocks and caused most often by group A B-hemolytic
streptococcus, in addition to staphylococcus aureus which also can be involved in the
infection. Ecthyma can present as small punched-out ulcers or a deep spreading
ulcerative process. The disorder begins in the same manner as impetigo, often
following infected insect bites or minor trauma, but penetrates through the epidermis
to produce a shallow ulcer. The initial lesion is a vesiculo-pustule with an
erythematous base and firmly adherent crust. Removal of the crust reveals a lesion
deeper than that seen in impetigo, with an underlying saucer-shaped ulcer and raised
margin. Ecthyma gangrenosum a cutaneous finding that may be seen in patients with
Pseudomonas aeruginosa bacteremia. Most of the affected individuals have an

216
underlying immunodeficiency (either congenital or acquired) or a history of cancer
chemotherapy. Neutropenia may be a risk factor for ecthyma gangrenosum

Q749. Treponema carateum is the cause of Pinta, which is primarily transmitted by


direct contact. Occasionally insect vectors have also been implicated, but sexual
contact is not the primary cause of spread. The other options (C. trachomatis, H.
ducreyi, C. granulomatis and T. pallidum) are all venereal diseases.

Q750. Necrolytic acral erythema is characterized by hyperkeratotic well-defined


plaques on the lower extremities, seen occasionally in patients with hepatitis C virus
infection. Treatment consists primarily of treating the underlying infection.
Diet supplementation with zinc has been shown to improve necrolytic acral erythema,
even in the presence of normal serum zinc levels

Q751. Actinomyces israelii is an anaerobic gram-positive rod, which causes chronic


suppurative nodules and sinus tracts with an exudate containing sulfur granules. It
most commonly affects the cervico-facial area, especially near the mandible. The
abdomen and thoracic areas can also be affected. The source of infection is
endogenous. Patients with poor oral hygiene, penetrating foreign bodies, and recent
dental procedures are at increased risk. Treatment of choice is penicillin.

Q753. Gonorrhea, caused by a gram negative intracellular diplococcus, often presents


cutaneously with hemorrhagic vesiculo-pustules of the distal extremities
(gonococcemia). Septic arthritis is also an important presentation of most commonly
involving the knee.

Q754. Lymphogranuloma venereum presents as a painless ulcer, and is caused by


Chlamydia trachomatis serotypes L1, L2, L3. Buboes can be aspirated, they should not be incised
and drained. Transmission occurs through sexual contact, but can occur via skin-to-skin
contact. Complications of late stage disease include procto-colitis, perirectal
abscesses, recto-vaginal fistulas, and anal fistulas.

Q755. The human body louse is implicated in the transmission of trench fever,
epidemic typhus, and relapsing fever. Trench fever usually affects alcoholic men and
is manifested by fevers. Treatment is with ceftriaxone, erythromycin, or doxycycline.
Epidemic typhus, is manifested by fevers, chills, malaise, and a pink macular eruption
beginning in the axilla and trunk. Treatment is with tetracycline or chloramphenicol.
Relapsing fever is manifested by paroxysmal fevers, headache, and
erythematous/petechial macules on trunk/extremities. Treatment is
with doxycycline.

Q756. The Lowenstein-Jensen medium is most commonly used to culture


mycobacteria. chocolate agar in 10% CO2, Thayer Martin, Martin-Lewis or New York
City medium also allow for N. gonorrhea growth on.. Agar supplemented with heme
(x-factor) and nicotinamide (v-factor) is needed for Hemophilus influenzae cultures.

217
Sheep blood agar is useful for identifying hemolytic strains of streptococcus and
staphylococcus.

Q758. Scabies is caused by the mite sarcoptes scabiei var. hominis. The life cycle of
the mite is 30 days. A female mite will lay 60-90 eggs during her life.

Q759. Mazzotti reaction is used to test foronchocerciasis in which a single dose of


dimethycarbamazine is given for reaction

Q760. Bipolar, safety pin-shaped intra-cytoplasmic inclusions on Giemsa stain -


known as Donovan bodies – are diagnostic of granuloma inguinale.

Q761. Beau’s s lines are transverse grooves in the nails. They are caused by a
generalized systemic condition, which disrupts the nail growth

Q765. Trichophyton rubrum is the most common organism involved in distal


subungual Onychomycosis
Distal subungual Onychomycosis begins under the leading edges and characterized by
the accumulation of subungual debris and thickened, discolored nails. In proximal
subungual Onychomycosis, infection begins at the proximal nail fold with infection of
the nail matrix. The most common cause of both types is trichophyton rubrum,
although trichophyton mentagrophytes and epidermophyton floccosum are also
frequently isolated. Superficial white onychomycosis is characterized by white patches
on the surface of the nail and is caused primarily by trichophyton mentagrophytes.
Tinea ungium is defined as fungal infection of the nail caused dermatophyte, whereas
Onychomycosis refer to any fungal infection of the nail.

Q768. Mee's lines are transverse white lines that affect all nails, grow out with nail
growth. They are seen in arsenic poisoning, rheumatic fever, congenital heart failure,
leprosy and with significant systemic disease. Brownish macules beneath the nail plate
are oil spots, seen in psoriasis. The double white transverse lines from abnormal
vascular bed are Muehrcke's lines which are caused by a nephrotic syndrome, low
albumin, liver disease or malnutrition

Q769. It is a type of drug known as a biological therapy. In rheumatoid arthritis and


some other inflammatory conditions, too much of a protein called TNF is produced
by immune system. This cause inflammation, pain and damage to the joint. .Anti-TNF,
such as etanercept block TNF and reduce inflammation

Q797. This elderly patient is presenting with asteatotic eczema, also known as eczema
craquelé. This form of eczema is most common in elderly patients and results from
extreme xerosis. It mostly occurs on the shins, flanks and axillae as a dry, scaly rash
with cracking giving the appearance of a dry riverbed. It is exacerbated by, low
humidity and excessive washing of dry skin. The skin changes related to
218
hypothyroidism also predispose to xerosis and subsequent asteatotic eczema. Patch
testing is not unreasonable in asteatotic eczema as sensitization to medicaments does
occur, but this is less likely when using greasy thick emollients

Q798. This elderly woman initially presented with venous eczema, also known as
stasis dermatitis. This form of eczema presents as a red itchy rash developing in an
area of longstanding venous hypertension. Often on the legs of elderly women the
eczema is associated with hemosiderin deposition, varicosities, oedema, induration,
atrophie blanche and lipodermatosclerosis. Treatment is both of the eczema itself with topical
steroids and emollients and of the venous hypertension with compression
therapy. The patient has subsequently developed disseminated eczema,also known as
an Id reaction or auto-sensitization dermatitis. This
is where widespread secondary lesions of eczema occur distant to the primary site.
The commonest cause of disseminated eczema is an allergic contact dermatitis
complicating stasis dermatitis. In this case, the patient has developed an allergic
contact dermatitis to neomycin which is a more common sensitizer than
betamethasone.

Q799. Juvenile plantar dermatosis affects atopic pre-pubertal children from the age
of three years when they start to wear shoes most of the time. It is thought to arise
due to the humid environment created within impermeable shoes made of plastic and
rubber. Treatment is advice on wearing breathable socks and leather shoes and allowing the
socks to dry whenever possible. Emollients and keratolytics may be used to
complement this advice, topical steroids are not needed and there is no fungal
component to this condition.

Q800. This gentle man has a rash consisting of beefy red plaques and overlying silvery
scale, a classical description of chronic plaque psoriasis. Psoriasis affects 1–2% of the world’s
population with a higher prevalence in Western Europe and America and a lower prevalence in
Africans, Chinese and Native Americans. The genetics of psoriasis are complicated
and polygenic but are a significant risk factor for developing the disorder. A monozygotic
twin such as this patient has a 73% chance of developing psoriasis if his twin has the
condition. Triggering events that can precipitate latent psoriasis

•Cutaneous injury – the Koebner phenomenon


•Infections – particularly streptococci, HIV
•Psychogenic stress
•Endocrine factors – pregnancy, hypocalcaemia
•Drugs – beta-blockers, lithium, anti-malarial drugs, interferon. In this question, the
likely precipitants are the psychogenic stress of the myocardial infarction and beta-
blockers. Psoriasis is not infectious and can-not be caught. The other drugs postulated are not
known precipitators of psoriasis.

Q801. The patient is presenting with guttate psoriasis following aprobable


streptococcal sore throat. Guttate psoriasis is most common in children and often
follows a streptococcal infection. In children the prognosis is excellent with most
cases resolving in a few weeks to months. In adults the prognosis is more guarded

219
with many cases becoming chronic. If antibiotics are given early to treat the infection
this can also resolve the rash. If the rash has developed it often responds well to
topical coal tar, mild topical steroids and light therapy. Potent topical steroids and
oral prednisolone risk precipitating an unstable pustular psoriasis and should be
avoided. Dithranol should only be applied to psoriatic plaques, avoiding normal skin,
it would be impossible to apply to guttate disease. Cyclosporine is
extremely effective in treating psoriasis but has many side effects and should be
reserved for resistant cases.

A. Generalised pustular psoriasis

The history is of an unwell, pyrexial patient with an extensiveerythematous rash and


multiple sterile pustules. This is a typical presentation of generalised pustular
psoriasis, an unusual manifestation of psoriasis, that may be triggered by pregnancy. Other
triggers include rapid tapering of systemic corticosteroids, hypocalcaemia and
infections. The rash is characterized by erythema and an epidermal neutrophilic
infiltrate causing multiple sterile pustules. Patients can be quite unwell, and pyrexic.
There is an overlap between this condition and acute generalized exanthematous
pustulosis (AGEP) a pustular drug eruption. Treatment options include retinoids, cyclosporine
and light therapy. Staphylococcal scalded skin syndrome more commonly presents in
children with thin walled blisters rather than pustules. Toxic epidermal necrolysis is
characterized by, sheets of epidermal detachment and mucous membrane
involvement. Eczema herpeticum occur on a background of eczema and has vesicles
rather than pustules .Gestational pemphigoid presents with itchy urticated plaques
that form thick-walled blisters.

Q 804. Pityriasis lichenoides et varioliformis acuta (PLEVA) and pityriasis lichenoides


chronica (PLC) are considered to be two ends of the same disease spectrum. Both
are characterized by, recurrent crops of self-resolving erythematous papules. The lesions of
PLEVA are more acute and tend to be crusty, vesicular and pustular whereas PLC tends to be more
chronic and scaly. Treatment consists o topical steroids, coal tar products, antibiotics and light
therapy. The recurrent crops of PLEVA/PLC are quite characteristic and exclude other
conditions such as mycosis Fungoides and guttate psoriasis. Parapsoriasis is a related
T-cell lymphoproliferative disorder that is considered by some to be a precursor to
mycosis fungoides. It presents with chronic, asymptomatic scaly patches.

220
Q805. This patient is presenting with clinical features consistent with pityriasis rubra
pilaris (PRP). Some of the characteristic features include follicular papules on
an erythematous base, islands of sparing and orange palmar-plantar keratoderma.
As in this case, the disorder can develop into an erythroderma. There are five types
of PRP of which the adult type is commonest, childhood and inherited forms also
exist. The classical adult variant of PRP normally resolves in 3–5 years.

Q806. 14 C. Reassure the patient and advise a little sun exposure

This young girl gives a typical history of a herald patch appearing on her back
followed by the development of pityriasis rosea. Pityriasis rosea is a self limiting
eruption that affects young adults. It is often asymptomatic or mildly pruritic. The
rash favours the trunk and proximal extremities and usually resolves in 6–8 weeks. It
is thought that the rash is precipitated by a viral infection, possibly of the
herpes virus family. In dark skin the rash tends to be hyperpigmented. Less common
variants can be inverse, vesicular, purpuric or pustular. In this case as the rash is
asymptomatic it is best not treated although a small amount of sun exposure may
hasten its resolution. If the rash was itchy then symptomatic relief may be obtained
using a topical steroid cream and oral antihistamines. Erythromycin and UVB
phototherapy have both been used in resistant cases. There is no indication for oral
corticosteroids.

Q807. The two commonest causes of erythroderma in adults. Common causes of


erythroderma in adults:

•psoriasis
•atopic dermatitis
•drug reactions
•idiopathic
•cutaneous T-cell lymphoma.
Common causes of erythroderma in children and neonates:
•atopic dermatitis
•seborrheic dermatitis
•psoriasis
•inherited ichthyosis
•immunodeficiency

infection. Given this scenario it is important to carefully examine the patient’s drug history; in
this case none of her medications are strongly associated with erythrodermic drug
reactions. Medications associated with erythrodermic drug reactions:

•allopurinol
•beta-lactam antibiotics
•anti-epileptic medications
•gold
•sulphonamides.

221
The most likely diagnosis is cutaneous T-cell lymphoma given thehistology of a
lichenoid interface dermatitis and the presence of atypical lymphocytes. A signifcant
proportion of cases labelled ‘idiopathic’ subsequently develop cutaneous T-cell lymphoma, it
is important to follow these patients up and consider repeat skin biopsies.

807. D. Admit the patient to the dermatology ward for assessment and stabilisation

This middle-aged patient is presenting with rapid onset erythrodermaand systemic


upset. He is shivering due to loss of thermoregulatory control and is dehydrated
leading to postural hypotension and thirst. He may also be suffering from other
complications of erythroderma including tachycardia, protein loss,
peripheral oedema and secondary infection. As the patient is systemically unwell
he needs admission to hospital or assessment and stabilization of his medical
condition. He is not unwell enough to require support from the intensive care
unit, although elderly patients and patients with other medical co-morbidities
may become this unwell. Systemic corticosteroids are used in erythroderma caused
by for example, a drug reaction, but at this early stage it is important to first exclude
conditions such as erythrodermic psoriasis in which systemic corticosteroids may be
contraindicated.

Q808. Phytophotodermatitis occurs when a plant derived chemical, commonly


psoralen, comes in contact with the skin and is activated by UVA light. This results in a
phototoxic reaction consisting of erythema, blistering and delayed hyperpigmentation.
The cutaneous features often start 24 hours after exposure and peak at 72 hours.
Most cases of phytophotodermatitis are due to contact with two families of plants –
apiaceae and rutaceae. The apiaceae family includes cow parsley, celery, fennel and
parsnip. The rutaceae family includes citrus fruits such as lime and orange.

Q809. D. She must not use permanent hair dyes in the future.

the history is suggestive of an allergic contact dermatitis to para-phenylene-diamine


(PPD) a component of temporary tattoos. The patient developed a reaction three days
after the tattoo was applied, which is suggestive of a type IV allergic reaction to
a compound she was not previously sensitized to. In this case, the reaction
was severe with bullae and postinfammatory hyperpigmentation. Patch testing woul
d be the investigation of choice to confirm the diagnosis of PPD allergy. PPD is most
commonly used in permanent hair dyes, it is also found in textile dyes, darkly coloured
cosmetics and inks. Given the severity of the original reaction, it is likely that the
patient will have a severe type IV allergic reaction to permanent hair dyes and these
should be avoided along with other PPD containing products. She is not allergic to
henna itself and as long as the henna product does not contain PPD it should be safe
to use

Q812. The antigen implicated in pemphigus foliaceus is Desmoglein 1. Desmoglein 1


is targeted in bullous impetigo, but this is an infectious condition, not an
autoimmune disease. The antigen in dermatitis herpetiformis is transglutaminase 3.
222
The antigens for both bullous pemphigoid and pemphigoid gestationis are BPAG1
and BPAG2

Q814. Eumelanin is the pigment of brown/black hairs, and pheomelanin is the


pigment of red/blonde hairs. Intensity of color is proportional to the amount
of pigment. The absence of pigment produces white hair, and markedly reduced
pigment produces gray hair

Q815, Q816, Q817, Q818. Swollen, itchy and or red after stroking the skin is referred to
Darier's sign and can be seen in systemic mastocytosis or urticaria pigmentosa.
Nikolsky sign can be seen when an intact epidermis shears away from the underlying
dermis, leaving a moist surface (seen in pemphigus vulgaris, staphylococcus scalded
skin syndrome (SSSS), and toxic epidermal necrosis). Spreading bulla phenomenon
with pressure on an intact bulla is referred to as Asboe-Hansen sign, commonly seen
with pemphigus vulgaris. Central depression within a lesion when squeezed is
referred to as the dimpling sign and is seen in dermatofibromas. Disappearance of
color or blanching when the lesion is pressed is commonly found on vascular lesions

Q819. C: Amiodarone-induced hyperpigmentation.

Amiodarone is an iodinated compound used in the treatment of ventricular


arrhythmias that are refractory to other medications.
Actinic lichen planus is a rare clinical form of lichen planus that occurs in exposed
skin areas. The pigmented variant of the disease typically involves only the face,
with clinical features resembling melasma. Histopathologic features seen on
skin biopsy of actinic lichen planus include a band-like lymphocytic infiltrate with
numerous melanophages observed in the dermis.
Hemochromatosis is one of the most common genetic errors of metabolism, and is characterized
by a diffuse cutaneous slate-gray pigmentation in light-exposed areas. The onset of
the disease generally occurs during the fourth or fifth decade of life. Men are more
commonly affected than women. Other findings may include diabetes and a high
serum iron level.
Ashy dermatosis, also known as erythema dyschromicum perstans, is a rare
idiopathic dermatosis characterized by ash-gray skin lesions. The ash-colored macules vary in size
and shape, and are localized to the arms, neck and trunk. The face is usually involved.
The characteristic blue-gray skin color reflects the presence of melanin in the
macrophages of the dermis.
Argyria occurs in persons exposed to silver salts and clinically appears as a slate-
gray pigmentation on sun-exposed areas of the skin, especially the forehead and
nose. The diagnosis of argyria is well established by skin biopsy. Conventional
histology shows silver granules in the dermis particularly evident near the basal layer
of sweat glands.

Q821. Collimation is the parallel nature of coherent light waves. Monochromicity is


the emission of a single wavelength. Coherence is the term for light waves traveling
223
in phase. Energy is the fundamental unit of work. Power is the rate of energy
delivery measured in watts

Q822. Imiquimod is an immunomodulator, which induces the production of cytokines


that are involved in cell-mediated immunity. These cytokines include interferon-
alpha, interferon-gamma, interleukin-1, interleukin-10 and tumor necrosis factor-
alpha. Induction of these cytokines results in antiviral and antitumor activity in vivo

Q823. Radiesse is an injectable, bio-degradable filler, can be seen on radiographic


imaging. Botox is botulinum toxin; Zyplast is bovine collagen; Cosmoderm is a non-
animal form of collagen; and Restylane is a non-animal form of hyaluronic acid.

Q824. Silk is a non-absorbable braided suture which has been shown to aid in the
production of infection

Q825. Postsurgical wound dressings are intended to aid in hemostasis, prevent


infection and provide a semi-occlusive environment. The most important
postoperative function is to immobilize the wound and surrounding tissue to allow
the newly formed platelet plugs to remain intact

Q826. Q-switched lasers can be effectively used to remove tattoos. However,


immediate darkening of white, flesh colored, and pink/red tattoos has been reported
after treatment with Q-switched lasers

Q827. Hyaluronic acid is a natural component of human connective tissue. As


a cosmetic filler, hyaluronic acid is an attractive alternative to currently available
filler substances: its ability to bind water creates volume and plumps the skin; its
duration of action of approximately 6-12 months is appreciably longer than
conventional collagen; and since hyaluronic acid is chemically identical across all
species, hypersensitivity reactions are rare.

Q829. Based on the histopathologic classification and location, this infiltrative basal
cell carcinoma (BCC) is best treated with MOHS surgery to preserve normal tissue
and structure. 5-fluorouracil cream is used to treat superficial BCC, squamous cell
carcinoma in situ (SCCIS), and actinic keratosis. Electrodessication and curettage is
used to treat SCCIS and superficial BCC. Standard excision would be possible if
preservation of normal tissue and structure were not as crucial as in this area (the
nasal ala). Imiquimod, is used to treat actinic keratosis and superficial basal cell
carcinoma.

Q830. Location of tumor is an important risk factor for metastasis of squamous cell
carcinoma. Compared with a 10% likelihood of metastasis for tumors located on the
ear or the lip, an SCC developing in the scar, however, has been estimated to
metastasize at a rate as high as 30-40%.

Q831. The use of epinephrine with local anesthesia has two main advantages. Firstly,
the epinephrine is a vasoconstrictor and assists in controlling bleeding during
surgery. Secondly, epinephrine helps prolong the duration of the anesthetic agent

224
100% to 200% by delaying its absorption from the surgical site. Although the
anesthetic properties of lidocaine take effect within the first few minutes of
infiltration, the vasoconstrictive effects of epinephrine require approximately 15
minutes to fully develop.

Q833. Injury from lasers may occur via direct or indirect ocular exposure. Damage is
generally wavelength specific. Laser that target hemoglobin or pigment may cause
damage to the retinal pigment or vasculature while lasers that target water as a
chromophore (carbon dioxide and erbium) can damage the cornea.

Q834. The carbon dioxide laser operates at a wavelength of 10,600 nm and targets
water as a chromophore. Because of the high water content of the cornea, it may be
damaged by exposure to irradiation from the carbon dioxide laser. Exposure to the
erbium may cause corneal damage as well.

Q835. The use of follicular-unit grafts, which contain one to four hair follicles,
represents the advancement in both surgical technique and aesthetic outcome in the
field of hair transplantation. The grafted hair follicles typically begin to grow within 8
to 10 weeks of implantation and are expected to survive for the individual’s lifetime.

Q836. Sculptra (poly L-lactic acid). Sculptra (called New-Fill outside of the US) is a
biodegradable filler composed of poly-L-lactic acid, the same material used in
absorbable sutures. Preliminary studies have demonstrated longevity of the filler at
two years post-treatment

Q853. The skin can provide an important clue to the presence of an underlying neural
tube defect, such as meningomyelocele and encephalocele. Cutaneous lesions along
the midline of the spine should always prompt consideration of this possibility.
Although, midline neural tube defects are uncommon, early recognition and
diagnosis of a spinal dysraphism can have important implications for early surgical
correction and minimizing loss of neurologic function. Clues to the diagnosis include
a midline dimple, tuft of hair, lipoma, or vascular lesion. In these instances, imaging
studies (MRI, CT, and ultrasound) should be promptly initiated.

Q854. Transient neonatal pustular melanosis typically begins with sterile pustules that
leave a characteristic collarette when ruptured. The lesions heal with hyperpigmented
macules

855. Subcutaneous fat necrosis of the newborn is a rare condition characterized by


firm, reddish or purple nodules which appear on the arms, thighs, buttocks, back and
cheeks. This condition is believed to result from cold injury. Onset of this condition
occurs within the first two weeks of life and usually resolved over a period of weeks
to months. Occasionally, lesions heal with atrophy. Hypercalcemia is an infrequent
complication of subcutaneous fat necrosis of the newborn. Associated symptoms can
include irritability, weight loss, vomiting and failure to thrive. Repeated serum
calcium tests are advised until one month after all the cutaneous lesions have
resolved
225
Q856. Ataxia Telangiectasia (Louis-Bar syndrome) is an autosomal recessive disorder
usually caused by mutations in the ATM gene. Cerebellar ataxia is the first sign,
followed by telangiectasia of the conjunctiva and skin

Q857. Mal de Meleda is an autosomal recessive disease characterized by palmo-


planter keratoderma, hyperhidrosis, keratotic plaques at knees and elbows,
subungual hyperkeratosis.

Q861. The earliest and most common cutaneous manifestation of tuberous sclerosis
are hypomelanotic macules (ash leaf spots), typically presenting at birth or early
infancy. 3 or more of these lesions are considered a major criteria in diagnosis.
Melanocyte numbers are normal. Confetti-like macules, on the other hand, are
usually not apparent until the second decade of life. Facial angiofibromas occur in
approximately 75% of patients, and tend to become more prominent with age.
Periungual fibromas are considered a major feature for diagnosis, usually presenting
around puberty to early adulthood

Q 862. Sturge-Weber syndrome is a sporadic disease characterized by a capillary


malformation (port-wine stain) in the trigeminal distribution (forehead, scalp, or
around the eye). Patients may have associated cerebral atrophy, vascular
malformations and seizures. All patients with Sturge-Weber should be referred to
the ophthalmologist for glaucoma screening

Q 863. Aplasia cutis congenita presents as well-demarcated erosions at birth with


heal with atrophic alopecic scars. It can be inherited in autosomal dominant,
autosomal recessive, or sporadic forms. Aplasia cutis congenita can be caused by
teratogens, particular methimazole (medication is given to treat an overactive
thyroid gland)

Q 864. Epidermolysis bullosa simplex is genetic condition that cause the skin to be
very fragile and to blister easily. Blister and erosion occur in response to minor injury
or friction such as rubbing or scratching. There are four types of epidermolysis
bullosa simplex. The types differ in severity and they are caused by mutation in the
same gene (keratin 5 and 14) resulting in bullae within basal cell keratinocytes

Q 865. Pseudoxanthoma elasticum is progressive disorder that is characterized by,


the accumulation of calcium and other minerals in elastic fibers of the skin, eyes, and
blood vessels. It is presented clinically with yellowish papules on the neck, under arm
and flexor area of the body. They may also have abnormalities in the eyes such as
angioid streaks and pigmented cells of the retina. Bleeding and scarring of the retina
may also occur, which can cause vision loss..

Q 866. Acrodermatitis enteropathica is an autosomal recessive metabolic disorder


affecting the zinc absorption and will respond to zinc supplementation. It is

226
characterized clinically by, inflammation of the skin around body opening (peri-
orificial) and the tips of fingers and toes, hair loss (alopecia) and diarrhea.

Q 867. Wiskott - Aldrich syndrome, is inherited in an X-linked recessive manner,


most often occur in males. It is characterized by; eczema, thrombocytopenia,
immune deficiency and bloody diarrhea (secondary to thrombocytopenia)
Sjogren-Larsson syndrome (inborn error of lipid metabolism), inherited as an
autosomal recessive pattern, characterized clinically by congenital ichthyosis,
intellectual disability and muscle spasm
Netherton syndrome is inherited in an autosomal recessive pattern. It is a disorder
that affect the skin (ichthyosiform erythroderma), hair (trichorrhexis nodosa or
invaginata) and immune system
a. Post herpetic neuralgia
b. Dissemination, especially in immunocompromized patient
c. Motor paresis
d. Herpetic encephalitis
e. Herpetic keratoconjunctivitis.

Q139. The followings are used in the treatment of post herpetic neuralgia except
a. Acupuncture
b. Acyclovir
c. Intralesional steroid injection
d. Amitriptyline
e. Gabapentin.

140. In chickenpox, the followings are true except


a. Caused by varicella zoster virus
b. Commonly seen in children
c. Polymorphic eruption and centripetal distribution
d. Transmitted by direct skin contact
e. Usually carry a good prognosis.

Q141. In chickenpox, the followings are true except


a. Patients are contagious
b. An attack of chicken pox usually confers life-long immunity
c. Incubation period varies from 1-2 days
d. It is spread by droplets infection
e. Sub-clinical infection may occur.

Q142. Complication of the chickenpox include the following except


a. Encephalitis
b. Pneumonitis
227
c. Glomerulonephritis
d. Reye’s syndrome
e. Septicemia.

Q143. In the treatment of chickenpox, the followings are true except


a. In severe cases, acyclovir either orally or intravenous are useful
b. Aspirin (not paracetamol) as it may predispose to Reye syndrome is useful for control
of fever, headache, and myalgia
c. Daily bath with antibacterial soap help prevent secondary bacterial infection of the
lesion
d. Oral antihistamine is helpful
e. Appropriate antibiotics for widespread cutaneous or other bacterial complications are
indicated.

Q144. In measles (Rubeola), the followings are true except


a. It is spread via close skin contact
b. The incubation period is 1-2 weeks
c. The skin rash may start on the fourth day as red macules and papules on the face
d. Koplik’s spots appear on the buccal mucosa during the prodrome
e. It may be complicated by secondary bacterial infection (bronchopneumonia and otitis
media).

Q145. In molluscum contagiosum, the followings are true except


a. Caused by human papilloma virus
b. Common in children and young adults
c. It is contagious, transmitted by autoinoculation and skin contact
d. Genital molluscum contagiosum in children may be a manifestation of sexual abuse
e. In adults, sexual transmission is also well recognized.

Q146. In molluscum contagiosum, the followings are true except


a. Clinically appear as pearly papules with central umblication
b. Caused by pox virus
c. Common in children
d. One of the sexually transmitted disease
e. Acyclovir is an effective treatment.

Q147. In molluscum contagiosum, the followings are true except


a. It is an asymptomatic flesh-colored dome-shaped papule
b. Caused by pox virus
c. It is common in middle and old age
d. The lesions are harmless and involute spontaneously
e. Liquid nitrogen can be used in the treatment.

Q148. In molluscum contagiosum, the followings are true except


a. Common in children
b. Caused by a human papilloma virus
228
c. Transmitted by sexual contact
d. Papule with central punctum is an important diagnostic feature
e. Widespread infection in adults is characteristic of HIV infection.

Q149. In viral warts, the followings are true except


a. Viral infection of keratinocyte caused by pox virus
b. It can affect the genital region
c. Transmitted by touch or contact with infected human
d. Commonly appearing at the site of trauma
e. Liquid nitrogen can be used in the treatment.

Q150. In plane warts, the followings are true except


a. Flesh colored or pigmented slightly elevated papules
b. May be few but generally they are numerous
c. They have smooth surface
d. Commonly appear on the face and back of the hands
e. Commonly in the adults but it may occur in children.

Q151. In genital warts in children, the followings are true except


a. It can be a result of sexual abuse
b. It can be acquired without sexual abuse
c. A child with warts on the hands can transmit the wart to the mouth or genitalia
d. Mother with hand wart can transmit it to her child
e. Acyclovir is the treatment of choice.

Q152. In general treatment of viral warts, the followings are true except
a. Electrocautery
b. Cryotherapy
c. Systemic acyclovir
d. Systemic retinoids
e. Interferon, both systemic or intralesional, can be used in the resistant cases.

Q 868. Epidermodysplasia verruciformis (tree-man syndrome) is a rare autosomal


recessive disorder in which an impaired cellular immunity, allows person highly
susceptible to infection caused by certain subtypes of the human papilloma virus
(HPV). Some of these lesions have a tendency for malignant transformation, most
commonly those verruca caused by HPV type 5 and 8

Q 869. KID Syndrome; is characterized by keratitis, ichthyosis (erythrokeratoderma),


and deafness (sensorineural). It is inherited in an autosomal dominant fashion.
Sparse hair and alopecia is also quite common.

Q 870. X-linked ichthyosis is a X-linked recessive disorder secondary to steroid


sulfatase deficiency, characterized by brown adherent scale on the neck, trunk and
extrimities especially the extensor surfaces). Additional findings include comma-
shaped corneal opacities, cryptorchidism, and failure to progress during labor

229
Q 871. Dyskeratosis congenita is a rare progressive congenital disorder usually
inherited in an X-recessive fashion due to mutations in the dyskerin gene. The less
common autosomal dominant form is caused by mutations in the telomerase gene.
Clinically, the triad of abnormal skin pigmentation, nail dystrophy and leukoplakia of
the oral mucosa. The disease initially mainly affects the skin but a major
consequence is progressive bone marrow failure, which occurs in the majority of the
patient causing early mortality.

Q 874. Sculptra (New-Fill) is a biodegradable filler composed of poly-L-lactic acid, the


same material used in absorbable sutures. Preliminary studies have demonstrated
longevity of the filler at two years post-treatment.

Q 875. Solid media lasers include; alexandrite, KTP, ruby, Nd:Yag, diode.
Liquid media is found in the pulsed dye laser.
Gas media is used in argon, carbon dioxide

Q 876. Salicylic acid is a very superficial chemical peel that is used for acne and milia.
This peel localizes to the pores given its lipophilic nature. It is a self-neutralizing peel.

Q 877. Silk is the most appropriate suture for closing a wound on the buccal mucosa.
Other sutures may be used as well, but classically, silk is used because of its soft
nature

Q 878. The Nd: YAG laser emits energy at 1064nm near the infrared range and
penetrates the skin to the level of the deep dermal blood vessels. The target
chromophore for this laser is melanin, thus its primary use is in treating lesions such
as nevus of Ota and removing black tattoo pigment. Although both the erbium: YAG
and CO2 lasers have longer wavelengths than the Nd: YAG, they are ablative lasers
which, only penetrate to the level of the stratum corneum and superficial epidermis.

Q 879. The most common side effect seen with betadine is allergic contact
dermatitis, secondary to the iodine component.

Q 880. Needles are either straight or curved. Curved needles have their curvature
described either as a fraction of a circle or a compound curve. The greater the
fraction of a circle, the more pronation and supination of the wrist required by the
surgeon to place the needle. The 3/8 circle needle is easy to use in large, superficial
areas and is the most commonly used needle for cutaneous surgery

Q881. Imiquimod (Aldara) is FDA-approved for the treatment of actinic keratosis and
the treatment of superficial basal carcinoma. With respect to basal cell carcinoma,
treatment is indicated for primary tumors that are 2.0 cm or less, and that are
located on areas of the body excluding the face, scalp and anogenital region.
230
Q 883. Thrombotic complications in patients who stopped warfarin included stroke,
TIA, myocardial infarction, cerebral embolism, death, DVT, pulmonary embolus, and
blindness. Thrombotic complications in patients who stopped aspirin included
stroke, TIA, myocardial infarction, cerebral embolism and death. No DVT or
pulmonary embolus was reported

Q 884. Treatment of benign vascular lesions with the pulsed dye laser often
produces significant postoperative purpura. Topical vitamin K has been shown to
decrease the severity of laser-induced purpura although its mechanism of action
has yet to be determined. No other cosmetic effects of topical vitamin K have been
proven to be statistically significant.

Q 885. Laser light is monochromatic light (single wavelength). that is emitted at a


single wavelength. The wavelength of the laser is determined by the medium in the
optical cavity of the laser through which the light passes. The medium may be solid,
liquid or gas

Q 886. Following dermabrasion, reepithelialization from adnexal structures occurs


with remodeling and replacement of collagen bundles in the papillary and reticular
dermis. This can result in improvement of surgical or acne scars. When used in the
post-operative period for surgical scars, dermabrasion is usually, carried out 6 to 8
weeks following the procedure. Regional dermabrasions are routinely carried out
under local anesthesia.

Q887. The excimer laser is a 308 nm wavelength laser that has been used to
treat psoriasis. The excimer laser has some advantages over ultraviolet therapy. By
treating only involved skin, higher doses can be used and clearance may occur with
fewer treatments

Q 888. Many factors contribute to the aggressive behavior of cutaneous squamous


cell carcinoma. For example, tumors in immunosuppressed patients and tumors with
the histologic subtypes desmoplastic and acantholytic are often biologically
aggressive squamous cell carcinomas. Compared with other risk factors, depth of
invasion >4 mm and size > 2cm demonstrate the greatest risk for metastasis.

Q889. The carbon dioxide laser is a 10,600 nm laser that can be used to treat
rhinophyma. The advantage of the carbon dioxide laser over steel or dermabrasion is
that the laser is relatively bloodless. The erbium YAG can also be used to treat
rhinophyma.
Q890. Lichenoid reaction refer to a lesion similar or identical clinically and
histopathologically to lichen planus. Lichenoid tattoo reactions are not as common
as eczematous reactions, both of which are most commonly caused by mercuric

231
sulfide. Lichenoid reaction are likely to be mediated by delayed hypersensitivity to a
lymphocytic T-cell infiltrate

Q891. Five years cure rates of basal cell carcinomas (BCC) treated with MOHS is
about 99%. The other options listed range from 90-92% five year cure rates.

Q892.. Restylane is a hyaluronic acid gel produced by the Streptococcus species of


bacteria. The package insert for Restylane warns that its use is contraindicated in
patients with severe allergies manifested by a history of anaphylaxis or the presence
of multiple severe allergies

Q893. The erbium (2940 nm) and carbon dioxide (10,600 nm) lasers may both cause
milia formation after laser skin resurfacing

Q894. Silver sulfadiazine has broad antibacterial against gram positive and gram
negative including pseudomonas aeruginosa and some yeasts & fungi. It is most
commonly used to prevent infection in second and third-degree burn patients. Rare
cases of leukopenia, neutropenia and kernicterus have been reported in patients
using this topical antibacterial agent.

Q896. Pulsed dye laser (The 585 nm), targets intravascular oxyhemoglobin and is
considered the treatment of choice for most benign vascular lesions.

Q896. The carbon dioxide laser uses a 10,600 nm wavelength to target water as a
chromophore. Lasers that target water may damage the cornea. The carbon dioxide
laser can be used to destroy epidermal and dermal lesions such as warts. HPV virus
has been recovered in the laser plume after treatment with carbon dioxide laser.
Scarring and recurrence have also been found to be potential complications

Q897. All organ transplant recipients are at increased risk for the development
of cutaneous malignancies. However, the above-mentioned factors, with the
exception of CD8 lymphocytopenia, place these individuals at further risk. CD4
lymphocytopenia, rather than CD8, is another identified risk factor as well as older
age, history of actinic keratosis and history of skin cancer

Q898. The maximum recommended dosage of lidocaine in adults is 4.5 mg/kg


without epinephrine, 7.0 mg/kg with epinephrine, and 55 mg/kg in tumescent
anesthesia for liposuction

Q899. Lidocaine reduce pain by blocking nerve impulses that send pain sensation to
the brain. Lidocaine starts working within 90 seconds and effects last about 20
minutes. Beta-blockers increase lidocaine levels. 1% lidocaine is equal to 10 mg/ml.
Duration of lidocaine without epinephrine is 30-60 minutes. Maximum dose of
lidocaine without epinephrine is 4.5 mg/kg and with epinephrine it is 7 mg/kg.
Allergy to lidocaine is most commonly due to paraben preservatives.

232
Q900. Silicone is not FDA-approved for soft tissue augmentation. In addition to the
occurrence of hypersensitivity reactions and product migration, there have been
many reports of granuloma formation after silicone injection, even many years post-
treatment

Q901. Imiquimod belongs to a group of drugs called immune response modifiers. It is


believed to work by helping to activate the immune system to fight the abnormal
skin growth. It is a topical immunomodulator and is used in the treatment of genital
warts, actinic keratosis, and superficial basal cell carcinomas. It has a newly reported
side effect of eruptive epidermoid cysts.

Q903. The erbium:YAG laser is an ablative laser whose target chromophore is water.
The cornea is an aqueous structure and is therefore the most susceptible to damage
from this laser

Q904. The most serious adverse effect of prilocaine (EMLA) is dose-dependent


methemoglobinemia resulting in impaired oxygen-carrying capacity. Cyanosis may
occur 2-3 hours following administration. Patients with congenital or idiopathic
methemoglobinemia or infants under 1 year of age are at higher risk for the
development of this side effect. It is usually applied at least 1 hour before needle
stick and 2 hours before minor skin procedure

Q905. Reduce severity of postoperative purpura. Topical vitamin K reduced post


later treatment purpura when applied after (not before) treatment of telangiectasia
with the 585 nm pulsed dye laser

Q906. Imiquimod (Aldara) was FDA-approved in 2004 for the treatment of superficial
basal cell carcinoma. The recommended treatment schedule is once daily, five times
per week for a total of six weeks

Q907. Catgut has a variable rate of absorption but typically lasts about 7-14 days.
Thus, it would be the first suture to be absorbed.

Q908. The procerus muscle is the pyramid-shaped muscle extending from the lower
part of the nasal bone to the middle area in the forehead between the eyebrows,
where it is attached to the frontalis muscle. The procerus muscle causes wrinkling at
the nasal root and is often targeted with botox therapy for improved cosmetic
appearance.

Q909. The chemical peel is a skin re-surfacing procedure in which a chemical solution
is applied to the skin to remove the top layers of the skin. The skin that grows back
after peeling is smoother and younger looking. Many different types of side effects
can occur with chemical resurfacing and are typically related to the depth of the
wound. Complications which are common to all peeling agents include: milia, acne,
pigmentary changes, contact dermatitis, scarring, infection, prolonged erythema.
233
Unlike other peeling agents, phenol has the potential to cause severe adverse
reactions, which are exclusive to its use only. They include atrophy, cardiac
arrhythmias and laryngeal edema

Q910. The maximum dosage of lidocaine is 4.5 mg/kg without epinephrine and 7.0
mg/kg with epinephrine. Signs of lidocaine toxicity start with increased anxiety,
talkativeness, tinnitus, numbness/tingling around lips, metallic taste, and double
vision. Higher levels of toxicity may cause nystagmus, muscle twitching, tremor and
finally seizures and respiratory arrest.

234

Potrebbero piacerti anche